You are on page 1of 97

ReSA - THE REVIEW SCHOOL OF ACCOUNTANCY

CPA Review Batch 44  Oct 2022 CPALE  25 September 2022  11:45 AM - 02:45 PM

FINANCIAL ACCOUNTING and REPORTING FINAL PRE-BOARD EXAMINATION

INSTRUCTIONS: Select the correct answer for each of the questions. Mark only one
answer for each item by shading the box corresponding to the letter of your choice on
the answer sheet provided. STRICTLY NO ERASURES ALLOWED. Use pencil no. 2 only.

Multiple Choices – Theories:


1. Which of the following is not a purpose of the Conceptual Framework for Financial
Reporting according to the Revised Conceptual Framework for Financial Reporting?
a. To assist regulatory agencies in enforcing compliance of companies to the
requirements of the applicable IFRSs.
b. To assist all parties to understand and interpret the Standards.
c. To assist the International Accounting Standards Board (Board) to develop IFRS
Standards (Standards) that are based on consistent concepts.
d. To assist preparers to develop consistent accounting policies when no Standard
applies to a particular transaction or other event, or when a Standard allows a
choice of accounting policy.

2. The following statements concerning the qualitative characteristics of useful


financial information:
I.Verifiability, comparability, understandability and timeliness are qualitative
characteristics that enhance the usefulness of information that both is relevant
and provides a faithful representation of what it purports to represent.
II.To be useful, financial information must not only represent relevant phenomena,
but it must also faithfully represent the substance of the phenomena that it
purports to represent.

a. Only statement I is correct.


b. Only statement II is correct.
c. Both statements are correct.
d. Both statements are incorrect.

3. Cash equivalents do not include:


a. Two-month treasury bills
b. Commercial papers purchased with three months of maturity
c. Money market instrument (one month)
d. Equity investments at FV through profit or loss (expected to be sold in two
months)

4. Which of the following is recorded by a credit to accounts receivable but do not


affect the net realizable value of the accounts receivable?
a. estimated sales return at year-end.
b. provisions for estimated bad debts.
c. write-off of accounts receivable proven to be uncollectible.
d. sale of merchandise on account with 2/10, n/30 credit terms.

5. Which of the following shall be included in the reported balance of inventory in


the statement of financial position?
a. cost of goods shipped out on consignment to another entity.
b. cost of goods purchased with a buyback agreement.
c. cost of goods sold in transit – CIF terms.
d. cost of undelivered goods sold in bill and hold agreement.

6. Which of the following will not affect the investment income reported in profit or
loss if the equity investment is considered having significant influence?
a. share in excess representing increase in depreciation expense.
b. cash dividend received.
c. share in excess of inventories carrying value and fair value on acquisition date
when these inventories were sold during the year.
d. all of these affect the investment income.

7. What is the treatment of any gain on subsequent increase in fair value less cost of
disposal of a noncurrent asset classified as held for sale?
a. gain recognized directly in retained earnings.
b. gain recognized in profit or loss in full amount.
c. gain recognized in profit or loss limited to any previous cumulative impairment
losses.
d. gain shall not be recognized.

Page 1 of 22 0915-2303213  resacpareview@gmail.com


FINANCIAL ACCOUNTING & REPORTING
ReSA Batch 44 – October 2022 CPALE Batch
25 September 2022  11:45 AM to 02:45 PM FAR Final Pre-Board
Exam
8. Bri-PH Company acquired machinery on January 1, 2017 which it depreciated under the
straight-line method with an estimated life of fifteen years and no residual value.
On January 1, 2022, Bri-PH estimated that the remaining life of this machinery was
six years with no residual value. How should this change be accounted for by Bri-
PH?
a. as the cumulative effect of a change in accounting principle in 2022.
b. by setting annual depreciation equal to one-sixth of the book value on January
1, 2022 starting 2022.
c. by continuing to depreciate the machinery over the original fifteen-year life.
d. as a prior-period adjustment.

9. Eduardo Company constructs a laboratory building to be used as a research and


development facility, the cost of the laboratory building is matched against
earnings as?
a. depreciation reported as part of research and development cost at such time as
productive research has been obtained from the facility.
b. depreciation or immediate write-off depending on company policy.
c. reduction in the reported research and development expense in the year of
construction.
d. research and development expense in the period of construction.

10. Accounting for product warranty costs under an assurance-type warranty:


a. is required for income tax purposes.
b. charges an expense account when the seller performs in compliance with the
warranty.
c. is frequently justified on the basis of expediency when warranty costs are
immaterial.
d. represents accepted practice and should be used whenever the warranty is an
integral and inseparable part of the sale.

11. Elijah Company’s December 31, 2022 Statement of Financial Position is to be issued
of April 15, 2023. A long-term obligation contracted in 2020 for settlement on
January 15, 2023 was extinguished through cash payment on its due date. On January
1, 2023, a 5-year note was issued to replace the cash used up for the payment made
on April 15, 2023. Which of the following statements is correct?
a. There should be no liability to be reported in the 2022 statement of financial
position since the original obligation was already extinguished before the date
of the authorization for issuance.
b. The new obligation entered into on January 2023 should be reported in the 2022
statement of financial position as a non-current liability because it is due to
be settled beyond twelve months after the reporting period.
c. The original obligation should be reported in the 2022 statement of financial
position as a non-current liability because the entity has a right to defer
settlement of the liability for at least twelve months after the reporting period.
d. The original obligation should be reported in the 2022 Statement of Financial
Position as a current liability because the entity does not have a right to defer
settlement of the liability for at least twelve months after the reporting period.

12. Ivan Company had the following items of liability in its book as of December 31,
2022. Determine the correct classification of the following liabilities in the
Statement of Financial Position of Ivan Company as of December 31, 2022:
[1.] Liability with a due date which can be accelerated to within one year of the
statement of financial position date; a reasonable probability exists that the
due date will be accelerated.
[2.] Liability due on demand by creditor, probability of the creditor calling the
in the liability within one year of the statement of financial position date
is remote.
[3.] Liability due on demand by creditor, probability of the creditor calling the
in the liability within one year of the statement of financial position date
is reasonable but not likely.
[4.] Bonds issued due in three years from date of issuance and currently maturing
in 10 months after reporting period.

a. Only numbers 1 and 4 are current.


b. Only numbers 2 and 3 are current.
c. All are non-current liabilities.
d. All are current liabilities.

Page 2 of 22 0915-2303213  resacpareview@gmail.com


FINANCIAL ACCOUNTING & REPORTING
ReSA Batch 44 – October 2022 CPALE Batch
25 September 2022  11:45 AM to 02:45 PM FAR Final Pre-Board
13. Laurence Company issued a P100,000, 10% bond at 99. The bond was: Exam
a. sold for P100,000 less P1,000 of accrued interest.
b. sold at a premium because the stated rate was higher than the yield rate.
c. sold at a discount because the stated rate was lower than the market interest
rate.
d. sold at a premium because the P1,000 accrued interest is added to the P100,000
face amount.

14. Identify the missing component (X) in the equation:

Retained Earnings, end = Net income to date + period-period adjustments – cash


dividends to date – property dividends to date – X

a. net unrealized loss on equity investmens at FV through OCI.


b. Share dividends declared to date.
c. Share split declared to date.
d. All of them may be the missing component.

15. Which of the following treasury share transactions do not affect the total
shareholder’s equity?
[1.] Acquisition of treasury shares.
[2.] Reissuance of treasury shares.
[3.] Declaration of treasury shares as dividends.
[4.] Retirement of treasury shares.

a. Number 1 only
b. Numbers 2 and 3 only
c. Numbers 3 and 4 only
d. Numbers 2, 3 and 4 only.

16. Danielle Company made the following journal entry on January 1, 2022 related to a
lease contract:

Lease receivable xx
Cost of sales xx
Sales xx
Inventory xx
Unearned interest income xx

Danielle Company must have a (an)?


a. sales-type lease
b. direct finance lease
c. operating lease
d. sale-leaseback transaction

17. Which of the following is included in the cost of the right-of-use asset of the
lessee?
a. free rent on the first year of lease contract granted by the lessor.
b. initial direct costs to effect the lease paid by the lessor.
c. reimbursements by the lessor of the lessee’s initial direct costs.
d. initial payment made by the lessee to the lessor at inception of the lease.

18. Sheen Company maintained a defined benefit plan for all employees. The service cost
component of Sheen Company shall exclude:
a. increase in the present value of a defined benefit obligation resulting from
employee service in the current period.
b. the change in the present value of the defined benefit obligation for employee
service in prior periods as a result of plan amendments.
c. decrease in the present value of defined benefit obligation due to effects of
changes in actuarial assumptions.
d. increase during a period in the present value of a defined benefit obligation
which arises because the benefits are one period closer to settlement.

19. The current income tax expense (CITE) is:


a. financial income multiplied by current year tax rate.
b. financial income multiplied by future enacted tax rate.
c. taxable income multiplied by future enacted tax rate.
d. taxable income multiplied by current year tax rate.

Page 3 of 22 0915-2303213  resacpareview@gmail.com


FINANCIAL ACCOUNTING & REPORTING
ReSA Batch 44 – October 2022 CPALE Batch
25 September 2022  11:45 AM to 02:45 PM FAR Final Pre-Board
Exam
20. Under PFRS 8, which of the following is not a criterion used to determine reportable
segments?
a. Segment assets
b. Segment liabilities
c. Segment sales
d. Segment operating profit or loss

Multiple Choices – Problems

Situation 1 – The data relates to three different companies:


On December 31, 2022, Villanueva Company reported cash and cash equivalents of
P5,255,000 comprised of the following:
Petty cash fund (imprest balance) P5,000
Undeposited collections 120,000
Cash in Bank - JPBank checking account 900,000
Cash in Bank – JPBank payroll fund 1,950,000
Cash in Bank - JPBank dividend fund 750,000
Cash in Bank - Binaluyo Bank of Asia (Foreign currency) 800,000
Cash in Bank - money market instrument, 60 days 250,000
Redeemable preference shares, due in 2 months as of 12/31/22 480,000
Total cash and cash equivalents P5,255,000

Additional information as of December 31, 2022:


The petty cash fund includes unreplenished vouchers amounting to P2,350 and P2,550 of
bills and coins. Included in the undeposited collections were: NSF check received from
customer on December 29, 2022 for P15,000 and P20,000 of customers check received on
December 30, 2022 dated January 4, 2023. Included in the checks drawn and recorded
against JP Bank checking account were: P180,000 payable to Moira Company dated January
3, 2023 and P145,000 of check issued to Bellie Company dated June 15, 2022 (not encash
as of 12/31/2022). The $16,000 cash in Binaluyo Bank of Asia is not restricted, closing
rate on December 31, 2022 is P56 =$1. The redeemable preference shares were originally
purchased by Villanueva Company last November 1, 2022.

Tan Company had the following transaction in its accounts receivable during 2022:

Accounts receivable, beginning balance P2,700,000


Allowance for bad debts, beginning 33,750
Sales on account made to customers 3,950,000
Collection of accounts from credit customers 4,350,000
Sales discounts granted 50,000
Accounts written off as uncollectible 80,000

The following transactions were included in the recorded sales on account during 2022:
[1.] Invoice dated December 28, 2022 for P420,000 was shipped and received by the buyer
on December 31, 2022, terms FOB Destination point.
[2.] Invoice dated and recorded on November 29, 2022 for P600,000 was received by
another entity on December 1, 2022. The agreement of the parties is to remit the
selling price less commission upon sale of the goods. None of the goods have been
sold yet as of December 31, 2022.
The following summary was prepared from an aging of accounts receivable outstanding on
December 31, 2022:
Number of days outs. % of the ending balance % uncollectible
0 - 30 days 50% 2%
31 - 60 days 30% 5%
Over 60 days 20% 15%

Pyrus Company conducted a physical count on December 31, 2022, which revealed inventory
with a cost of P4,410,000. The following items were excluded from the physical count:
Goods held by Pyrus on consignment P700,000
Goods shipped by Pyrus FOB Destination to a customer on December 31,
2022 and was received by the customer on January 3, 2023 400,000
Goods shipped by Pyrus FOB Shipping Point to a customer on December 31,
2022 and was received by the customer on January 6, 2023 420,000
Goods shipped by a vendor FOB Destination on December 31, 2022 and was
received by Pyrus on January 10, 2023 910,000
Goods purchased FOB Shipping Point was shipped by the supplier on
December 31, 2022 and received by Pyrus on January 5, 2023 640,000

Page 4 of 22 0915-2303213  resacpareview@gmail.com


FINANCIAL ACCOUNTING & REPORTING
ReSA Batch 44 – October 2022 CPALE Batch
25 September 2022  11:45 AM to 02:45 PM FAR Final Pre-Board
Exam
21. What amount should Villanueva Company report as cash and cash equivalents on December
31, 2022?
a. P5,638,650 c. P5,255,550
b. P5,542,650 d. P5,158,550

22. What amount should Tan Company recognized as bad debt expense in profit or loss for
period ending December 31, 2022?
a. P132,600 c. P86,350
b. P120,100 d. P56,750

23. What amount should Pyrus Company report as inventory as of December 31, 2022?
a. P2,160,000 c. P5,450,000
b. P4,350,000 d. P5,640,000

Situation 2 – Information relevant to three different companies follows:


Thirdy Company had the following portfolio of financial instrument as of December 31,
2022. All securities were acquired at the end of 2021:
Security Denomination Initial recorded value
First shares 200,000 shares P11,000,000
Second shares 20,000 shares 900,000
[1.] First shares were acquired and designated as equity investment at fair value
through profit or loss. The shares were acquired at P55 per share which included
P1.00 per share transaction cost. Half of the First shares were sold at P62 per
share on September 30, 2022. Each share is selling at P65 on December 31, 2022.
[2.] Second shares were acquired and designated as equity investments at FV through
other comprehensive income. The shares were purchased at P45 including P1.00
per share transaction cost. The FV of each share on December 31, 2022 was at
P50.

At the beginning of 2022, Tristan Co. purchased 250,000 ordinary shares of Daniella
Co. at P45 per share, giving Tristan 40% ownership and a significant influence over
Daniella. On this date, the book value Daniella’s net asset amounted to P23,125,000.
On the date of acquisition, the carrying amount of Daniella’s identifiable assets and
liabilities approximate their carrying amounts, except for the following:
[1.]Aggregate fair value of Daniella’s depreciable property, plant, and equipment is
P2,500,000 greater than its carrying value. Such item is depreciated over 8-
years remaining useful life on the date of acquisition.
[2.]The fair value of Daniella’s inventories was P2,000,000 greater than its carrying
amount. All of these inventories were sold in 2022.
[3.]A land costing P1,000,000 had fair value of P1,500,000.

During the year, the Daniella earned and reported net income of P10,200,000, and paid
cash dividends of P1,800,000 to Tristan Company.

Lance Company acquired on January 1, 2022 a 5 year, 8%, P5,000,000 face value bonds,
for P4,445,615 dated January 1, 2022. The bonds which pay interest every December 31
had 11% prevailing interest rate on the date of acquisition. Lance’s business model is
to collect contractual cash flows and the cash flows are solely payment of principal
and interest and to sell the bonds when circumstances warrants. The prevailing interest
rate on December 31, 2022 is 10%.

24. What amount should Thirdy Company report as unrealized gain (loss) in its Statement
of Comprehensive Income for the period ending December 31, 2022?
a. P100,000 c. P1,200,000
b. P1,000,000 d. P1,220,000

25. What amount should Tristan Company report its investment in the Statement of
Financial Position as of December 31, 2022?
a. P12,605,000 c. P14,300,000
b. P13,685,000 d. P14,315,000

26. What amount should Lance Company report its debt investment in the Statement of
Financial Position as of December 31, 2022?
a. P5,450,000 c. P4,683,013
b. P4,696,128 d. P4,534,633

Page 5 of 22 0915-2303213  resacpareview@gmail.com


FINANCIAL ACCOUNTING & REPORTING
ReSA Batch 44 – October 2022 CPALE Batch
25 September 2022  11:45 AM to 02:45 PM FAR Final Pre-Board
Situation 3 – Information relevant to four different companies follows: Exam
Love Company reported an impairment loss of P500,000 in its income statement for the
year 2019. This impairment loss was related to an item of property, plant, and equipment
(a building) which was acquired on January 1, 2011 with a cost of P4,000,000 (no
salvage value). Depreciation on this building is computed on straight-line basis and
annual depreciation on cost is P160,000. Depreciation for the years 2020 and onwards
were computed based on the asset’s recoverable amount at December 31, 2019. On December
31, 2022, the fair value of the building is P3,240,000. Love Company uses revaluation
model to account this class of property, plant and equipment.

Thunder Company constructed its own building which qualifies for interest
capitalization. Thunder incurred the following cost and had the following outstanding
borrowings while the building is under construction.
January 1, 2021 P3,000,000
June 30, 2021 2,500,000
November 30, 2021 2,000,000
April 1, 2022 1,500,000
October 1, 2022 1,000,000

Outstanding borrowings:
Dated January 1, 2021 - 2,000,000; 8% (specific)
Dated January 1, 2021 - 3,000,000; 9% (general)
Dated January 1, 2022 - 3,875,000; 12% (general)

The construction was completed on December 31, 2022 and ready for its intended use.

Thor Company has a single investment property, which had original cost of P580,000 on
January 1, 2021. At December 31, 2021, its fair value was P600,000 and at December 31,
2022, it had a fair value of P590,000. On acquisition, the property had a useful life
of 40 years.

Mjolnir Company was able to patent one of its new machines with the Intellectual
Property Office of the Philippines on January 2, 2022. The cost of the patent recorded
by the client included the following items:
Purchase of special equipment for use in operations (5-year EUL) P1,500,000
Research salaries and fringe benefits for engineers and scientists 540,000
Cost of testing pre-production prototype 890,000
Legal cost of filing for patent 520,000
Fees paid to government patent office 260,000
Drawings required by patent office to be filed with patent application 145,000
The patent’s economic life is 10 years.

27. What amount should Love Company recognized in profit or loss as gain on recovery of
impairment and the balance of revaluation surplus, respectively in year 2022?
a. P93,750 and 1,566,250 c. P502,250 and P1,566,250
b. P406,250 and P1,160,000 d. P612,500 and P1,160,000

28. What is the capitalized interest, interest expense and cost of building should
Thunder report in its Financial Statements for year 2022?
a. P935,358 (capitalized interest); P11,312,858 (cost of building); P40,358
(interest expense)
b. P935,358 (capitalized interest); P11,272,858 (cost of building); P25,358
(interest expense)
c. P895,000 (capitalized interest); P11,312,500 (cost of building); P40,358
(interest expense)
d. P895,000 (capitalized interest); P11,272,500 (cost of building); P0 (interest
expense)

29. What should be the total expense recognized by Thor Company in the profit or loss
for the year ended December 31, 2022 under fair value and cost model, respectively?
a. P10,000 and P14,500, respectively. c. P14,500 and P10,000, respectively.
b. P14,750 and P14,500, respectively. d. P10,000 and P14,750, respectively.

30. What amount should Mjolnir Company report as patent in its Statement of Financial
Position as of December 31, 2022, and research and development expense in its 2022
profit or loss, respectively?
a. P925,000 and P2,930,000 c. P693,750 and P2,040,000
b. P832,500 and P1,430,000 d. P693,750 and P1,730,000
Page 6 of 22 0915-2303213  resacpareview@gmail.com
FINANCIAL ACCOUNTING & REPORTING
ReSA Batch 44 – October 2022 CPALE Batch
25 September 2022  11:45 AM to 02:45 PM FAR Final Pre-Board
Situation 4 – Information relevant to four different companies follows: Exam
Candy Crush Company offers a coffee mug as a premium for every ten candy bar wrappers
presented by customers. Each candy bar is sold at P5. The purchase price of each mug
to the company is P1. Each mug can be sold for P3 if not use in premium promotional
program. The results of the premium plan for the years 2021 and 2022 are as follows:

All in units 2021 2022


Coffee mugs purchased 720,000 800,000
Candy bars sold 5,600,000 6,750,000
Total wrappers redeemed in 2,800,000 4,200,000
2021 wrappers expected to redeem in 2022 2,000,000
2022 wrappers expected to redeem in 2023 2,700,000

A customer is suing Lock Heart Company for P550,000 in damages because her child was
injured in November 2022 while riding an escalator that stopped suddenly in one of its
state-of-the-art stores in Makati when he tripped and fell while walking down an
escalator that was going up. Legal counsel that it is probable that the lawsuit will
be settled for between P200,000 and P550,000, with the following likely outcome: 20%
to be settled at P200,000; 30% to be settled at P370,000 and 50% to be settled at
P550,000.

Ludwig Company started its business in selling printers with three-year warranty. It
estimates its warranty cost as a percentage of peso sales. Based on past experience,
it is estimated that 3% will be repaired during the first year of warranty, 5% will be
repaired during the second year of warranty and 7% will be repaired in the third year.
The product warranty provides assurance based on agreed upon specification. In 2021
and 2022, the company was able to sell 12,000 units and 14,500 units, respectively at
a total price of P7,000 per unit. The company also incurred actual repair costs of
P6,500,000 and P12,500,000 in 2021 and 2022, respectively. The selling price of the
warranty is P2,000 per unit. The printer is selling at P5,000 if without the warranty.

Graceful Corp. has a three-year, 10%, long-term bond for P3,500,000 with a maturity
date of November 1, 2025. On October 1, 2022, it breaches a covenant related to this
debt and the loan becomes due on demand. Graceful reaches an agreement with the lender
on January 2, 2023, to provide a waiver of the breach not to demand payment until
December 31, 2023. The financial statements are authorized for issue on March 5, 2023.

31. What amount should Candy Crush Company report as unearned premium income in its
December 31, 2021 and December 31, 2022 Statement of Financial Position,
respectively?
a. P460,623 and P748,678 c. P570,652 and P776,193
b. P520,772 and P727,281 d. P582,352 and P730,660

32. What amount should Lock Heart Company shall reflect the situation in its December
31, 2022 Financial Statements?
a. P426,000 recognized as provision and P124,000 disclosed as contingent liability.
b. P373,333 recognized as provision and P176,667 disclosed as contingent liability.
c. P550,000 recognized as provision and no disclosure for contingent liability.
d. P550,000 recognized as provision and P200,000 disclosed as contingent liability.

33. What amount should Ludwig Company shall report the balance of warranty liability as
of December 31, 2022?
a. P7,625,000 c. P9,125,000
b. P8,825,000 d. P9,375,000

34. How should Graceful Corp. shall report the situation in its Financial Position as
of December 31, 2022?
a. Graceful shall report the bond payable as non-current because a grace period was
obtained related to the debt.
b. Graceful shall report the bond payable as non-current because a grace period was
not obtained related to the debt.
c. Graceful shall report the bond payable as current because a grace period was
obtained related to the debt regardless of the date of when the grace period was
obtained.
d. Graceful shall report the bond payable as current because a grace period was
obtained related to the debt after the end of the reporting period.

Page 7 of 22 0915-2303213  resacpareview@gmail.com


FINANCIAL ACCOUNTING & REPORTING
ReSA Batch 44 – October 2022 CPALE Batch
25 September 2022  11:45 AM to 02:45 PM FAR Final Pre-Board
Situation 5 – Information relevant to three different companies follows: Exam
On January 1, 2021 Matthew Company bought an equipment by paying cash of P2,000,000
and issued a 10% 4-year note payable in equal annual installment of P1,000,000 at the
end of each year. Interest is payable at each year-end based on outstanding balance of
the note. At issuance date, Matthew recorded the transaction by recognizing a discount
on note payable of P160,442.

On December 31, 2022, Matthew recorded a credit to discount on notes payable of P48,037
when required payment of principal and interest due on this date was made. At the end
of 2023, the balance of unamortized discount is P17,857.

Solaire Corporation issued P1,500,000 of 8% bonds on October 1, 2020 due on October 1,


2023 at 105. The interest is to be paid twice a year on April 1 and October 1. When
the bonds was issued, the prevailing market rate was 10% without the conversion
privilege. The corporation closes its books annually on December 31. Each P1,000 bond
is convertible into 10 shares of P100 par value ordinary share. The bonds were retired
on April 1, 2023 at 102 and on this date, the prevailing market rate was 9% without
the conversion privilege.

City of Dreams Company has an overdue note payable to ReSA Bank of P10,000,000 and
recorded accrued interest of P1,000,000. On December 31, 2022, ReSA Bank agreed to the
following restructuring agreement:
• Reduce the principal obligation by P3,000,000
• Waive the P1,000,000 accrued interest
• Extend the maturity date to December 31, 2025.
• Annual interest of 8% of the new principal is to be paid on December 31, 2023,
December 31, 2024 and December 31, 2025.
• The prevailing market interest rate for similar debt instrument on the date of
restructuring is 12%

35. What is the carrying value of the bonds payable reported in Matthew Company’s
statement of financial position as of December 31, 2022?
a. P2,003,281 c. P1,936,816
b. P1,948,342 d. P1,922,162

36. How much gain (loss) should Solaire record upon retirement of these bonds?
a. P7,109 loss c. P37,177 loss
b. P7,109 gain d. P37,177 gain

37. How much is the gain on debt restructuring should City of Dreams recognized in
profit or loss?
a. P4,348,169 c. P4,592,669
b. P4,469,372 d. P4,672,512

Situation 6 – Information relevant to three different companies follows:


Presented below is the equity section of Coca-Cola Corporation at December 31, 2022:
Share capital—ordinary, par value P20; authorized
75,000 shares; issued and outstanding
45,000 shares P 900,000
Share premium—ordinary 250,000
Retained earnings 500,000
Total Shareholder’s equity P1,650,000

During 2022, the following transactions occurred relating to equity:


3,000 shares were reacquired at P28 per share.
3,000 shares were reacquired at P35 per share.
1,800 shares of treasury shares were sold at P30 per share.

For the year ended December 31, 2022, Coca-cola reported net income of P450,000.

Blue Corporation has 50,000 shares of P10 par ordinary shares authorized. The following
transactions took place during 2022, the first year of the corporation’s existence:
Sold 5,000 ordinary shares for P18 per share.
Issued 5,000 ordinary shares in exchange for a patent with fair value of P100,000.

Page 8 of 22 0915-2303213  resacpareview@gmail.com


FINANCIAL ACCOUNTING & REPORTING
ReSA Batch 44 – October 2022 CPALE Batch
25 September 2022  11:45 AM to 02:45 PM FAR Final Pre-Board
Malaki Corporation has an investment in 5,000 shares of Wallace Company ordinaryExam
shares
with a cost of P218,000. These shares are used in a property dividend to shareholders
of Malaki. The property dividend is declared on May 25 and scheduled to be distributed
on July 31 to shareholders of record on June 15. The market value per Wallace share is
P63 on May 25, P66 on June 15, and P68 on July 31.

38. Assuming Coca-cola accounts for treasury under the cost method, what amount should
it report as total equity on its December 31, 2022, statement of financial position?
a. P1,965,000 c. P1,985,000
b. P1,975,000 d. P1,995,000

39. At the end of the Blue Corporation’s first year of operations, total contributed
capital amounted to?
a. P180,000 c. P200,000
b. P190,000 d. P210,000

40. The net effect of property dividend declaration on retained earnings of Malaki
Corporation for year ended December 31, 2022 is a reduction of?
a. P218,000 c. P330,000
b. P315,000 d. P340,000

Situation 7 – Information relevant to three different companies follows:


On January 1, 2022, One Direction Corporation signed a ten-year noncancelable lease
for certain equipment. The terms of the lease called for One Direction to make annual
payments of P150,000 at the end of each year for ten years with title to pass to One
Direction at the end of lease term. The equipment has an estimated useful life of 15
years and no residual value. One Direction uses the straight-line method of depreciation
for all of its fixed assets. One Direction accordingly accounted for this lease
transaction as a finance lease. The interest rate of 8% is implicit in the lease. One
Direction has an option to purchase the asset at the end of lease term at P120,000
which is reasonably certain to exercise by One Direction. Estimated residual value at
the end of 10 years is P100,000 and at the end of 15 years is P80,000. One Direction
incurred a total of P350,000 direct cost to enter the lease.

On January 1, 2022, Harry Styles Company enters into a seven-year lease of machinery.
The machinery has estimated useful life of 10 years. Lease payments are P2,220,000 per
year all payable at the beginning of each year. To obtain the lease, Harry Styles
Company incurs initial direct cost of P500,000. Harry Styles guarantees a residual
value of P400,000 at the end of lease term. The machine is expected to dismantle at a
cost of P75,000 after the lease contract.

Implicit rate in the lease contract 11%


Effective rate of interest (dismantling) 12%

On January 1, 2022, Zayn Corporation (lessor) enters into a ten-year lease of equipment
to Malik Corporation (lessee). Malik Corporation do not have enough cash for an outright
purchase of equipment. Zayn Corporation use the lease contract to sell the asset to
Malik Corporation. The equipment has estimated useful life of 15 years. Lease payments
are P380,000 per year all payable at the beginning of each year. The fair value of the
leased asset on this date P3,132,161. The rate implicit in the lease is 6%. Mulak
Corporation, a subsidiary of Malik Corporation guaranteed a residual value of P300,000.
The cost of the asset in the book of Zayn Corporation was P2,500,000.

41. One Direction should record for 2022:


a. lease expense of P150,000.
b. interest expense of P85,681 and depreciation expense of P131,210.
c. interest expense of P80,521 and depreciation expense of P85,101.
d. interest expense of P84,968 and depreciation expense of P88,806.

42. How much should Harry Styles Company report the right-of-use asset as of December
31, 2023 and the amount of depreciation expense for year 2023, respectively?
a. P8,257,132 and P1,075,483 c. P8,105,475 and P1,541,095
b. P8,927,417 and P1,705,483 d. P8,107,045 and P1,541,409

43. How much should Zayn Corporation report the balance of lease receivable – net as of
December 31, 2022 and the gross profit recognized at inception of the lease?
a. P2,917,291and P632,161 c. P2,686,921 and P799,681
b. P2,955,789 and P632,161 d. P2,739,722 and P799,681

Page 9 of 22 0915-2303213  resacpareview@gmail.com


FINANCIAL ACCOUNTING & REPORTING
ReSA Batch 44 – October 2022 CPALE Batch
25 September 2022  11:45 AM to 02:45 PM FAR Final Pre-Board
Situation 8 – Information relevant to three different companies follows: Exam
Liam Company reported pretax income of P6,400,000 for the year ended December 31, 2022.
The company record shows the following differences:
Tax depreciation in excess of book depreciation P200,000
Fines and penalties 60,000
Proceeds from life insurance policy upon death of an officer* 340,000
Interest revenue on bank deposits 125,000
Impairment loss on goodwill 34,000
Provision for litigation expected to settle in 2023 and 2024 in
140,000
equal amounts
Installment sales recognized in the book to be collected 60% in
400,000
2023 and 40% in 2024
*The beneficiary of the insurance policy is Liam Company.

Tax rate is 25% in 2022 and 30% in the future as enacted by the congress. Payments in
previous quarters totaled P350,000.

Niall Incorporated’s partial income statement after its first year of operation (2022)
is as follows:
Income tax expense:
Current P1,050,000
Deferred 100,000
Total income tax expense P1,150,000

Niall Incorporated uses straight-line method of depreciation for financial reporting


purposes and accelerated depreciation method for tax purposes. The amount charged to
depreciation expenses on its book this year was P1,500,000. There were penalties paid
by Niall amounting to P240,000 because of late filing in BIR in previous quarters. No
other temporary differences existed between book income and taxable income except for
the amount of depreciation. tax rate is 25%.

On January 1, 2022, Tomlinson Company reported the fair value of plan assets at
P8,000,000 and defined benefit obligation at P7,100,000. Transactions affecting the
balances for the current year are as follows:

Current service cost P1,410,000


Past service cost 730,000
Contribution to the plan 2,880,000
Benefits paid to retirees on scheduled retirement date 2,000,000
Benefits paid to retirees earlier scheduled retirement date 650,000
Carrying value of benefit obligation of early retired employees 600,000
Actual return on plan assets ?
Actuarial loss on plan asset 45,000
Increase in defined benefit obligation due to changes in actuarial
assumption 360,000
Rate of return on high-quality corporate bonds 11%
Rate of return on government bonds 11.5%

44. How much should Liam Company report the deferred tax asset and deferred tax liability
as of December 31, 2022, respectively?
a. P42,000 and P185,000 c. P42,000 and P180,000
b. P85,000 and P100,000 d. P80,900 and P93,600

45. What amount should Niall Incorporated’s report as net income after in its Income
Statement for the year 2022?
a. P2,890,000 c. P3,290,000
b. P3,450,000 d. P3,210,000

46. How much is the balance of the FV of plan asset and defined benefit obligation as
of December 31, 2022 disclosed in Tomlinson’s Financial Statement?
a. P9,115,000 and P7,781,000 c. P9,065,000 and P7,831,000
b. P9,065,000 and P7,781,000 d. P9,115,000 and P7,831,000

Page 10 of 22 0915-2303213  resacpareview@gmail.com


FINANCIAL ACCOUNTING & REPORTING
ReSA Batch 44 – October 2022 CPALE Batch
25 September 2022  11:45 AM to 02:45 PM FAR Final Pre-Board
47. Tomlinson Company: Exam
Statement 1: Total benefit expense for the year ending December 31, 2022 is
P2,496,000.
Statement 2: The amount presented in the statement of financial position as of
December 31, 2022 is P1,334,000 net defined benefit asset.
a. Only statement 1 is correct c. Both statements are correct
b. Only statement 2 is correct d. Both statements are incorrect
Situation 9 – Information relevant to two different companies follows:
Mighty Thor Corporation’s December 31, 2022 balance sheet reports the following
shareholders’ equity:
10% Cumulative Preference share capital, P100 par value per share,
30,000 shares issued and outstanding, liquidation value of P105 P3,000,000
Ordinary share capital, P100 par value, 60,000 shares issued 6,000,000
Share premium 500,000
Treasury shares, (ordinary) 5,000 shares at cost 600,000
Retained Earnings 4,000,000
Subscribed ordinary share (10,000 shares subscribed,
net of P400,000 subscription receivable 1,000,000
Revaluation surplus 700,000

Preference dividends have not been paid since last year up to the end of 2022.

48. What is the book value per share on ordinary share?


a. 173.08 c. P166.92
b. 163.04 d. P157.25

49. What is the book value per share on preference share?


a. 105.00 c. P120.00
b. 115.00 d. P125.00

Groot Corp. had P500,000 net income in 2022. On January 1, 2022, there were 200,000
shares of ordinary outstanding. On April 1, 20,000 shares were issued and on September
1, bought 30,000 shares of treasury shares. There are 30,000 options to buy ordinary
shares at P40 per share. The market price of the ordinary shares averaged P50 during
2022. The tax rate is 40%.

During 2022, there were 40,000 shares of cumulative preference shares outstanding. The
preference has P100 par, pays dividend of P3.50 per year, and is convertible into three
shares of ordinary. Groot issued P2,000,000 of 8% convertible bonds at face value
during 2021. Each P1,000 bond is convertible into 20 shares of ordinary.

50. How much is the basic earnings per share for 2022?
a. P1.71 c. P1.60
b. P1.76 d. P1.17

51. How much is the diluted earnings per share for 2022?
a. P1.71 c. P1.51
b. P1.68 d. P1.46

52. On January 2, 2021, Cleo Company receives a government loan of P2,000,000 paying a
coupon interest of 1% per year. The loan is repayable at the end of year 6. Cleo
Company’s borrowing cost is 7% per annum. The below-market interest is provided by
the government to enable Cleo Company to bear cost of 1% per annum on the nominal
value of the loan. Cleo Company should recognize income from government grant in
2022 amounting to?
a. P85,558 c. P87,261
b. P86,625 d. P88,972

Use the following information for the next two questions:


The shareholders' equity section of Peter Corporation as of December 31, 2021, contained
the following accounts:
Ordinary share capital, P20 par, 4,500,000 shares P4,500,000
authorized; 225,000 shares issued and outstanding
Share premium 2,500,000
Retained earnings 7,200,000

Page 11 of 22 0915-2303213  resacpareview@gmail.com


FINANCIAL ACCOUNTING & REPORTING
ReSA Batch 44 – October 2022 CPALE Batch
25 September 2022  11:45 AM to 02:45 PM FAR Final Pre-Board
Peter Corporation’s board of directors declared a 10 Exam
percent bonus issue on April 1,
2022, when the market value of the share was P24 per share. Accordingly, new shares
were issued. Another P2.50 per cash dividends were declared on September 1, 2022 and
the equipment with carrying value of P650,000 currently having fair value of P720,000
was declared as dividends on December 1, 2022. Peter Corporation sustained a net loss
of P810,000 for the year ended December 31, 2022.

53. What amount should Peter Corporation report as retained earnings as of December 31,
2022?
a. P4,657,500 c. P4,511,250
b. P4,567,500 d. P5,321,250
54. How much is the total shareholder’s equity should Peter Corporation in its December
31, 2022 Statement of Financial Position?
a. P11,511,250 c. P13,151,250
b. P12,051,250 d. P13,671,250

Use the following information for the next two questions:


John Corp.'s transactions for the year ended December 31, 2022 included the following:
• Issued 500 of its 11% debenture bonds, due 2026, for P392,000 cash.
• Purchased a patent for P220,000 cash.
• Paid P120,000 bank loan to BDO.
• Sold an equity investment at FVTOCI for P796,000.
• Sold treasury shares with cost of P70,000 for P95,000.
• Acquired 50% of Ford Corp.'s ordinary shares for P180,000 cash which was borrowed
from a bank.
• Issued 5,000 shares of its preference shares for land having a fair value of
P320,000.
• Sold an old equipment having carrying value of P210,000 receiving P100,000, 10%
notes and P110,000 in cash.

55. John’s net cash provided by investing activities for 2022 was
a. P826,000 c. P396,000
b. P506,000 d. P296,000

56. John’s net cash provided by financing activities for 2022 was
a. P325,000 c. P445,000
b. P367,000 d. P547,000

Situation 10 – Information relevant to three different companies follows:


57. Juancho Company had a net income after tax of P3,500,000 for the year ended December
31, 2022 after giving effect to the following events which occurred during the year.
The decision was made on May 31 to discontinue the bottles manufacturing segment.
The bottle manufacturing segment was sold on June 30. Operating profit from January
1 to May 30 for the bottle manufacturing segment amounted to P950,000 before tax.
Bottle manufacturing equipment with a book value of P1,900,000 was sold for
P1,150,000. The tax rate was 30%. For the year ended December 31, 2022, how much
was the company’s after-tax income from continuing operations?
a. P3,300,000 c. P3,440,000
b. P3,360,000 d. P3,500,000

58. An equipment was purchased by Miguel Company on January 1, 2020 for P5,000,000 with
estimated useful life of 10 years and no salvage value. On January 1, 2022, the
entity classified the asset as held for sale in accordance with PFRS 5. As of this
date the fair value of the asset is P3,300,000 and cost to sell is P100,000. As of
December 31, 2022, the entity believes that the criteria for classification as held
for sale can no longer be met. The fair value of the equipment is P3,800,000 and
cost to sell is P200,000 as of December 31, 2022 while the value in use amounted to
3,200,000. At what amount should the asset be reported in its 2022 statement of
financial position?
a. P3,200,000 c. P3,700,000
b. P3,500,000 d. P3,800,000

59. Nikko Company has several manufacturing plants all over the country. On December
29, 2022, a super typhoon hit the province of Bicol where one of the entity’s large
and major manufacturing plant is located. Because of the damages caused by the
calamity, the entity decided to abandon the plant which constitute a major line of
business. All work stops at the manufacturing plant during the year ended 2022.

Page 12 of 22 0915-2303213  resacpareview@gmail.com


FINANCIAL ACCOUNTING & REPORTING
ReSA Batch 44 – October 2022 CPALE Batch
25 September 2022  11:45 AM to 02:45 PM FAR Final Pre-Board
The carrying amount of the entire manufacturing Exam
plant amounted only to P2,000,000
as of the end of the year. The operations of this manufacturing plant managed to
generate P100,000 profit from operations before tax. The prevailing tax rate was at
30%. The fair value less cost to sell was P1,980,000 at the end of the year. How
much should be reported as non-current asset held for sale as of December 31, 2022?
a. P0 c. P2,000,000
b. P1,980,000 d. P3,980,000

60. How much should be the reported net income of Nikko Company for the year ending
December 31, 2022?
a. P100,000 c. P80,000
b. P70,000 d. P56,000
61. Richard Company and its divisions are engaged solely in manufacturing. The data
pertain to the industries in which operations were conducted for the year ended
December 31, 2022:
Operating Intersegment External
Segment Sales Revenues
A P1,000,000 P5,000,000
B 1,500,000 3,000,000
C 4,000,000 8,000,000
D 500,000 1,300,000
E 2,000,000 2,800,000
F 200,000 900,000
Total P9,200,000 P21,000,000

What is the minimum total external revenue of the reportable segments of Richard
Company?
a. P2,100,000 c. P15,750,000
b. P6,900,000 d. P22,650,000

62. Jonar Company operates a copper mine in Northern Mindanao. The entity paid P5,000,000
in 2022 for the mining site and spent an additional P3,000,000 to prepare the mine
for extraction of the copper. After the copper is extracted in approximately four
years, the entity is required to restore the land to its original condition after
which the land can be sold for P1,000,000. The cash outflow possibility for the
restoration cost is P2,000,000. The credit adjusted risk-free rate of interest is
10%. The present value of 1 at 10% for 4 periods is 0.6830. The entity expects to
extract 4,000,000 tons of copper from the mine. Actual production was 500,000 tons
in 2022 and 450,000 tons were sold in 2022. What amount of depletion should Jonar
include in cost of goods sold in 2022?
a. P1,005,250 c. P1,045,750
b. P1,105,175 d. P941,175

63. On April 1, 2022 Elizabeth Company entered into a cash-settled share-based


transaction with a supplier whereby it acquired wood with a fair value of P2,000,000
in return for a cash payment based on the market value of Elizabeth’s shares.
Payment will be made on August 31, 2022 and based on the fair value of 50,000
shares. The share price was P41.00 per share on April 1, 2022 and P48.00 at year-
end of July 31, 2021. What amounts are reported in Victory Company’s financial
statements at the year-end of July 31, 2022 in respect of the transaction?
a. an expense of P2,050,000 and liability of P2,050,000
b. an expense of P2,400,000 and liability of P2,400,000
c. an expense of P2,000,000 and equity balance of P2,000,000
d. an expense of P2,050,000 and equity balance of P2,050,000

64. In the year ended December 31, 2022, William Company, a drinks manufacturer, sold
a vat of maturing whisky to Harry Company for P4,600,000. Willian Company has signed
a contract agreeing to repurchase the whiskey in eight years' time at P5,400,000.
What amount of revenue from sale should William recognized on December 31, 2022?
a. P5,400,000 c. P900,000
b. P4,600,000 d. P0

Page 13 of 22 0915-2303213  resacpareview@gmail.com


FINANCIAL ACCOUNTING & REPORTING
ReSA Batch 44 – October 2022 CPALE Batch
25 September 2022  11:45 AM to 02:45 PM FAR Final Pre-Board
65. Megan Company issues P2,000,000 6% convertible Exam
bonds on June 1, 2022. Interest is
payable annually. The conversion option can be exercised in a number of years’ time.
The liability component is initially measured at P1,841,160 based on an 8% market
rate of interest for a similar nonconvertible bond. What amounts are recognized in
the statement of financial position at May 31, 2023?
a. a liability of P1,813,867 and equity of P186,133
b. a liability of P1,868,453 and equity of P158,840
c. a liability of P1,961,160 and equity of P38,840
d. b a liability of P1,988,453 and equity of P11,547

66. Sun and Moon Inc. factors P2,000,000 of its accounts receivables without recourse
for a finance charge of 5%. The finance company retains an amount equal to 10% of
the accounts receivable for possible adjustments. What would be recorded by Sun and
Moon as a gain (loss) on the transfer of receivables?
a. P100,000 gain c. P200,000 loss
b. P100,000 loss d. P300,000 loss

67. Mayka Company’s reporting period ends on June 30, 2022 and the financial statements
are authorized for issue on August 31, 2022. On July 30, 2022 a major drop in the
price of shares means that the value of the company’s investments has declined by
P1,300,000 since the period end. The fall in value is material. How should this
event be treated in the financial statements for the period ended June 30, 2022?
a. An adjusting event without separate disclosure
b. A non-adjusting event without separate disclosure
c. An adjusting event with disclosure that a major fall in the price of shares has
resulted in a loss of P1,300,000
d. A non-adjusting event with disclosure that a major fall in the price of shares
has resulted in a loss of P1,300,000

68. Everlasting Corporation provided the following information regarding its Research
JPB-04 included in the company’s Intangible account as of December 31, 2022:

Research JPB-04 is for a research project which consists of the following charges:
Salaries of research staff P18,000
Patent acquired solely for the use in the project 12,000
Special equipment acquired and useful for various
Similar research activities 10,000
Patent acquired for use in several research
Projects including JPB-04 16,000

The equipment and patents have been found to be useful for approximately four years.
Both the patents and equipment were acquired at the beginning of 2022. How much
should be recognized as research and development expense for the year 2022?
a. P56,000 c. P35,200
b. P36,500 d. P26,000

69. If the month-end bank statement shows a balance of P36,000, outstanding checks are
P12,000, a deposit of P4,000 was in transit at month end, and a check for P500 was
erroneously charged by the bank against the account, the correct balance in the
bank account at month end is?
a. P27,500 c. P20,500.
b. P28,500 d. P43,500

70. Rivera Company purchased a tooling machine on January 3, 2015 for P500,000. The
machine was being depreciated on the straight-line method over an estimated useful
life of 10 years, with no residual value. At the beginning of 2022, the company
paid P125,000 to overhaul the machine. As a result of this improvement, the company
estimated that the useful life of the machine would be extended an additional 5
years (15 years total). What should be the depreciation expense recorded for the
machine in 2022?
a. P34,375 c. P50,000
b. P41,667 d. P55,000

- END of EXAMINATION -

Page 14 of 22 0915-2303213  resacpareview@gmail.com


FINANCIAL ACCOUNTING & REPORTING
ReSA Batch 44 – October 2022 CPALE Batch
25 September 2022  11:45 AM to 02:45 PM FAR Final Pre-Board
Exam
ANSWERS & SOLUTIONS/CLARIFICATIONS
1 A 26 C 51 C
2 C 27 B 52 A
3 D 28 D 53 C
4 C 29 A 54 B
5 A 30 B 55 B
6 B 31 C 56 D
7 C 32 A 57 B
8 B 33 B 58 B
9 A 34 D 59 A
10 D 35 B 60 D
11 D 36 A 61 C
12 D 37 D 62 D
13 C 38 A 63 B
14 B 39 B 64 D
15 C 40 A 65 B
16 A 41 D 66 B
17 D 42 B 67 D
18 D 43 A 68 B
19 D 44 C 69 B
20 B 45 D 70 A
21 D 46 B
22 A 47 A
23 C 48 A
24 C 49 D
25 A 50 B

21 PCF (remaining bills and coins only) 2,550


Undeposited collections (120,000 - 15,000 - 20,000) 85,000
Cash in Bank - Checking (900,000 + 180,000 + 145,000) 1,225,000
Cash in Bank - JPB payroll fund 1,950,000
Cash in Bank - JPB dividend fund 750,000
Cash in Bank - Binaluyo Bank of Asia (16,000 x 56) 896,000
Cash in Bank - money market instrument, 60 days 250,000
Total cash and cash equivalents 5,158,550

22 Accounts receivable, beginning 2,700,000


Sales on accout (3,950,000 - 600,000) 3,350,000
Collections from credit customers (4,350,000)
Sales discounts granted (50,000)
Accounts written off (80,000)
Accounts receivable, ending 1,570,000

0 - 30 days (1,570,000 x 50% x2%) 15,700


31 - 60 days (1,570,000 x 30% x 5%) 23,550
over 60 days (1,570,000 x 20% x 15%) 47,100
Allowance for bad debts, end 86,350

Allowance for bad debts, end 86,350


Accounts written off 80,000
Allowance for bad debts, beginning (33,750)
Bad debt expense 132,600

Page 15 of 22 0915-2303213  resacpareview@gmail.com


FINANCIAL ACCOUNTING & REPORTING
ReSA Batch 44 – October 2022 CPALE Batch
25 September 2022  11:45 AM to 02:45 PM FAR Final Pre-Board
23 Unadjusted balance Exam
4,410,000
Goods held by Pyrus on consignment -
Goods shipped by Pyrus FOB Destination to a customer on December
31, 2022 and was received by the customer on January 3, 2023 400,000
Goods shipped by Pyrus FOB Shipping Point to a customer on
December 31, 2022 and was received by the customer on January 6,
2023 -
Goods shipped by a vendor FOB Destination on December 31, 2022 and
was received by Pyrus on January 10, 2023 -
Goods purchased FOB Shipping Point was shipped by the supplier on
December 31, 2022 and received by Pyrus on January 5, 2023 640,000
Adjusted inventory balance 5,450,000

24 First Shares (FVTPL) {65 - 54 x 100,000} - UGOL - PL 1,100,000


Second shares (FVTOCI) {(50 - 45) x 20,000} - UGOL - OCI 100,000
Total unrealized gain in comprehensive income 1,200,000

25 Initial cost (250,000 x 45) 11,250,000


Share of profit in associate:
Share in reported net income (10.2M x 40%) 4,080,000
Share in increase in deprn (2.5M / 8 x 40%) (125,000)
Share in increase in cost of sales (2M x 40%) (800,000) 3,155,000
Dividends (1,800,000)
CV of investment in associate as of 12/31/2022 12,605,000

26 FV of debt investment as of 12/31/2022:


PV of principal (5M x 1.10^-4) 3,415,067
PV of interest (5M x 8% x 3.169865) 1,267,946
FV of debt investment as of 12/31/2022 4,683,013

27 Impairment loss on 2019 500,000


Partial recovery (160,000 - 128,750) x 3 (93,750)
Limit on recovery 406,250

Recoverable amount 12/31/22 3,240,000


CV of asset 12/31/2022 (2,060,000 x 13/16) (1,673,750)
Increase in value 1,566,250
Gain on recovery (PL) (406,250)
Revaluation surplus 1,160,000

28 Weighted Average Accumulated Expenditures - 2021:


1/1/2021 3,000,000 x 12/12 = 3,000,000
6/30/2021 2,500,000 x 6/12 = 1,250,000
11/30/2021 2,000,000 x 1/12 = 166,667
Total WAAE in 2021 4,416,667
less: Total specific borrowings (2,000,000)
WAAE for general borrowings 2,416,667
Multiplied by WAR in 2021 9%
Potential interest to general borrowings (lower) 217,500
Actual interest on specific borrowing (2M x 8%) 160,000
Total capitalized interest in 2021 377,500
Total actual expenditures in 2021 (3M + 2.5M + 2M) 7,500,000
Total accumulated expenditures as of 12/31/2021 7,877,500

1/1/2022 7,877,500 x 12/12 = 7,877,500


Page 16 of 22 0915-2303213  resacpareview@gmail.com
FINANCIAL ACCOUNTING & REPORTING
ReSA Batch 44 – October 2022 CPALE Batch
25 September 2022  11:45 AM to 02:45 PM FAR Final Pre-Board
4/1/2022 1,500,000 x 9/12 = 1,125,000 Exam
10/1/2022 1,000,000 x 3/12 = 250,000
Total WAAE in 2022 9,252,500
less: Total specific borrowings (2,000,000)
WAAE for general borrowings 7,252,500
Multiplied by WAR in 2022 (735,000 / 6,875,000) 10.69%
Potential interest to general borrowings 775,358

Actual interest on general borrowings (lower) 735,000


Actual interest on specific borrowing (2M x 8%) 160,000
Total capitalized interest in 2022 895,000
Actual expenditures in 2022 (1.5M + 1M) 2,500,000
Total accumulated expenditures as of 12/31/2021 7,877,500
Cost of asset constructed 11,272,500

No interest expense in 2022 since the capitalized interest for BG is the


actual.

29 Under FV method (600,000 - 590,000) 10,000


Under Cost method (580,000 / 40) 14,500

30 Legal cost of filing for patent 520,000


Fees paid to government patent office 260,000
Drawings required by patent office to be filed with patent
application 145,000
Total cost of patent 925,000
X 9/10
CV as of 12/31/2022 832,500

Research salaries and fringe benefits for engineers and scientists 540,000
Cost of testing pre-production prototype 890,000
Total R&D expense 1,430,000

31 2021 2022

Total wrapper expected to redeem in 2021 (2.8M + 2M)


/ 10 480,000
Total wrapper expected to redeem in 2022 (4.2M - 2M + 2.7M)/10 490,000

Total FV of candy bars (5,600,000 x 5) / 6,750,000 x


5 28,000,000 33,750,000
Total FV of units (480,000 x 3)/ 490,000 x 3 1,440,000 1,470,000
Total FV 29,440,000 35,220,000
Beginning balance - 570,652
Unearned premium liability - added 1,369,565 1,408,646
Less: earned during the year
2021 - 1,369,565 x 280,000 / 480,000 (798,913)

2022 - 1,369,565 x 200,000 / 480,000 (570,652)

2022 - 1,408,646 x 220,000 / 490,000 (632,453)


Balance of unearned premium as of 12/31 570,652 776,193

32 200,000 x 20% = 40,000


370,000 x 30% = 111,000
550,000 x 50% = 275,000
Provision 426,000

Page 17 of 22 0915-2303213  resacpareview@gmail.com


FINANCIAL ACCOUNTING & REPORTING
ReSA Batch 44 – October 2022 CPALE Batch
25 September 2022  11:45 AM to 02:45 PM FAR Final Pre-Board
33 Total warranty expense (12,000 + 14,500) x 7,000 x 15% 27,825,000 Exam
Less: Total actual expenditures (6.5M + 12.5M) (19,000,000)
Warranty liability as of 12/31/2022 8,825,000

34 The bond is reported as current because there is a breach of agreement and the
grace period was obtained after the reporting period.

35 Date Principal NI (10%) EI (12%) Amort CV


1/1/21 3,839,558
12/31/21 1,000,000 400,000 460,747 60,747 2,900,305
12/31/22 1,000,000 300,000 348,037 48,037 1,948,342
12/31/23 1,000,000 200,000 233,801 33,801 982,143
12/31/24 1,000,000 100,000 117,857 17,857 (0)

36 PV of principal (1.5M x 1.05^-6) 1,119,323


PV of interest (1.5M x 4% x 5.075692) 304,542
Total 1,423,865

Retirement price of bonds (at 9%) 1,492,823


CV of bonds retired 1,485,715
Loss on retirement 7,109

37 Total debt outstanding (10M + 1M) 11,000,000


Less: PV of new cash flows
Principal (7M x 1.10^-3) 5,259,204
Interest (7M x 8% x 2.486852) 1,392,637 6,651,841
Difference 4,348,159
Divided by 11,000,000
% difference 39.53%
Note: Substantial modification

Total debt outstanding (10M + 1M) 11,000,000


Principal (7M x 1.12^-3) 4,982,462
Interest (7M x 8% x 2.401831) 1,345,026
Total PV of new liability 6,327,488
Gain on debt restructuring 4,672,512

38 Beginning SHE 1,650,000


Acquisition of TS (3,000 x 28) (84,000)
Acquisition of TS (3,000 x 35) (105,000)
Sale of TS (1,800 x 30) 54,000
Net income 450,000
Ending balance of SHE 1,965,000

39 Issuance of shares (5,000 x 18) 90,000


Issuance of shares 100,000
Total contributed capital 190,000

40 FV of property dividends declared (5,000 68) 340,000


less: gain on distribution (340,000 -
218,000) (122,000)
Net effect on RE (decrease) 218,000

Page 18 of 22 0915-2303213  resacpareview@gmail.com


FINANCIAL ACCOUNTING & REPORTING
ReSA Batch 44 – October 2022 CPALE Batch
25 September 2022  11:45 AM to 02:45 PM FAR Final Pre-Board
41 PV of periodic payment (150,000 x 6.710081) 1,006,512 Exam
PV of CPO (120,000 x 1.08^-10) 55,583
Total PV of lease liability 1,062,095
Add: IDC 350,000
Cost of ROUA 1,412,095
Less: RV at the end of EUL (80,000)
Depreciable cost 1,332,095
Divided by EUL 15
Depreciation expense 88,806

Interest expense (1,062,095 x 8%) 84,968

42 PV of periodic payment (2,220,000 x 5.230538) 11,611,794


PV of guaranteed residual value (400,000 x 1.11^-
7) 192,663
Total PV of lease liability 11,804,457
Add: IDC 500,000
Add: PV of dismantling cost (75,000 x 1.12^-7) 33,926
Initial cost of ROUA 12,338,384

CV of ROUA 12/31/2023 (12,338,384 - (1,705,483 x


2)) 8,927,417
Depreciation expense (12.338,384 - 400,000) / 7 1,705,483

43 FV of asset (PV of lease payments) 3,132,161


Less: Cost (2,500,000)
Gross profit 632,161

Lease receivable net 12/31/22 (3,132,161 - 380,000 x


1.06) 2,917,291

44 Tax depreciation in excess of book depreciation 200,000


Installment sales 400,000
Total FTA 600,000
Multiplied by future enacted tax rate x 30%
DTL 180,000
Provision for litigation (FDA) 140,000
Multiplied by future enacted tax rate x 30%
DTA 42,000

45 Taxable income (1,050,000 / 25%) 4,200,000


Add: Future taxable amount (100,000 / 25%) 400,000
Less: Penalties paid (240,000)
Pretax financial income 4,360,000
Less: TITE (given) (1,150,000)
Net income after tax 3,210,000

46 F 8,000,000
A (8M x 11%) - 45,000 835,000
C 2,880,000
B (2,650,000)
F 9,065,000

D 7,100,000
I (7.1M x 11%) 781,000
S 2,500,000
B (2,600,000)
D 7,781,000

Page 19 of 22 0915-2303213  resacpareview@gmail.com


FINANCIAL ACCOUNTING & REPORTING
ReSA Batch 44 – October 2022 CPALE Batch
25 September 2022  11:45 AM to 02:45 PM FAR Final Pre-Board
47 Current service cost 1,410,000 Exam
Past service cost 730,000

Net interest cost (8M-7.1M) x 11% (99,000)


Loss on early settlement (650,000 - 600,000) 50,000
Total benefit expense in PL 2,091,000
Actuarial loss on PA 45,000
Actuarial loss on DBO 360,000
Total benefit expense in OCI 405,000
Total benefit expense 2,496,000

Net defined benefit asset (9,065,000 - 7,781,000) 1,284,000

48 Total SHE 15,000,000


Less; Attributable to PS

Liquidation value (3M x 1.05) (3,150,000)

Dividends to PS (3M x 10% x 2) (600,000)


Attributable to ordinary 11,250,000
Divided by ordinary shares outs(6M+1M)/100 = 70,000 -
5,000 65,000
Book value per share of OS 173.08

49 Attributable to preference 3,750,000


Divided by preference shares outs 30,000
Book value per share of PS 125.00

50 1/1 200,000 12/12 200,000

4/1 20,000 9/12 15,000

9/1 (30,000) 4/12 (10,000)


WANOS 205,000

Net income 500,000


Preference dividends (40,000 x 3.50) (140,000)
Net income to ordinary 360,000
Divided by WANOS 205,000
BEPS 1.76

51 Net income 500,000


Interest expense (2M x 8% x 60%) 96,000
Adjusted net income 596,000
WANOS + POS 395,000
DEPS 1.51

52 PV of principal (2M x 1.07^-6) 1,332,684


PV of interest (2M x 1% x 4.766540) 95,331
Total PV of loan 1,428,015

Date NI (1%) EI (7%) Amort CV


1/2/2021 1,428,015

12/31/2021 20,000 99,961 79,961 1,507,977

12/31/2022 20,000 105,558 85,558 1,593,535

12/31/2023 20,000 111,547 91,547 1,685,082

Income from government grant - 2022 85,558


Page 20 of 22 0915-2303213  resacpareview@gmail.com
FINANCIAL ACCOUNTING & REPORTING
ReSA Batch 44 – October 2022 CPALE Batch
25 September 2022  11:45 AM to 02:45 PM FAR Final Pre-Board
53 Retained earnings, beginning 7,200,000
Exam
Share dividends (225,000 x 10% x 24) (540,000)
Cash dividends (225,000 x 1.1) x 2.5 (618,750)
Property dividends (720,000)
Net loss (810,000)
Retained earnings, ending 4,511,250

54 Total SHE, beginning 14,200,000


Share dividends -
Cash dividends (225,000 x 1.1) x 2.5 (618,750)
Property dividends (720,000)
Net loss (810,000)
Total SHE, end 12,051,250

55 Purchase patent (220,000)


Sold ei at FVTOCI 796,000
Sold equipment 110,000
Purchased ordinary shares (180,000)
Net cash 506,000

56 Issued bonds 392,000


Paid bank loan (120,000)
Sold treasury shares 95,000
Borrowed from bank 180,000
Net cash 547,000

57 Net income after tax 3,500,000


less: discontinued operations
(900,000 - (1,150,000 - 1,900,000) = 200,000 x
70% 140,000
Net income from continuing operations 3,360,000

58 CV if not classified as NCAHFS (5M x 7/10) 3,500,000 lower


RA as of 12/31/2022 (3.8M - 200,000) 3,600,000

59 Abandoned asset only not NCAHFS or disposal group

60 Profit from operations 100,000


Less; Impairment loss (2M - 1,980,000) (20,000)
Profit before tax 80,000
Less: Tax expense (80,000 x 30%) (24,000)
Profit after tax 56,000

61 75% of total external revenue (21M x 75%) 15,750,000

Depletable Cost (5M +3M + (2M x 0.6830)-


62 1M 8,366,000
Divided: Total estimated tons 4,000,000
Cost per ton 2.09
Multiplied by tons sold 450,000
Cost of sales 941,175

63. The transaction is initially measured at the fair value of the amount payable
50,000 shares x P41 = P2,050,000. It is subsequently remeasured based on the year-end
share price. As this is a cash-settled transaction a liability is recognized rather
than equity. The year-end liability balance is calculated at P2,400,000 (50,000 shares
x P48)

Page 21 of 22 0915-2303213  resacpareview@gmail.com


FINANCIAL ACCOUNTING & REPORTING
ReSA Batch 44 – October 2022 CPALE Batch
25 September 2022  11:45 AM to 02:45 PM FAR Final Pre-Board
64. The legal form of the transaction is a sale followed by a repurchase at aExam
later
date. It is however, clear from the scenario that the economic reality is that William
is using the maturing whisky as security for an eight-year loan with the bank.

65 1,841,160 x 1.08 - 120,000 1,868,453


Share premium (2M - 1,841,160) 158,840

66. 2,000,000 x 5% = 100,000 loss

67. D

68 Salaries 18,000
Patent solely for RD 12,000
Depreciation expense (10,000/4) 2,500
Amortization (16,000/4) 4,000
Total RD 36,500

69. P36,000 – P12,000 + P4,000 + P500 = P28,500.

70. 500,000 x 3/10 = 150,000 + 125,000 = 275,000 / 8 = 34,375

Page 22 of 22 0915-2303213  resacpareview@gmail.com


ReSA - THE REVIEW SCHOOL OF ACCOUNTANCY
CPA Review Batch 44  Oct 2022 CPALE  24 Sept 2022  8:00 - 11:00 AM

MANAGEMENT SERVICES FINAL PRE-BOARD EXAMINATION

INSTRUCTIONS: Select the correct answer for each of the questions. Mark only one
answer for each item by shading the box corresponding to the letter of your choice on
the answer sheet provided. STRICTLY NO ERASURES ALLOWED. Use pencil no. 2 only.
Set A
1. In typical financial markets, money market instruments are mostly:
B a. Long-term debt securities
b. Short-term debt securities
c. Long-term equity securities
d. Short-term equity securities

2. Naksu, Inc., a valued client, wants your advice on which of two alternatives it
should choose. One alternative is to sell an investment now for P 10,000. Another
alternative is to hold the investment for three days, after which it can sell it
for a certain selling price based on the following probabilities:
Selling Price Probability
P 5,000 .4
P 8,000 .2
P 12,000 .3
P 30,000 .1
Using probability theory that ignores aversion to risks, which of the following
is the most reasonable statement?
B a. Naksu shall hold the investment three days because of the chance of
getting P 30,000 for it.
b. Naksu shall hold the investment three days because the expected value
of holding exceeds the current selling price.
c. Naksu shall sell the investment now because the current selling price
exceeds the expected value of holding.
d. Naksu shall sell the investment now because there is a 60% chance
that the selling price will fall in three days.

3. The presence of inventory units usually causes the difference between:


B a. Debt ratio and equity ratio
b. Quick ratio and current ratio
c. Normal operating cycle and cash conversion cycle
d. Inventory conversion period and receivable collection period

4. If the first unit required 100 hours to complete, then how many hours are required
to produce the second unit given a 75% learning curve?
C a. 100 hours
b. 75 hours
c. 50 hours
d. 25 hours

5. Demand-pull inflation is induced by:


C a. Inward shift in the aggregate demand curve
b. Inward shift in the aggregate supply curve
c. Outward shift in the aggregate demand curve
d. Outward shift in the aggregate supply curve

6. A world-class manufacturer considers to introduce a new product that requires a


P 250,000 investment of capital. The necessary funds would be raised through a
bank loan at an interest rate of 8%. The fixed operating costs associated with
the product would be P 122,500, while the contribution margin percentage would
be 42%. Assuming a selling price of P 15 per unit, determine the number of units
(rounded to the nearest whole unit) the manufacturer would have to sell to
generate earnings before interest and taxes (EBIT) of 32% of the amount of
capital invested in the new product.
B a. 35,318 units
b. 32,143 units
c. 25,575 units
d. 23,276 units

7. Which of the following specific budgets is the key driving force of the overall
operational budget?
A a. The sales budget
b. The production budget
c. The capital projects budget
d. The pro forma income statement

Page 1 of 12 0915-2303213  resacpareview@gmail.com


MANAGEMENT SERVICES
ReSA Batch 44 – October 2022 CPALE Batch
24 September 2022  8:00 AM to 11:00 AM MS Final Pre-Board Exam
8. Business Unit A produces a widget product that can be sold either to external
customers or internally to Business Unit B. Business Unit B is currently
purchasing 15,000 widgets from an external supplier for P 25 per unit. Both
Business Unit A and Business Unit B are evaluated as profit centers. Production
and sales data for Business Unit A are provided below. Assuming that no costs
can be avoided if an internal transfer takes place, should Business Unit A
transfer widgets to Business Unit B? If so, what should the transfer price be?
Business Unit A
Capacity in units 90,000
Number of units being sold to external customers 75,000
Selling price per unit to external customers P 29
Variable costs per widget P 26
Fixed costs per unit (based on capacity) P 2

A a. No transfer should take place.


b. A transfer should take place based on a price of P 25.
c. A transfer should take place based on a price between P 26 and P 29.
d. A transfer should take place based on a price between P 25 and P 29.

9. In economics, scarcity refers to the situation of:


B a. Optimizing with the use of limited information
b. Having more wants than the amount of available resources
c. Rationing of available goods and services by the government
d. Sellers setting the prices of their products too high for people to
be able to afford them

10. 8 weeks of production data from a mobile phone manufacturer are presented below:
Quarter Phones Cost
1 2,331 P 3,245,874
2 2,657 P 3,474,318
3 1,987 P 2,883,675
4 2,412 P 3,287,621
5 2,583 P 3,354,966
6 2,497 P 3,428,752
7 2,285 P 3,152,347
8 2,645 P 3,271,899
The regression analysis results on these data are displayed below.
Coefficients Standard Error t Stat P-value Lower 95% Upper 95%
Intercept P 1,473,119 P 356,978 4.13 0.01 P 599,625 P 2,346,614
Phones P 738 P 147 5.03 0.00 P 379 P 1,097
Regression Statistics
Multiple R 0.90
R Square 0.81
Adjusted R Square 0.78
Standard Error P 87,127
Observations 8
What is the regression equation (total cost equation) for the above information?
D a. Total costs = P 147(Phones) + P 356,978
b. Total costs = P 1,473,119 (Phones) + 738
c. Total costs = P 356,978 (Phones) + P 147
d. Total costs = P 738(Phones) + P 1,473,119

11. Which of the following correctly represents the management-by-exception concept


in variance analysis?
C a. Only favorable variances provide a signal to management indicating
that something in the organization is out of compliance with budget
standards or performance expectations.
b. Only unfavorable variances provide a signal to management indicating
that something in the organization is out of compliance with budget
standards or budget performance.
c. Both favorable and unfavorable variances provide a signal to
management indicating that something in the organization is out of
compliance with budget standards or performance expectations.
d. Variances by themselves do not provide a signal to management that
something in the organization is out of compliance with budget
standards or performance expectations.

Page 2 of 12 0915-2303213  resacpareview@gmail.com


MANAGEMENT SERVICES
ReSA Batch 44 – October 2022 CPALE Batch
24 September 2022  8:00 AM to 11:00 AM MS Final Pre-Board Exam
12. When calculating the weighted-average cost of capital (WACC), an adjustment is
made for taxes because:
D a. Equity is risky
b. Preferred stock is used
c. Equity earns higher return than debt
d. The interest on debt is tax-deductible

13. Maidservant Kim, Inc. (MKI) provides contracted bookkeeping services. MKI has
annual fixed costs of P 100,000 and variable costs of P 6 per hour. This year,
the company budgeted 50,000 hours of bookkeeping services. MKI prices its
services based on full cost and uses a cost-plus pricing approach. The company
developed a billing price of P 9 per hour. The company’s markup level would be:
A a. 12.5%
b. 33.3%
c. 50.0%
d. 66.6%

14. The uncertainty in return on assets due to the nature of a firm’s operations is
known as:
A a. Business risk
b. Tax efficiency
c. Financial leverage
d. Financial flexibility

15. Songrim Company’s data on direct labor costs are given below:
Standard direct labor hours 30,000
Actual direct labor hours 29,000
Direct labor usage variance P 4,000 Favorable
Direct labor rate variance P 5,800 Favorable
Total payroll P 110,200
What was standard direct labor rate?
D a. P 3.60
b. P 3.80
c. P 3.67
d. P 4.00

16. Which of the following is best described as unsecured (without collateral),


short-term borrowing by very large, creditworthy firms?
B a. Eurodollars
b. Commercial papers
c. Banker’s acceptances
d. Repurchase agreements

17. Ice Stone Company applies overhead based on machine hours and reported the
following data:
Estimated annual overhead cost P 2,250,000
Actual annual overhead cost P 2,227,500
Estimated machine hours 300,000
Actual machine hours 295,000
The amount of overhead for the period is:
C a. P 22,500 under-applied
b. P 15,000 over-applied
c. P 15,000 under-applied
d. P 22,500 over-applied

18. When purchasing temporary investments, which of the following best describes the
risk associated with the ability to sell the investment in a short time without
significant price concessions?
A a. Liquidity risk
b. Investment risk
c. Interest rate risk
d. Purchasing power risk

19. If the consumer price index is 135 and was 122 a year ago, inflation rate was:
C a. 9.6%
b. -9.6%
c. 10.7%
d. -10.7%

Page 3 of 12 0915-2303213  resacpareview@gmail.com


MANAGEMENT SERVICES
ReSA Batch 44 – October 2022 CPALE Batch
24 September 2022  8:00 AM to 11:00 AM MS Final Pre-Board Exam
20. An investor owns stock and is concerned that prices may fall in the future. Which
strategy could help the investor hedge against adverse market conditions?
A a. Buy a put option
b. Buy a call option
c. Buy a futures contract
d. Buy an over-the-counter forward

21. Jin Mu Company is considering a change in technology that would reduce fixed
costs from P 800,000 to P 700,000. However, the ratio of variable costs to sales
will increase from 68% to 80%. What will happened to breakeven level of revenues?
A a. Increase by P 1,000,000
b. Decrease by P 1,812,500
c. Decrease by P 500,000
d. Decrease by P 301,471

22. Which ratio must be calculated using an amount external to a company’s financial
statements?
C a. Current ratio
b. Inventory turnover
c. Price-earnings ratio
d. Times interest earned

23. Shaman Choi Company predicts sales of 40,000 units in Q1 at P 5.00 per unit.
Shaman Choi predicts that unit sales will grow by 4% each quarter and that
selling price will increase by 5% each quarter. Each unit costs P 3.00 to produce
in Q1. Costs are expected to grow by 3% each quarter. Using a quarterly sales
budget, sales revenue for the year will be closest to:
A a. P 917,327
b. P 800,000
c. P 383,598
d. P 169,859

24. Most capital market transactions take place in


D a. Primary debt markets
b. Primary stock markets
c. Secondary debt markets
d. Secondary stock markets

25. Crown Prince, Inc. produced and sold 5,000 units during its most recent fiscal
year. Direct materials were P 9 per unit, direct labor costs were P 4 per unit,
and variable overhead costs were 110% of direct labor costs. Fixed overhead was
P 50,000, fixed selling and administrative expenses totaled P 50,000, and
variable selling and administrative expenses were a combined P 8 per unit.
Calculate inventory costs based on using the absorption (full) costing method.
C a. P 87,000
b. P 127,000
c. P 137,000
d. P 227,000

26. Which holds true if the market for PCs is in equilibrium at a price of P 40,000?
A a. The quantity of PCs produced will equal the quantity of PCs bought.
b. Sellers of PCs will have an incentive to charge a price higher than
P 40,000.
c. Buyers of PCs will want to buy fewer PCs than they are purchasing at
equilibrium.
d. If the cost of producing PCs falls below P 40,000 per unit, all
seller will stop supplying PCs.

27. Master Lee issues a P 10 million bond with a 6% coupon rate, 4-year maturity,
and annual interest payments when market interest rates are 7%.
➢ PV of P1 @ 7% for 4 years = 0.7629
➢ PV ordinary annuity @ 7% for 4 years = 3.38721
Using the discounted cash flow technique for bond valuation, what is the initial
carrying value of the bonds?
A a. P 9,661,326
b. P 9,400,000
c. P 10,000,000
d. P 10,338,721

Page 4 of 12 0915-2303213  resacpareview@gmail.com


MANAGEMENT SERVICES
ReSA Batch 44 – October 2022 CPALE Batch
24 September 2022  8:00 AM to 11:00 AM MS Final Pre-Board Exam
28. Soul Shifter, Inc. (SSI) is an international manufacturer of stylish shoulder
bags. SSI is split up into three regions: Europe Region, Asia Eastern Region,
U.S. Region. The Europe Region is split into three operating units: England
Operations, France Operations, Germany Operations. The France operating unit is
split up into the Sales Division and Manufacturing Division. The France sales
division is set up to focus exclusively on generating sales and revenue, without
responsibility for cost. Which of the following most accurately identifies the
responsibility center for the Sales Division, Manufacturing Division, and France
Operations of SSI?
B a. Sales Division: cost center; Manufacturing Division: revenue center;
France Operations: profit center.
b. Sales Division: revenue center; Manufacturing Division: cost center;
France Operations: profit center.
c. Sales Division: profit center; Manufacturing Division: cost center;
France Operations: cost center.
d. Sales Division: profit center; Manufacturing Division: revenue
center; France Operations: revenue center.

29. Early in 2022, Petrify Company switched to a JIT (Just-In-Time) inventory system.
Financial information for the two most recent years are listed here:
2021 2022
Net sales revenue P 2,000,000 P 1,800,000
Cost of goods sold 800,000 788,000
Beginning inventory 200,000 130,000
Ending inventory 130,000 30,000
How many times did inventory turnover increase as a result of the switch to the
JIT system?
C a. 2.1 times
b. 3.8 times
c. 5.0 times
d. 20.1 times

30. A bank is considering building a branch on a piece of property it already owns.


Which of the following cash flows should not be considered in the capital
budgeting analysis?
A a. Money spent when the property was originally purchased to determine
whether there are any environmental issues regarding the property
b. Cash the firm will forgo from the sale of the property if the company
decides to build
c. Cash flows resulting from several hundred new customers who will
switch from local competitors to the new branch if the bank makes the
investment
d. Shipping and installation charges the bank must spend to get equipment
in the new branch

31. Using the capital asset pricing model, what is the firm’s cost of equity (rounded
to the nearest whole percentage)?
• Risk-free rate of return: 0.05
• Market rate of return: 0.12
• Firm’s beta: 1.2
A a. 13%
b. 15%
c. 18%
d. 19%

32. Which of the following most accurately defines value chain analysis?
B a. An approach used by an organization to allocate capacity costs while
tracking and reporting the fixed costs of idle capacity in the
organization’s production process
b. An approach used by an organization to evaluate its internal
activities determine how essential each activity is strategically
creating and capturing value for the organization’s customers
c. A system based on the idea that inventory is not really an asset in
the production process, but is often a barrier to achieving the
organization’s main goals
d. A system that supports a very specific and extremely short-term
managerial view of an operation – the incremental value from a more
effective employment of a constrained resource

Page 5 of 12 0915-2303213  resacpareview@gmail.com


MANAGEMENT SERVICES
ReSA Batch 44 – October 2022 CPALE Batch
24 September 2022  8:00 AM to 11:00 AM MS Final Pre-Board Exam
33. Gone Wild Company incurs a cost of P 35 per unit, of which P 20 is variable, to
make a product that normally sells for P 58. A foreign wholesaler offers to buy
6,000 units at P 30 each. Gone Wild has the capacity to take on this order, but
will incur additional costs of P 4 per unit to imprint a logo and to pay for
shipping. Compute the increase (decrease) in net income that will be realized by
accepting the special order.
D a. P 60,000
b. (P 54,000)
c. (P 30,000)
d. P 36,000

34. When evaluating financial risk, which ratios will be most beneficial to the
analyst?
C a. Gross profit margin and the operating profit margin
b. Collection period and inventory turnover
c. Total debt and interest coverage ratios
d. Current and quick ratios

35. Jang-Uk (JU) Company is the leading textbook printer in the country. JU is
starting a new line of Managerial Accounting textbooks this year and would like
to calculate the budgeted total contribution margin for this line for 2023. JU
plans to sell every textbook they manufacture within the year. The following
information is available:
Sales Budget Q1 Q2 Q3 Q4 Total
Sales Volume 1,300 1,600 1,900 1,500 6,300
Budgeted Sales Price per Unit
Sales Price P 200.00
Standard Production Costs per Unit
Variable Production Costs P 47.00 per book
Fixed Production Costs 19.00 per book
Total Production Costs P 66.00 per book
Standard Selling & Admin Costs per Unit
Variable Selling & Admin Costs P 25.00 per book
Fixed Selling & Admin Costs 35.00 per book
Total Selling & Admin Costs P 60.00 per book
Using a contribution margin statement, calculate JU’s budgeted total contribution
margin for the new line of Managerial Accounting textbooks for 2023.
D a. P 74.00
b. P 128.00
c. P 466,200
d. P 806,400

36. Based on economic principles, the problem of “free-riding” refers to the


situation wherein some people (free-riders) do not contribute any but still
benefit from the actions that others undertake. This often happens when people
pursue their own private interests and do not contribute voluntarily to the
public interest. Which of the following is an example of free-riding?
B a. A housekeeper clearing a house
b. A tax evader enjoying national security
c. A consumer paying for pollution control
d. An individual who buys a ticket for a basketball match

37. The following information relates to the production department of the Seo Yul
Company for the first quarter:
Actual variable overhead P 68,500
Variable overhead application rate P 0.50 per hour
Total overhead application rate P 1.50 per hour
Variable spending variance P 8,000 unfavorable
Fixed volume variance P 5,000 favorable
What were the actual hours worked in this department during the quarter?
B a. 137,000
b. 121,000
c. 127,000
d. 119,000

Page 6 of 12 0915-2303213  resacpareview@gmail.com


MANAGEMENT SERVICES
ReSA Batch 44 – October 2022 CPALE Batch
24 September 2022  8:00 AM to 11:00 AM MS Final Pre-Board Exam
38. Financial managers who require an increase in return for a given increase in
risk are said to be:
C a. risk-indifferent
b. risk-seeking
c. risk-averse
d. risk-free

39. King’s Star Corporation manufactures two products: X and Y. The company has
4,000 hours of machine time available and can sell no more than 800 units of
Product X. Other pertinent data follow.
Product X Product Y
Selling price P 8.00 P 19.00
Variable cost 3.00 5.00
Fixed cost 3.50 6.25
Machine time per unit 2 hours 3 hours
Which of the following is the right objective function?
B a. Maximize Z = 8X + 19Y
b. Maximize Z = 5X + 14Y
c. Maximize Z = 1.50X + 7.75Y
d. Minimize Z = 6.50X + 11.25Y

40. A company is experiencing a sharp increase in sales activity and a steady increase
in production, so management has adopted an aggressive working capital policy.
Therefore, the company’s current level of net working capital.
B a. Would most likely be the same as in any other type of business condition
as business cycles tend to balance out over time.
b. Would most likely be lower than under other business conditions in order
that the company can maximize profits while minimizing working capital
investment.
c. Would most likely be higher than under other business conditions so that
there will be sufficient funds to replenish assets.
d. Would most likely be higher than under other business conditions as the
company’s profits are increasing.

41. Cho-Yeon Corporation is evaluating two mutually exclusive projects with the net
cash flows shown here:
Project X Project Z
0 (P 100,000) (P 100,000)
1 P 50,000 P 10,000
2 P 40,000 P 30,000
3 P 30,000 P 40,000
4 P 10,000 P 60,000
If cost of capital is 15%, which project should be chosen?
C a. Project X should be chosen since it has a better internal rate of
return (IRR).
b. Project X should be chosen since it has a better net present value
(NPV).
c. Neither project should be chosen.
d. Project Z should be chosen since it has the higher IRR.

42. All of the following describe key performance indicators (KPIs) in a balanced
scorecard system, except:
C a. A small set of critical data points
b. A good way to get a quick sense of an organization’s strategy
c. A term describing all the measures used in an organization in the
process of running the business
d. Measures that indicate to the executive team and other stakeholders
whether the organization is on track to accomplishing its strategic
objectives

43. At 40% capacity, overhead cost is computed to be P 1,450; at 75% capacity,


overhead cost is P 2,150. The fixed overhead cost at 80% capacity will be:
B a. P 20
b. P 650
c. P 1,600
d. P 2,250

Page 7 of 12 0915-2303213  resacpareview@gmail.com


MANAGEMENT SERVICES
ReSA Batch 44 – October 2022 CPALE Batch
24 September 2022  8:00 AM to 11:00 AM MS Final Pre-Board Exam
44. A company is expected to have unsold units on its first year of operations, then:
C a. Sales must be below breakeven point
b. Manufacturing variances are expected to be mostly unfavorable
c. Profit under absorption costing must be higher than the profit under
variable costing
d. Decision to accept or reject a special order on the unsold units
shall include an element of opportunity cost

45. Park Jin Co. has current assets of P 400,000 and current liabilities of P 500,000.
Current ratio will be increased by
A a. The purchase of P 100,000 of inventory on account
b. The payment of P 100,000 of accounts payable
c. The collection of P 100,000 of accounts receivable
d. Refinancing a P 100,000 long-term loan with short-term debt

46. After investing in a new project, Lee Company discovered that its residual income
remained unchanged. Which one of the following must be true about the new project?
B a. The net present value of the new project must have been positive.
b. The return on investment (ROI) of the new project must have been
equal to the firm’s cost of capital.
c. The net present value of the new project must have been negative.
d. The return on investment (ROI) of the new project must have been less
than the firm’s cost of capital.

47. What is the benefit for a firm with daily cash receipts of P 15,000 to be able
to speed up collections by 2 days, assuming an 8% annual return on short-term
investments and no cost to the company to speed up collections?
B a. P 2,400 daily benefit
b. P 2,400 annual benefit
c. P 15,000 annual benefit
d. P 30,000 annual benefit

48. Which of the following correctly describes theoretical capacity?


C a. Theoretical capacity is the level of output that the organization
actually achieves in a period.
b. Theoretical capacity is the amount of capacity that management
predicts the organization will produce in the period.
c. Theoretical capacity represents the level of output if all policy
constraints and scheduling limitations are removed. It also assumes
that no productivity is lost due to breakdowns, errors, etc.
d. Theoretical capacity represents the level of output that can be
realistically achieved based on current management policies, as well
as based on machine and labor scheduling expectations.

49. If a firm pays a constant dividend of P 10 and the required rate of return or
discount rate is equal to 10%, what is the price of the stock using the zero-
growth dividend model?
D a. P 1
b. P 10
c. P 50
d. P 100

50. Workers temporarily unemployed but who normally find jobs quickly are called:
C a. Cyclically unemployed
b. Seasonally unemployed
c. Frictionally unemployed
d. Structurally unemployed

51. Jang Gang Company planned to produce 3,000 units of its single product, S-
Ejector, during November. The standard specifications for one unit of S-Ejector
include 6 pounds of materials at P 0.30 per pound. Actual production in November
was 3,100 units of S-Ejector. The accountant computed a favorable direct
materials purchase price variance of P 120. Based on these variances, one could
conclude that:
C a. More materials were purchased than were used
b. More materials were used than were purchased
c. The actual cost of materials was less than the standard cost
d. The actual usage of materials was less than the standard allowed

Page 8 of 12 0915-2303213  resacpareview@gmail.com


MANAGEMENT SERVICES
ReSA Batch 44 – October 2022 CPALE Batch
24 September 2022  8:00 AM to 11:00 AM MS Final Pre-Board Exam

52. Which of the following is likely to encourage a firm to increase the amount of
debt in its capital structure?
C a. The firm’s earnings become more volatile
b. The firm’s assets become less liquid
c. The corporate tax rate increases
d. The personal tax rate increases

53. Park Dang Gu (PDG) Company plans to discontinue a department that currently
provides a P 24,000 contribution margin and has allocated overhead of P 48,000,
of which P 21,000 cannot be eliminated. PDG’s average income tax rate is 30%.
The effect of this discontinuance on PDG’s after-tax profit would be a(n):
A a. Increase of P 2,100
b. Increase of P 3,000
c. Decrease of P 24,000
d. Decrease of P 16,800

54. If a company’s cash conversion cycle increases, then the company:


C a. Becomes more profitable
b. Incurs more shortage or stockout costs
c. Increases its investment in working capital
d. Reduces its payable deferral period (age of payable)

55. Jin Ho-Kyung’s Wholesale Flowers produces two types of flower bouquets that it
distributes to retail shops: simple bouquets and upgraded bouquets. Total fixed
costs for the firm are P 184,000. Variable costs and sales data for these bouquets
are presented here.
Simple Bouquets Upgraded Bouquets
Selling price per unit P 24.00 P 40.00
Variable cost per unit P 20.00 P 32.00
Budgeted sales (units) 20,000 30,000

If the product sales mix were to change to three upgraded bouquets sold for each
simple bouquet sold, the breakeven volume for each of these products would be:
D a. 11,500 simple bouquets and 17,250 upgraded bouquets
b. 15,333 simple bouquets and 15,333 upgraded bouquets
c. 27,600 simple bouquets and 9,200 upgraded bouquets
d. 6,571 simple bouquets and 19,714 upgraded bouquets

56. If a capital project has a hurdle rate higher than its internal rate, then its
profitability index is:
D a. Higher than the net present value
b. Higher than the cost of capital
c. Less than zero
d. Less than one

57. If the investment turnover decreased by 20% and profit margin decreased by 30%,
then return on investment would:
C a. Increase by 30%
b. Decrease by 20%
c. Decrease by 44%
d. Decrease by 50%

58. Period costs under variable costing include:


B a. Variable manufacturing and non-manufacturing costs
b. Fixed manufacturing and non-manufacturing costs
c. Variable and fixed manufacturing costs
d. Indirect materials and labor

59. The first order of 500 units incurred P 120,000 of labor costs; the next order
of 500 units required an additional P 72,000 of labor costs. What percentage of
learning occurred?
A a. 80%
b. 85%
c. 90%
d. 95%

Page 9 of 12 0915-2303213  resacpareview@gmail.com


MANAGEMENT SERVICES
ReSA Batch 44 – October 2022 CPALE Batch
24 September 2022  8:00 AM to 11:00 AM MS Final Pre-Board Exam
60. A well-known manufacturer paid more property tax on their manufacturing facility
this year than they were expecting. This would most likely cause an:
B a. Unfavorable fixed overhead volume variance
b. Unfavorable fixed overhead spending variance
c. Unfavorable variable overhead spending variance
d. Unfavorable variable overhead efficiency variance

61. Short-term objectives, tactics for achieving these objectives, and operational
planning (master budget) must be congruent with what?
C a. The organization’s external environmental factors
b. The organization’s internal environmental factors
c. The organization’s strategic plan and long-term strategic goals
d. The organization’s performance evaluation and incentive compensation
factors

62. So-yi Company is considering a change in collection procedures that would result
in an increase of the average collection period from 28 to 36 days. So-yi
anticipates that next year’s sales to be P 9 million and that 80% of the sales
will be on credit. So-yi estimates short-term interest rates at 6% and uses a
360-day year for decision making. What minimum savings in collection costs would
the procedure change have to generate to offset the increased investment in
accounts receivable?
B a. P 1,200
b. P 9,600
c. P 12,000
d. P 33,600

63. The cost-plus target pricing approach is generally in what formula?


B a. Unit cost ÷ Selling price = Markup percentage
b. Unit cost x (1 + Markup % on unit cost) = Targeted selling price
c. Variable cost + Fixed cost + Contribution margin = Targeted selling
price
d. Cost base + Gross margin = Targeted selling price

64. In which scenario would a horizontal analysis be the best choice?


B a. A bank wishes to compare progress among different companies.
b. A company wishes to market its growth to potential stockholders.
c. A vendor wishes to evaluate financial statement data in a given year.
d. An investor wishes to evaluate financial statement data by expressing
each item in a financial statement as a percentage of a base amount.

Items 65 and 66 are based on the following information


Bu-yeon Company provides the following information based on its accounting
records:
Current assets P 2,000,000
Noncurrent assets 7,000,000
Current liabilities 1,000,000
Noncurrent liabilities 4,000,000
Pretax operating profit 1,500,000
Pretax cost of equity 15%
Pretax cost of debt 5%
Tax rate 40%

The carrying amounts and market values of above amounts do not differ
significantly.

65. What is Bu-yeon Company’s weighted average cost of capital (WACC)?


B a. 6%
b. 9%
c. 10%
d. Cannot be determined from the given information

66. What is Bu-yeon Company’s economic value added (EVA)?


A a. P 180,000
b. P 120,000
c. P 90,000
d. P 0

Page 10 of 12 0915-2303213  resacpareview@gmail.com


MANAGEMENT SERVICES
ReSA Batch 44 – October 2022 CPALE Batch
24 September 2022  8:00 AM to 11:00 AM MS Final Pre-Board Exam

Items 67 and 68 are based on the following information


Go Won buys toy guns at P 25 per dozen from its wholesaler. Go Won will sell
35,000 dozens of toy guns evenly throughout the year and it desires a 12% return
on investment (cost of capital) on its inventory investment. In addition, rent,
insurance and related taxes for each dozen toy guns in inventory amounts to P
0.50. The cost per order is P 8. Go Won uses a 350-day year.

67. What is the average number of toy guns does Go Won maintain?
C a. 200 toy guns
b. 400 toy guns
c. 2,400 toy guns
d. 4,800 toy guns

68. How often shall Go Won place the orders within a year?
A a. Every 4 days
b. Every 5 days
c. Every 6 days
d. Every week

Items 69 to 70 are based on the following information


Alchemy of Souls (AOS), Inc. is considering the purchase of a new machine that
costs P 150,000 and saves P 85,000 per year in operating expenses. The machine
has an estimated useful life of 3 years. Assume that 30% of the depreciable base
will be depreciated in the first year, 40% in the second year, and 30% in the
third year. At the end of its useful life, the new machine will have a P 10,000
resale value that is not to be considered in computing the depreciation. AOS
uses a 40% estimated income tax rate and a 16% hurdle rate to evaluate capital
investment projects.

Discount rates for a 16% rate are as follows:


Present Value of an
Present Value of P 1 Ordinary Annuity of P 1
Year 1 0.862 0.862
Year 2 0.743 1.605
Year 3 0.641 2.246

69. What is the net present value of this project?


B a. P 9,432
b. P 13,278
c. P 15,842
d. P 40,910

70. The payback period for this investment would be:


B a. 2.94 years
b. 2.09 years
c. 1.76 years
d. 1.14 years

- END of EXAMINATION -

NOTE: Summary of answers as well as


solutions and clarifications to selected items are found on page 12.

Page 11 of 12 0915-2303213  resacpareview@gmail.com


MANAGEMENT SERVICES
ReSA Batch 44 – October 2022 CPALE Batch
24 September 2022  8:00 AM to 11:00 AM MS Final Pre-Board Exam

Page 12 of 12 0915-2303213  resacpareview@gmail.com


ReSA - THE REVIEW SCHOOL OF ACCOUNTANCY
CPA Review Batch 44  Oct 2022 CPALE  25 September 2022  8:00 - 11:00 AM

TAXATION FINAL PRE-BOARD EXAMINATION

INSTRUCTIONS: Select the correct answer for each of the questions. Mark only one
answer for each item by shading the box corresponding to the letter of your choice on
the answer sheet provided. STRICTLY NO ERASURES ALLOWED. Use pencil no. 2 only.

1. The gross estate of a decedent is valued at P3,000,000 composed of registered and


registrable properties. Total deductions are P4,000,000. Which of the following
statements is/are correct?
I - The estate of the decedent is required to submit notice of death.
II - The estate of the decedent is required to file an estate tax return.
III - The estate of the decedent is required to submit a statement certified
by a CPA.
a. Statements I, II and III
b. Statements II and III
c. Statement II
d. None of the statements
2. Gross gifts of non-resident alien donor shall be composed of:
a. real properties, personal properties and taxable transfers wherever
situated.
b. real properties and personal properties wherever situated.
c. real properties, personal properties and taxable transfers situated
in the Philippines.
d. real properties and personal properties situated in the Philippines
except intangible personal properties when there is reciprocity.

3. A VAT-registered taxpayer service provider has the following data taken from the
books of account for the month of January 2021:
Accounts receivable, January 1, 2021 P 560,000
Sales on account for the month of January 1,120,000
Services paid in cash for the month of January 336,000
Accounts receivable, January 31, 2021 784,000
Cash purchases for the month of January 2021 448,000

How much is the output tax for the month of January, 2021 using 12% rate?
a. P201,600 c. P132,000
b. P147,840 d. P96,000

4. Who of the following individual taxpayer shall not be required to file an income
tax return?
a. Individuals deriving compensation from two or more employers concurrently or
successively at any time during the taxable year;
b. Individual taxpayer receiving purely compensation income, regardless of
amount, from only one employer in the Philippines for the calendar year, the
income tax of which has been withheld correctly by the said employer.
c. Employees deriving compensation income, regardless of the amount, whether from
a single or several employers during the calendar year, the income tax of
which has not been withheld correctly (i.e., tax due is not equal to the tax
withheld) resulting to collectible or refundable return.
d. Individuals receiving purely compensation income from a single employer,
although the income tax of which has been correctly withheld, but whose
spouse is required to file income tax return

5. The books of account may be kept in:


a. Native language. c. Chinese language.
b. Korean language. d. Japanese language.
6. The filing of the withholding tax returns (BIR Form No. 1601EQ) for creditable
withholding tax and Form Nos. 1602 for final tax on interest on bank deposits, 1603Q
for final tax withheld on fringe benefits, and 1601FQ for all other final withholding
taxes and payment of the taxes withheld at source shall be made:
a. not later than the last day of the month following the close of the quarter
during which the withholding was made.
b. not later than the 20th day of the month following the close of the quarter
during which the withholding was made.
c. not later than the 25th day of the month following the close of the quarter
during which the withholding was made.
d. not later than the first day of the month following the close of the quarter
during which the withholding was made.
Page 1 of 14 0915-2303213  resacpareview@gmail.com
TAXATION
ReSA Batch 44 – October 2022 CPALE Batch
25 September 2022  8:00 AM to 11:00 AM TAX Final Pre-Board Exam
7. Which of the following is not an effect of failure to affix the required DST?
a. An instrument, document or paper which is required by law to be stamped and
which has been signed, issued, accepted or transferred without being duly
stamped, shall not be recorded.
b. An instrument, document or paper or any copy thereof or any record of transfer
of the same which is required by law to be stamped shall not be admitted or
used in evidence in any court until the requisite stamp or stamps are affixed
thereto and cancelled.
c. No notary public or other officer authorized to administer oaths shall add
his jurat or acknowledgment to any document subject to documentary stamp tax
unless the proper documentary stamps are affixed thereto and cancelled.
d. An instrument, document or paper which is required by law to be stamped and
which has been signed, issued, accepted or transferred without being duly
stamped, shall make the transaction void.

8. The Commissioner may compromise the payment of any internal revenue tax, when:
I - there is a reasonable doubt as to the validity of the claim against the
taxpayer
II - the tax or any portion thereof appears to be unjustly or excessively
assessed.
a. Both I and II are correct c. Only I is correct
b. Both I and II are incorrect d. Only II is correct

9. Who has the power to lift the order of distraint?


a. Secretary of Finance
b. Commissioner of Internal Revenue exclusively
c. Commissioner of Internal Revenue or his duly authorized representative
d. Revenue District Officer

10. A taxpayer paid excessive tax on April 15, 2000. On December 20, 2001, she filed a
written claim for refund. Her claim was denied by the BIR and she received the
denial on March 16, 2002. She filed a motion for reconsideration with the BIR on
March 31, 2002. On April 18, 2002, she received the final denial of the BIR. What
will be the taxpayer’s remedy?
a. File another motion for reconsideration with the BIR within 30 days after the
receipt of the final denial
b. File an appeal with the Court of Tax Appeals within 30 days after the receipt
of the final denial
c. File an appeal with the Court of Tax Appeals within 15 days after the receipt
of the final denial
d. The taxpayer has no more remedy against the final denial

11. The following are common to the inherent power of taxation, power of eminent domain
and police power, except for which of the following?
a. They are necessary attributes of sovereignty.
b. They interfere with private rights and property.
c. They affect all persons or the public.
d. They are legislative in implementation.

12. Which statement is wrong? A revenue bill:


a. must originate from the House of Representative and on the same bill the
Senate may propose amendments.
b. may originate from the Senate and on which same bill the House of
Representative may propose amendments.
c. may have a House version and a Senate version approved separately.
d. may be recommended by the President to Congress.

13. One of the following is not a characteristic or an element of tax:


a. It is levied by the legislature.
b. It is payable in money or in kind.
c. It is proportionate in character.
d. It is an enforced contribution.

14. The power to interpret the provision of the Tax Code and other tax laws shall be
under the exclusive and original jurisdiction of the Commissioner of Internal
Revenue subject to review by:
a. Secretary of Finance. c. Court of Appeals.
b. Court of Tax Appeals. d. Municipal Courts.

Page 2 of 14 0915-2303213  resacpareview@gmail.com


TAXATION
ReSA Batch 44 – October 2022 CPALE Batch
25 September 2022  8:00 AM to 11:00 AM TAX Final Pre-Board Exam
15. Unless otherwise provided in Local Government Code, all local taxes, fees, and
charges shall be paid:
a. within the first twenty (20) days of January or of each subsequent quarter,
as the case may be.
b. within the first fifteen (15) days of January or of each subsequent quarter,
as the case may be.
c. within the first ten (10) days of January or of each subsequent quarter, as
the case may be.
d. within the first five (5) days of January or of each subsequent quarter, as
the case may be.

16. A senior citizen is engaged in the sale of VATable goods. His gross annual sales,
however, do not exceed the VAT threshold and he is not VAT-registered. During the
first quarter of the current year, his gross sales amounted to P250,000. How much
is the business tax due, if any?
a. P7,500
b. P5,000
c. P2,500
d. None, senior citizens are exempt from business taxes

17. An individual is operating a trading business enterprise. He is asking you to assist


him to register his business under the Barangay Micro Business Enterprise (BMBE) so
he can avail of the incentives granted under it. His total assets including the cash
of P300,000 he loaned from a bank amounted to P3,500,000. The total assets also
include P500,000 value of the land on which the particular business entity's office,
plant and equipment are situated. Is the individual qualified?
a. No. Because he is engaged in a trading business which is specifically excluded
from those who can register under BMBE.
b. Yes. Because his total assets excluding the land on which the particular
business entity's office, plant and equipment are situated do not exceed
P3,000,000.
c. No. Because only juridical persons are qualified to register under BMBE.
d. Yes. Because trading business is automatically qualified regardless of the
total assets.

18. This model generally favors retention of greater so called “source country” taxing
rights under a tax treaty—the taxation rights of the host country of investment—as
compared to those of the “residence country” of the investor.
a. OECD Model Tax Convention
b. UN Model Double Taxation Convention
c. United States Model Income Tax Convention
d. None of the choices

19. If the goods or services are used in both the registered project or activity and
administration purposes and the proper allocation could not be determined, the
purchase of such goods and services shall be:
a. subject to 12% VAT. c. exempt from VAT.
b. subject to 0% VAT. d. disregarded.

20. First statement: Registered Business Enterprises (RBEs) which are categorized as
Domestic Market Enterprises (DME) are not entitled to VAT zero-rating on local
purchases.
Second statement: Sale of goods or services to a registered domestic market
enterprise shall be exempt VAT.
a. Both statements are correct c. Only the first statement is correct
b. Both statements are incorrect d. Only the second statement is correct

SITUATIONAL
X Company has been assessed deficiency income tax of P1,000,000, exclusive of interest
and surcharge, for taxable year 2015. The tax liability has remained unpaid despite
the lapse of June 30, 2017, the deadline for payment stated in the notice and demand
issued by the Commissioner. Payment was made by the taxpayer only on February 10, 2018.
How much is the total amount due on February 10, 2018?

21. How much is the interest on deficiency from April 16, 2016 to June 30, 2017 (441
days)?
a. P302,054.79 c. P181,232.87
b. P241,643.83 d. P144,986.30
Page 3 of 14 0915-2303213  resacpareview@gmail.com
TAXATION
ReSA Batch 44 – October 2022 CPALE Batch
25 September 2022  8:00 AM to 11:00 AM TAX Final Pre-Board Exam
22. How much is the additional interest on deficiency from July 1, 2017 to December
31, 2017 (184 days)?
a. P126,027.39 c. P75,616.43
b. P100,821.91 d. P60,493.50

23. How much is the interest on delinquency from July 1, 2017 to December 31, 2017
(184 days)?
a. P151,232.87 c. P90,234.23
b. P150,390.39 d. P75,616.43

24. How much is the interest on delinquency from January 1, 2018 to February 10, 2018
(41 days)?
a. P33,698.63 c. P22,465.75
b. P33,510.90 d. P20,106.54

25. How much is the total amount due on February 10,2018?


a. P2,012,962.67 c. P1,512,962.67
b. P1,762,962,67 d. P1,262,962.67

SITUATIONAL

On July 1, 2021, Ms. Cris Cruz sold shares of stock for P200,000. The shares, acquired
on June 1, 2018 at a par value of P150,000, were held as investment, and were sold
directly to a buyer under the following terms:
Down payment, July 1, 2021 P 30,000
Installment due, October 10, 2021 30,000
Installment due, October 10, 2022 75,000
Installment due, October 10, 2023 75,000

26. How much was the capital tax due in July 1, 2021?
a. P7,500 c. P1,125
b. P1,875 d. None

27. When should the capital gains tax return be filed?


a. August 1, 2021 c. July 16, 2021
b. July 31, 2021 d. July 30, 2021

28. How much was the documentary stamp tax due?


a. P1,500 c. P563
b. P1,125 d. None

SITUATIONAL
The following were taken from the Statement of Income and Expenses of ABC Corporation
for the taxable year 2021:
Gross profits from sales P800,000
Less: Business expenses 440,000
Provision for bad debts 80,000
Write-off of inventories lost due to spoilage 20,000 540,000
Net income from operation 260,000
Add: Other income
Dividend income from a domestic corporation 50,000
Interest on Philippine bank peso deposit 30,000 80,000
Net income P340,000

Additional information:
1) Accounts written off during the year and charged to allowance for bad debts,
P50,000;
2) Recoveries on accounts receivable previously written off in 2020 and
credited to allowance for bad debts:
Allowed as deduction by BIR, P30,000;
Disallowed as deduction by BIR, P20,000.
29. How much is the total non-deductible expenses/taxable other income?
a. P410,000 c. P110,000
b. P130,000 d. P100,000

30. How much is the non-taxable income and income subject to final tax?
a. P130,000 c. P80,000
b. P100,000 d. P30,000

Page 4 of 14 0915-2303213  resacpareview@gmail.com


TAXATION
ReSA Batch 44 – October 2022 CPALE Batch
25 September 2022  8:00 AM to 11:00 AM TAX Final Pre-Board Exam
31. How much was the taxable net income using Reconciliation Net Income Per Books
Against Taxable Income?
a. P340,000 c. P390,000
b. P370,000 d. P440,000
32. LAB Corporation, a domestic manufacturing corporation, had a gross sales of
P100,000,000.00 for fiscal year ending June 30, 2021. It incurred cost of sales
of P60,000,000.00 which includes Direct Labor Wage of P20,000,000.00 and operating
expenses of P17,500,000.00 which include training expenses amounting to
P3,000,000.00. The corporation complied with all the prescribed requirements (e.g.
Apprenticeship Agreement, Certification from DepED or TESDA or CHED, whichever is
applicable). How much is the corporation’s net taxable income for the taxable year?
a. P22,500,000 c. P20,500,000
b. P21,000,000 d. None of the choices
33. CPL Corporation secured in 2018 a bank loan for its business expansion, and incurred
interest expense of P2,000,000.00 in calendar year 2020 on the said bank loan. In
the same year, it likewise earned interest income of P300,000.00 subjected to final
tax of 20% and royalties of P100,000 subjected to 20% final tax. For calendar year
2020, its gross income amounted to P20,000,000.00. Its gross assets, including the
P20,000,000 value of the land where its building and plant are situated, is
P120,000,000.00. Its operating expenses amounted to P15,000,000.00, inclusive of
the interest expense of P2,000,000.00. How much is the allowable interest expense?
a. P1,894,000 c. P1,920,500
b. P1,934,000 d. P1,950,500

34. GCC Corporation, a domestic corporation, owns 20% of the outstanding shares of BTS
Corporation, a non-resident foreign corporation (NRFC), since August 1, 2015. On
May 1, 2021, it received dividends amounting to P1,000,000 from the said NRFC. On
September 1, 2022, GCC Corporation utilized P800,000 for its dividend payments. On
January 1, 2023, it utilized the remaining P200,000 for its working capital.
How much is tax-exempt dividend, if any?
a. P1,000,000 c. P200,000
b. P 800,000 d. None of the choices

35. In 2021, Taprolani Corporation purchased a residential house and lot for P2,300,000.
The property was sold to the President of the corporation for P2,500,000. The fair
market value per BIR and per Assessor's Office were P3,500,000 and P3,000,000
respectively.
How much was fringe benefits tax, if any?
a. P1,884,615 c. P538,462
b. P1,615,385 d. None

36. KLM Corporation, a manufacturer, has gross sales of P190,000,000 for CY 2021, its
fourth year of operation. Its total assets amounted to P150,000,000, net of the
value of the land of P6,000,0000 where its manufacturing plant and business
operations are situated. Its cost of sales and allowable operating expenses amounted
to P100,000,000 and P89,000,000, respectively.
How much is the income tax due?
a. P1,800,000 c. P200,000
b. P 250,000 d. None of the choices

37. Using the same data in preceding number and it is the corporation’s fifth year of
operation. How much is the income tax due?
a. P1,800,000 c. P250,000
b. P 900,000 d. P200,000

38. Mr. Julian Cruz procured a life insurance upon his own life. He designated his
estate’s executor as an irrevocable beneficiary. For estate tax purposes, the
proceeds of life insurance are:
a. included in the gross estate of Mr. Julian Cruz because when the executor of
the estate is a beneficiary the proceeds are included in the gross estate
regardless of the designation.
b. not included in the gross estate of Mr. Julian Cruz because the designation of
the beneficiary is irrevocable.
c. included in the gross estate of Mr. Julian Cruz because proceeds of life
insurance are always subject to estate tax.
d. not included in the gross estate because, as a rule, proceeds of life insurance
are generally not subject to estate tax.
Page 5 of 14 0915-2303213  resacpareview@gmail.com
TAXATION
ReSA Batch 44 – October 2022 CPALE Batch
25 September 2022  8:00 AM to 11:00 AM TAX Final Pre-Board Exam
39. Mr. Rigoberto Collado, a citizen and resident of Puerto Rico, dies during the year.
Puerto Rico does not impose transfer taxes on properties of decedent not residing
therein. He left the following properties among others:
Shares of stock, San Miguel Corporation, Manila
House and lot, Puerto Rico
Leasehold on a condominium unit, Philippines
Contract for public works, Philippines

The executor of his estate in Philippines asked you what properties are to be
included in his Philippine gross estate. What answer will you give him?
a. Include all the properties.
b. Include contract for public works only
c. Include all properties except shares of stock and house and lot
d. Include all properties except house and lot in Puerto Rico.

40. How much discount is granted to senior citizens on his purchase of basic and prime
commodities, subject to limitations?
a. 20% c. 10%
b. 15% d. 5%

SITUATIONAL

Mr. Pat Tayna, a single and a non-resident alien, died of a heart attack in 2018,
leaving the following properties in favor of his heirs:
Gross estate within the Philippines
(including P8,000,000 family home) P 30,000,000
Gross estate outside of the Philippines P 20,000,000
Funeral expense P 500,000
Judicial and administrative expenses P 2,000,000
Claims against the estate P 5,000,000
Unpaid taxes accrued before death P 300,000
Losses due to robbery P 200,000
Medical expenses P 500,000

41. How much is the taxable gross estate of the Estate of Pat Tayna?
a. P50,000,000 c. P35,000,000
b. P45,000,000 d. P30,000,000

42. How much is the deductible ordinary deductions of the Estate of Pat Tayna?
a. P8,000,000 c. P4,800,000
b. P5,500,000 d. P3,300,000

43. How much is the deductible special deductions of the Estate of Pat Tayna?
a. P8,500,000 c. P500,000
b. P5,000,000 d. None

44. How much is the estate tax due of the Estate of Pat Tayna?
a. P2,070,000 c. P1,782,000
b. P2,010,000 d. P1,572,000

SITUATIONAL

The following data are taken from the books of a dealer in securities:
Selling price, shares held as inventory P3,000,000
Selling price, shares held as investment 2,000,000
Acquisition cost, shares held as inventory 1,000,000
Acquisition cost, shares held as investment 1,500,000
Other income, shares held as inventory 200,000

45. How much is the output tax using 12% rate?


a. P360,000 c. P240,000
b. P264,000 d. P220,000

46. How much is the capital gains tax assuming that the shares held as investment are
sold not through the local stock exchange?
a. P300,000 c. P45,000
b. P 75,000 d. Not subject capital gains tax.

Page 6 of 14 0915-2303213  resacpareview@gmail.com


TAXATION
ReSA Batch 44 – October 2022 CPALE Batch
25 September 2022  8:00 AM to 11:00 AM TAX Final Pre-Board Exam
47. How much is the percentage tax on the shares held as investment assuming that the
shares are listed and traded in the local stock exchange?
a. P12,000 c. P3,060
b. P10,000 d. Not subject percentage tax.

48. How much is the percentage tax on the shares held as investment and are from a
closely held corporation and were sold in the secondary offering where the ratio
of the shares sold over the outstanding shares is 25%?
a. P80,000 c. P20,000
b. P40,000 d. Not subject to percentage tax

49. Which of the following has an option to register under the VAT system?
a. Common carriers by land transporting passengers the gross receipts of which
exceed P3,000,000 during the year
b. VAT-registered seller with mixed transactions as far as his VAT-exempt sales
are concerned
c. Operator of cockpit the receipts of which do not exceed P3,000,000 during the
year
d. Seller of VAT-subject services the gross receipts of which exceed P3,000,000
during the year

50. A keeper of garage whose gross receipts for the year exceed P3,000,000 is subject
to:
a. value-added tax. c. garage sales tax.
b. common carrier’s tax. d. franchise tax.

51. As a general rule, proceeds of insurance are not taxable because they only
constitute a return of what was lost (return of capital). Which of the following
are taxable proceeds?
a. Proceeds of life insurance c. Proceeds of property insurance
b. Proceeds of accident or health insurance d. Proceeds of crop insurance.
52. Which of the following statements in INCORRECT?
a. No uniform method of accounting can be prescribed for all taxpayers.
b. Each taxpayer is required by law to make a return on his true income.
c. The taxpayer has to adopt accrual method of accounting because it is in
accordance with generally accepted accounting principles.
d. Where purchase or sale of merchandise is an income-producing factor;
inventories on hand shall be taken at the beginning and at the end of year.

53. Papable Aljon Lee sold a painting (capital asset) which he purchased in 2018 at a
cost of P30,000. He sold the painting to Mamable Tonee Dee, on the following terms:
June 1, 2021 down payment P10,000
August 1, 2021 installment due 10,000
October 1, 2021 installment due 20,000
October 1, 2022 installment due 40,000
October 1, 2023 installment due 40,000

For the year 2021, Papable Aljon will report a gross income of:
a. P90,000 c. P30,000
b. P45,000 d. P15,000

54. Using the same data in the preceding number, assuming Papable Aljon is a dealer in
paintings, how much will he report as gross income for 2021?
a. P15,000 c. P45,000
b. P30,000 d. P90,000

55. The results of operation of Yummy Convenient Store, not VAT-registered sole
proprietorship, owned by Ms. Alicia, for the first three (3) quarters of 2021 are
as follows:
Gross income Deductions
First quarter P300,000 P150,000
Second quarter 350,000 200,000
Third quarter 250,000 150,000

How much is taxable income for the third quarter assuming Ms. Alicia is married and
has five (5) qualified dependent children?
a. P400,000 c. P250,000
b. P300,000 d. P100,000

Page 7 of 14 0915-2303213  resacpareview@gmail.com


TAXATION
ReSA Batch 44 – October 2022 CPALE Batch
25 September 2022  8:00 AM to 11:00 AM TAX Final Pre-Board Exam
56. One of the following individuals is not subject to income tax in the Philippines.
a. An American engineer who makes the Philippines his residence because of the
nature of his work
b. A Japanese visiting professor who occasionally teaches at one of the leading
universities in the Philippines, and stays here for an aggregate of 185 days
c. A Taiwanese singer who performs in one of the hotels in the Philippines for
one (1) week, and then leaves after he is through with his performances
d. A Filipino residing and working in America who comes to the Philippines to
visit his relatives

57. A domestic corporation has the following data for three (3) years:
Normal income tax MCIT
2019 P 50,000 P 75,000
2020 P 60,000 P100,000
2021 P100,000 P 60,000

How much is the tax payable in 2021?


a. P100,000 c. P35,000
b. P 60,000 d. Zero

58. A VAT-registered seller sold goods to Asian Development Bank. It did not get
permission from the BIR to zero-rate the sale. As a consequence of the failure to
get the permission, the sale:
a. should be exempt from VAT.
b. should be subject to 12% VAT.
c. should be subject to 0% VAT.
d. should not be subject to any business tax.

59. Which of the following is allowed to claim presumptive input tax?


a. Processor of canned fruits
b. Miller of refined sugar not for his account
c. Miller of refined sugar for his own account
d. Public works contractor

60. Tax laws, being imposition of burden, are construed:


a. strictly against the taxpayer.
b. strictly against the government.
c. by filing a case before a regular court.
d. by strict adherence to opinions of tax expert.

61. On August 11, 2018, an individual taxpayer not engaged in real estate business acquired
a parcel of land which would be used as a site for office building for P3,000,000.
Unfortunately, the planned office building was not built due to financial problem that
hounded the taxpayer. The land was idle up to August 11, 2022. The taxpayer presented
a proof to the BIR that the land had not been used in business since its acquisition.
Subsequently, the land was sold by the taxpayer for P5,000,000 on September 5, 2022.

What was the tax consequence of the sale?


a. There was taxable gain of P2,000,000 which would be subject to Section 24 (A).
b. The sale would be subject to 6% capital gains tax based on the selling price.
c. The sale would be exempt from tax because the property was not used for
business.
d. The gain of P2,000,000 will be subject to 6% capital gains tax.

62. One of the following is deductible from the gross income even if not business related.
a. Rent expense c. Bad debts
b. Travel expenses d. Charitable contributions

63. An operator of an illegal horse-betting business, single, has the following data:
Receipts from illegal bets P 500,000
Rent of space where bets are received, gross of 5% withholding tax 20,000
Salaries of assistants, gross of creditable withholding tax 100,000
Bribe money to obtain protection from arrest and prosecution 50,000

How much is the income tax due?


a. P66,000 c. P25,000
b. P26,000 d. Not subject income tax

Page 8 of 14 0915-2303213  resacpareview@gmail.com


TAXATION
ReSA Batch 44 – October 2022 CPALE Batch
25 September 2022  8:00 AM to 11:00 AM TAX Final Pre-Board Exam
64. Maris bought from Pretty Corporation ten (10) shares of stock. Sixty (60) days
thereafter, the corporation was adjudged bankrupt and its stock as worthless. The
loss of Maris to be reported for income tax purposes is classified as:
a. a wagering loss.
b. non-deductible loss for income tax purposes.
c. short-term capital loss.
d. casualty loss.

65. Fly Me to the Moon, a resident international carrier, has the following data for
the current year: Gross income of P700,000 and expenses of P200,000 from the
Philippines; Gross income of P500,000 and expenses of P100,000 from Hongkong. How
much is the Philippine income tax payable of the corporation?
a. P288,000 c. P30,000
b. P160,000 d. P17,500

66. The imposition of this tax will require payment of corporate income tax even if the result
of operation is a net loss.
a. Optional corporate income
b. Minimum corporate income tax
c. Regular corporate income tax
d. Improperly accumulated earnings tax

67. The following data are presented to you by the administrator of the Estate of Mr.
X:
Gross sales P500,000
Cost of sales 200,000
Business expenses 50,000
Distribution of year’s income to the heirs 30,000

How much is the taxable income using optional standard deduction?


a. P300,000 c. P180,000
b. P270,000 d. P150,000

68. There is no taxable income until such income is recognized. Taxable income is
recognized when the:
a. taxpayer fails to include the income in his income tax return.
b. income has been actually received in money or its equivalent.
c. income has been received, either actually or constructively.
d. transaction that is the source of the income is consummated.

69. With the following data, how much is the deductible pension/retirement plan
contribution for 2022?
Pension/retirement plan contribution (2020) P500,000
Pension/retirement plan contribution (2021) P500,000
Pension/retirement plan contribution (2022) P500,000

The breakdown of the P500,000 per actuarial valuation:


Normal cost P400,000
Past service cost 100,000
Total P500,000
a. P500,000 c. P420,000
b. P430,000 d. P400,000

70. A system of taxation that places emphasis on direct rather than indirect taxation,
with ability to pay as the principal criterion.
a. Regressive tax system c. Global tax system
b. Progressive tax system d. Schedular tax system

- END of EXAMINATION -

Page 9 of 14 0915-2303213  resacpareview@gmail.com


TAXATION
ReSA Batch 44 – October 2022 CPALE Batch
25 September 2022  8:00 AM to 11:00 AM TAX Final Pre-Board Exam

ANSWERS & SOLUTIONS/CLARIFICATIONS


1 C 26 A 51 D
2 D 27 B 52 C
3 C 28 B 53 B
4 B 29 B 54 B
5 A 30 C 55 A
6 A 31 A 56 D
7 D 32 B 57 C
8 C 33 D 58 C
9 C 34 B 59 C
10 D 35 D 60 B
11 C 36 D 61 B
12 B 37 Bonus 62 D
13 B 38 A 63 B
14 A 39 B 64 C
15 A 40 D 65 D
16 C 41 D 66 B
17 B 42 D 67 A
18 B 43 C 68 C
19 A 44 D 69 B
20 C 45 B 70 B
21 B 46 B
22 B 47 A
23 B 48 D
24 D 49 B
25 B 50 B

3.
Services paid in cash for the month (336,000/1.12 P 300,000
Account receivable beginning (560,000/1.12) 500,000
Add: Sales on account (1,120,000/1,12) 1,000,000
Total 1,500,000
Less: Accounts receivable, ending (784,000/1.12) 700,000
Collections 800,000
Total 1,100,000
Tax rate 12%
Output tax P 132,000

9.
The Commissioner or his duly authorized representative shall, subject to rules and regulations
promulgated by the Secretary of Finance, upon recommendation of the Commissioner, have the
power to lift such order of distraint: Provided, further, That a consolidated report by the Revenue
Regional Director may be required by the Commissioner as often as necessary.

10.
The last day to appeal to the Court of Tax Appeals is April 15, 2002 which is within two (2) yeas
from the date of payment of tax.
Claims for refund must be elevated to the CTA before the expiration of the two-year period
because the prescriptive period will not be suspended regardless of any supervening event.

11.
Similarities among the three inherent powers:
1) They are inherent in the State;
2) They exist independently of the Constitution;
3) They constitute the three methods by which the State interferes with private rights and
property;
4) They are legislative in nature and character;
5) Each presupposes an equivalent compensation.

12.
All appropriation, revenue or tariff bills authorizing increase of the public debt, bills of application,
and private bills shall originate exclusively in the House of Representatives, but the Senate may
propose or concur with amendments (Art. VI, Sec. 24).

Page 10 of 14 0915-2303213  resacpareview@gmail.com


TAXATION
ReSA Batch 44 – October 2022 CPALE Batch
25 September 2022  8:00 AM to 11:00 AM TAX Final Pre-Board Exam
13.
Essential 1) It is an enforced contribution;
Characteristics of 2) It is levied pursuant to legislative authority;
Tax 3) It is proportionate in character;
4) It is payable in money;
5) It is levied on persons and property within the jurisdiction of the State;
6) It is levied and collected for the purpose of raising revenue to be used for public
purpose;
7) It is commonly required to be paid at regular intervals (not all taxes).

16.
Gross sales P250,000
Tax rate 1%
Percentage tax P 2,500

17.
BMBE refers to any business entity or enterprise engaged in the production, processing or manufacturing of
products or commodities, including agro-processing, trading and services, whose total assets including those
arising from loans but exclusive of the land on which the particular business entity's office, plant and
equipment are situated, shall not be more than Three Million Pesos (P3,000,000.00).

“Services" shall exclude those rendered by any one, who is duly licensed by the government after having
passed a government licensure examination, in connection with the exercise of one's profession.

20.
RBEs which are categorized as Domestic Market Enterprises (DME) are not entitled to VAT zero-rating on
local purchases.

Sale of goods or services to a registered domestic market enterprise shall be subject to VAT at 12%.

21.
Interest on deficiency from April 16, 2016 to June 30, 2017 (1,000,000 x 20% x 441/365 days)=
P241,643.83

22.
20% interest on deficiency from July 1, 2017 to December 31, 2017 (1,000,000 x 20% x 184/365 days) =
P100,821.91

23.
20% interest on delinquency from July 1, 2017 to December 31, 2017 (1,491,643.83 x 20% x 184/365
days) = P150,390.39

24.
12% interest on delinquency from January 1, 2018 to February 10, 2018 (1,491,643.83 x 12% x 41/365
days) = P20,106.54

25.
Basic tax due (income tax) P1,000,000.00
Add: 25% surcharge for late payment (25% x 1,000,000) 250,000.00
Interest on deficiency from April 16, 2016 to June 30, 2017
(1,000,000 x 20% x 441/365 days) 241,643.83 491,643.83
Total amount due, June 30, 2017 1,491,643.83
Add: 20% interest on deficiency from July 1, 2017 to December 31,
2017 (1,000,000 x 20% x 184/365 days) 100,821.91
20% interest on delinquency from July 1, 2017 to December 31,
2017 (1,491,643.83 x 20% x 184/365 days) 150,390.39
12% interest on delinquency from January 1, 2018 to February 10,
2018 (1,491,643.83 x 12% x 41/365 days) 20,106.54 271,318.84
Total amount due, February 10, 2018 P1,762,962.67

26.

Selling price P200,000


Less: Acquisition cost 150,000
Capital gain 50,000
Tax rate 15%
Capital gains tax P 7,500
Initial payments are P60,000 (30,000 + 30,000). Initial payments over selling price 60,000/200,000 is
30%. Capital gains tax cannot be paid in installment because initial payments exceed 25% over the
selling price.

Page 11 of 14 0915-2303213  resacpareview@gmail.com


TAXATION
ReSA Batch 44 – October 2022 CPALE Batch
25 September 2022  8:00 AM to 11:00 AM TAX Final Pre-Board Exam
28.
150,000/200 x 1.50 = P1,125

29-30
Using Reconciliation of Net Income Per Books Against Taxable Income
Net income per books P340,000
Add: Non-Deductible Expenses/Taxable Other Income
Provision for bad debts 80,000
Write-off of inventories lost due to spoilage 20,000
Bad debts recovery 30,000
Total 470,000
Less: A) Non-Taxable Income and Income Subject to Final Tax
Dividend income 50,000
Interest income 30,000
B) Special/Other Allowable Deductions
Accounts written-off 50,000
Total 130,000
Net Taxable Income (Loss) P340,000

32.
Gross sales P100,000,000.00
Less: Cost of sales 60,000,000.00
Gross income 40,000,000.00
Less: Operating expenses (before the
additional deduction) 17,500,000.00
Additional deduction from training
expenses
1/2 x 3,000,000.00) (lower) 1,500,000.00
10% x 20,000,000.00 2,000,000.00 1,500,000.00 19,000,000
Net taxable income P21,000,000.00

33.
Interest expense before interest arbitrage P2,000,000.00
Less: Interest arbitrage
January 1 to June 30, 2020
(300,000.00 x 33% x 6/12) 49,500.00
July 1, to December 31, 2020)
(300,000.00 x 0% x 6/12) 0.00 49,500.00
Allowable interest expense P1,950,500.00

34.
The P800,000.00 shall be treated as tax-exempt since it was properly utilized within 2022. On the other
hand, the P200,000.00 shall be declared as taxable income for the calendar year 2021, subject to surcharge,
interest and penalty, since the utilization in not within the following taxable year, which is in 2022.

36.
Gross sales P190,000,000.00
Less: Cost of sales 100,000,000.00
Gross income 90,000,000.00
Less: Allowable deductions 89,000,000.00
Net taxable income P 1,000,000.00
Normal/regular corporate income tax (1,000,000 x 25%) P 250,000.00
Minimum corporate income tax (not yet subject) -
Tax due (higher) P 250,000.00

37. BONUS
Gross sales P190,000,000.00
Less: Cost of sales 100,000,000.00
Gross income 90,000,000.00
Less: Allowable deductions 89,000,000.00
Net taxable income P 1,000,000.00
Normal/regular corporate income tax (1,000,000 x 25%) P 250,000.00
Minimum corporate income tax (90,000,000 x 1%) P 900,000.00
Tax due (higher) P 900,000.00

MCIT rate: 2% - Up to June 30, 2020; 1% - July 1, 2020 to June 30, 2023; 2% - July 1, 2023

Page 12 of 14 0915-2303213  resacpareview@gmail.com


TAXATION
ReSA Batch 44 – October 2022 CPALE Batch
25 September 2022  8:00 AM to 11:00 AM TAX Final Pre-Board Exam
42.
30,000,000/50,000,000 x 5,500,000 = 3,300,000

43.
Standard deduction – P500,000

44.
Gross estate P30,000,000
Less: Ordinary deductions 3,300,000
Net estate before special deductions 26,700,000
Less: Standard deduction 500,000
Net taxable estate 26,200,000
Tax rate 6%
Estate tax due P 1,572,000

45.
Selling price, shares held as inventory P3,000,000
Less: Acquisition cost, shares held as inventory 1,000,000
Gross income 2,000,000
Add: Other income, shares held as inventory 200,000
Gross receipts 2,200,000
Tax rate 12%
Output tax P 264,000

46.
Selling price, held as investment P2,000,000
Less: Acquisition cost, shares held as investment 1,500,000
Capital gain 500,000
Tax rate 15%
Capital gains tax P 75,000

47.
Selling price, held as investment P2,000,000
Tax rate .006
Stock transactions tax P 12,000

48.
Repealed under Sec. 6 RA No. 11494 (Bayanihan to Recover as One Act)
See Sec. 2 RR No. 23-2020

53.
Selling price P120,000
Less: Cost 30,000
Capital gain (long-term) P 90,000
Income to be recognized (50%) P 45,000

Initial payments over selling price = 40,000/120,000 = 33%

54.
Selling price P120,000
Less: Cost 30,000
Ordinary gain P 90,000

Income to be reported in 2021 (90,000/120,000 x 40,000 = 30,000)

55.
Gross income P250,000
Less: Deductions 150,000
Taxable income this quarter 100,000
Add: Taxable income, previous quarters
(150,000 + 150,000) 300,000
Taxable income to date P400,000

57.
Tax due (higher) P100,000
Less: Unexpired excess MCIT (25,000 + 40,000) 65,000
Tax payable P 35,000

Page 13 of 14 0915-2303213  resacpareview@gmail.com


TAXATION
ReSA Batch 44 – October 2022 CPALE Batch
25 September 2022  8:00 AM to 11:00 AM TAX Final Pre-Board Exam
63.
Gross receipts P500,000
Less: Rent expense 20,000
Salaries expense 100,000 120,000
Taxable net income P380,000
Tax due under Section 24 (A)250,000 0
130,000 x 20% 26,000 P 26,000

65.
Gross Philippines Billings P700,000
Rate 2.5%
Tax payable P 17,500

67.
Gross sales P500,000
Less: OSD (40% x 500,000) 200,000
Taxable net income P300,000

69.
Normal cost for 2022 P400,000
Add: 1/10 x 100,000 (2020) 10,000
1/10 x 100,000 (2021) 10,000
1/10 x 100,000 (2022) 10,000
Total P430,000

Page 14 of 14 0915-2303213  resacpareview@gmail.com


ReSA - THE REVIEW SCHOOL OF ACCOUNTANCY
CPA Review Batch 44  Oct 2022 CPALE  25 September 2022  03:00 PM – 06:00 PM

ADVANCED FINANCIAL ACCOUNTING and REPORTING FINAL PRE-BOARD EXAMINATION

INSTRUCTIONS: Select the correct answer for each of the questions. Mark only one
answer for each item by shading the box corresponding to the letter of your choice on
the answer sheet provided. STRICTLY NO ERASURES ALLOWED. Use pencil no. 2 only.

Set A
1. On October 2, 2028, Tamayao, Inc. ordered a custom-built passenger van from a
Japanese firm. The purchase order is noncancelable. The purchase price is 1,000,000
yens with delivery and payment to be made on March 31, 2029. On October 2, 2028,
Tamayao, Inc. entered into a forward contract to buy 1,000,000 yens on March 31,
2029 for P.57. On March 31, 2029, the custom-built passenger van was delivered.
10/2/28 12/31/28 3/31/29
Spot rate (yen) P .50 P .56 P .57
Forward rate (yen) .53 .58 .57

The December 31, 2028 profit and loss statement, foreign exchange gain or loss (on
hedged item / commitment) amounted to:
Fair Value Cash Flow Fair Value Cash Flow
Hedge Hedge Hedge Hedge
a. P60,000 loss P50,000 loss c. P50,000 loss P 0
b. P 0 P50,000 loss d. P50,000 gain P 0

2. Using the same information in No. 1. compute the December 31, 2028, foreign exchange
gain on forward contract amounted to (income statement or equity):
Fair Value Cash Flow Fair Value Cash Flow
Hedge Hedge Hedge Hedge
a. P50,000 I/S P50,000 equity c. P50,000 I/S Not applicable
b. P50,000 equity P50,000 I/S d. Not applicable P50,000 equity

3. Certain balance sheet accounts of a foreign subsidiary of Rose Company have been
stated in Philippine pesos as follows:
Stated
Current Rates Historical Rates
Accounts receivable, current P 200,000 P 220,000
Accounts receivable, long-term 100,000 110,000
Prepaid insurance 50,000 55,000
Goodwill 80,000 85,000
P 430,000 P 470,000
I. The subsidiary’s functional currency is the local currency unit. What amount
should Rose’s balance sheet include for the preceding items?
a. P430,000 b. P435,000 c. P440,000 d. P450,000

II. The subsidiary’s functional currency is peso. What total amount Rose’s balance
sheet include for the preceding items?
a. P430,000 b. P435,000 c. P440,000 d. P450,000
a. I – c; II – a c. I – a; II – c
b. I – a; II – d d. None of the above

4. The following are information regarding partnership business:

I. A partnership has the following capital balances:


Allen, capital P60,000
Burns, capital 30,000
Costello, capital 90,000

Profits and losses are split as follows: Allen (20%), Burns (30%), and Costello
(50%). Costello wants to leave the partnership and is paid P100,000 from the
business based on provisions in the articles of partnership. If the partnership
uses the bonus method, what is the balance of Burns’s capital account after
Costello withdraws?
a. P24,000 b. P27,000 c. P33,000 d. P36,000

Page 1 of 28 0915-2303213  resacpareview@gmail.com


ADVANCED FINANCIAL ACCOUNTING & REPORTING
ReSA Batch 44 – October 2022 CPALE Batch
25 September 2022  03:00 PM to 06:00 PM AFAR Final Pre-Board Exam
II. At year-end, the Cisco partnership has the following capital balances: Exam
Montana, capital P130,000
Rice, capital 110,000
Craig, capital 80,000
Taylor, capital 70,000
Profits and losses are split on a 3:3:2:2 basis, respectively. Craig decides to
leave the partnership and is paid P90,000 from the business based on the original
contractual agreement. If the goodwill method is to be applied, what is the
balance of Montana’s capital account after Craig withdraws?
a. P133,000 b. P137,500 c. P140,000 d. P145,000
a. I – a; II – d c. I – b; II – d
b. I – b; II – c d. None of the above

5. The following are information regarding a partnership undergoing liquidation:

I. A local partnership is liquidating and is currently reporting the following


capital balances:
Angela, capital (50% share of
all profits and losses) P 19,000
Woodrow, capital (30%) 18,000
Cassidy, capital (20%) (12,000)

Cassidy has indicated that a forthcoming contribution will cover the P12,000
deficit. However, the two remaining partners have asked to receive the P25,000
in cash that is presently available. How much of this money should each of the
partners be given?
a. Angela, P13,000; Woodrow, P12,000
b. Angela, P11,500; Woodrow, P13,500
c. Angela, P12,000; Woodrow, P13,000
d. Angela, P12,500; Woodrow, P12,500

II. A partnership has the following balance sheet just before the final liquidation
is to begin:
Cash P26,000 Liabilities P 50,000
Inventory 31,000 Art, capital (40%) 18,000
Other assets 62,000 Raymond, capital (30%) 25,000
Darby, capital (30%) 26,000
Total P119,000 Total P119,000

Liquidation expenses are estimated to be P12,000. The other assets are sold
for P40,000. What distribution can be made to the partners?
a. P-0- to Art, P1,500 to Raymond, P2,500 to Darby.
b. P1,333 to Art, P1,333 to Raymond, P1,334 to Darby.
c. P-0- to Art, P1,200 to Raymond, P2,800 to Darby.
d. P600 to Art, P1,200 to Raymond, P2,200 to Darby.
a. I – b; II – b c. I – b; II – a
b. I – c; II – a d. None of the above

6. Components of the December 17, 2028, statement of affairs of Liquo Company, which
was undergoing liquidation, included the following:
Assets pledged to fully secured creditors,
at current fair value…………………………………………………………………………………… P150,000
Assets pledged to partially secured creditors,
at current fair value…………………………………………………………………………………… 104,000
Free assets, at current fair value…………………………………………………………… 80,000
Fully secured liabilities…………………………………………………………………………………… 60,000
Partially secured liabilities………………………………………………………………………… 120,000
Unsecured liabilities with priority………………………………………………………… 14,000
Unsecured liabilities without priority…………………………………………………… 224,000

Determine the estimated payment to partially secured liabilities?


a. P 78,000 c. P115,333
b. P114,400 d. P115,143

Page 2 of 28 0915-2303213  resacpareview@gmail.com


ADVANCED FINANCIAL ACCOUNTING & REPORTING
ReSA Batch 44 – October 2022 CPALE Batch
25 September 2022  03:00 PM to 06:00 PM AFAR Final Pre-Board Exam
7. Agency NNN paid the final billing for Exam
the construction of a building was computed
as follows:

Final billings: 30% x P10,000,000……………………………………………………… P 3,000,000


Less: Liquidated damages……………………………………………………………………………… 10,000
Net cost………………………………………………………………………………………………………………………… P 2,990,000
Less: Recoupment of advances…………………………………………………………………… 450,000
Accounts payable balance……………………………………………………………………………… P 2,540,000
Less: Withholding tax (10% x P2,990,000) ………………………………… 299,000
Net amount………………………………………………………………………………… P 2,241,000

The entry to record the final payment of billings would be:


a. Accounts payable………………………………………………………………………… 2,540,000
Due to BIR………………………………………………………………………… 299,000
Cash-disbursing officer……………………………………… 2,241,000
b. Accounts payable………………………………………………………………………… 2,540,000
Due to BIR………………………………………………………………………… 299,000
Cash-MDS, Regular……………………………………………………… 2,241,000
c. Accounts payable………………………………………………………………………… 2,241,000
Cash-MDS, Regular …………………………………………………… 2,241,000
d. Buildings…………………………………………………………………………………………… 2,241,000
Cash-MDS, Regular …………………………………………………… 2,241,000

8. A chemical company manufactures joint products Pep and Vim, and a by-product. Zest.
Costs are assigned to the joint products by the market value method, which considers
further processing costs in subsequent operations. For allocating joint costs to the
by-product, the market value or reversal cost method is used. The total manufacturing
costs for 10,000 units were P172,000 during the quarter. Production and cost data
follow:
Pep Vim Zest
Units produced 5,000 4,000 1,000
Sales price per unit P50 P40 P 5
Further processing cost per unit 10 5 -
Selling and administrative expense per unit 2
Operating profit per unit 1

I. The value of Zest to be deducted from the joint costs is:


a. P5,000 b. P3,000 c. P2,000 d. Zero

II. Compute the gross profit for Pep:


a. P 0 b. P70,000 c. P 80,000 d. P100,000
a. I – c; II – a c. I – c; II – d
b. I – d; II – d d. None of the above

9. The following are information regarding parent and subsidiary:

I. Clark Company had the following transactions with affiliated parties during 2028:

• Sales of P60,000 to Dean, with P20,000 gross profit. Dean had P15,000 of this
inventory on hand at year-end. Clark owns a 15% interest in Dean and does not
exert significant influence.
• Purchases of raw materials totaling P240,000 from Kent Corporation, a wholly-
owned subsidiary. Kent’s gross profit on the sale was P48,000. Clark had
P60,000 of this inventory remaining on December 31, 2028.
Before eliminating entries, Clark had consolidated current assets of P320,000.
What amount should Clark report in its December 31, 2028, consolidated balance
sheet for current assets?
a. P320,000 b. P317,000 c. P308,000 d. P303,000

II. Par Company owns 60% of Sub Corp.’s outstanding capital stock. On May 1, 2028,
Par advanced Sub P70,000 in cash, which was still outstanding at December 31,
2028. What portion of this advance should be eliminated in the preparation of
the December 31, 2028 consolidated balance sheet?
a. P70,000 b. P42,000 c. P28,000 d. P 0
a. I – c; II – a c. I – c; II – d
b. I – d; II – d d. None of the above

Page 3 of 28 0915-2303213  resacpareview@gmail.com


ADVANCED FINANCIAL ACCOUNTING & REPORTING
ReSA Batch 44 – October 2022 CPALE Batch
AFAR Final Pre-Board Exam
25 September 2022  03:00 PM to 06:00 PM

10. On December 1, 20x8, a Philippine firm, Aldrin Inc. estimates that at leastExam
5,000
units of inventory will be purchased from a company in Taiwan during January of
20x9 for 500,000 Nt dollars. The transaction is probable, and it is to be
denominated in Nt dollar. Sales of the inventory are expected to occur in the six
months following the purchase.

The company enters into a forward contract to purchase 500,000 Nt dollars on


January 31, 20x9 for P1.01.

Spot rates and forward rates at the January 31, 20x9, settlement were as follows
(pesos per Nt dollar):
Forward Rate
Spot Rate for 1/31/x9
December 1, 20x8 P 1.03 P 1.01
December 31, 20x8 1.00 .99
January 31, 20x9 .98

The December 31, 20x8, foreign exchange loss on forward contract amounted to
(indicate whether income statement or equity section):
a. P30,000 separate component of equity/OCI
b. P30,000 current earnings
c. P10,000 current earnings
d. P10,000 separate component of equity/OCI

11. Using the same information in No. 10, the foreign exchange gain or loss on forward
contract on January 31, 20x9 amounted to (indicate whether income statement or
equity section)
a. P15,000 separate component of equity/OCI (debit balance)
b. P10,000 separate component of equity/OCI (debit balance)
c. P 5,000 separate component of equity/OCI (debit balance)
d. P15,000 current earnings

12. A company has identified the following overhead costs and cost drivers for the
coming year:
Overhead Item Cost Driver Budgeted Cost Budgeted Activity Level
Machine Setup Number of setups P 20,000 200
Inspection Number of Inspections
P 130,000 6,500
Material Number of Material moves
handling P 80,000 8,000
Engineering Engineering Hours P 50,000 1,000
P 280,000

The following information was allocated on three jobs that were completed during
the year:
Job 101 Job 102 Job 103
Direct materials P 5,000 P12,000 P 8,000
Direct labor P 2,000 P 2,000 P 4,000
Units completed 100 50 200
Number of setups 1 2 4
Number of inspections 20 10 30
Number of material moves 30 10 50
Engineering hours 10 50 10

Budgeted direct labor cost was P100,000 and budgeted direct material cost
was P280,000.

Compute the cost of each unit of Job 102 using Activity-Based Costing:
a. P340 c. P440
b. P392 d. P520

13. Lucille Inc. manufactures a product that gives rise to a by-product called "Robon."
The only costs associated with Robon are additional processing costs of P1.00 for
each unit. Lucille accounts for "Robon" sales first by deducting its separable
costs from such sales and then by deducting this net amount from the cost of sales
of the major product. For the past year 2,000 units of Robon were produced which
were sold for P3.00 each.

Page 4 of 28 0915-2303213  resacpareview@gmail.com


ADVANCED FINANCIAL ACCOUNTING & REPORTING
ReSA Batch 44 – October 2022 CPALE Batch
25 September 2022  03:00 PM to 06:00 PM AFAR Final Pre-Board Exam
Sales revenue and cost of goods sold Exam
from the main product were P500,000 and
P400,000 respectively. Compute the gross margin after considering the by-product
sales and costs.

If Lucille changes its method of accounting for Robon sales by showing the net
amount as "Other Income," the effect on the gross margin would be:
a. P 0 c. P4,000
b. P2,000 d. P6,000

14. On July 1, 20x4, Andres, Bantug, and Carlos formed a joint operation for the sale
of merchandise. Andres was designated as the managing operator. Profits or losses
are to be divided as follows: Andres, 50%; Bantug, 25%; and Carlos, 25%. On October
1, 20x4, though the joint operation was still uncompleted, the operators agreed to
recognize profit or loss on the operation to date. The cost of inventory on hand
was determined at P25,000. The joint operation account has a debit balance of
P15,000 before distribution of profit and loss. No separate books is maintained
for the joint operation and the operators record in their individual books all
operation transactions.

The joint operation profit or loss on October 1, 20x4 is:


a. P10,000 profit c. P15,000 loss
b. P25,000 profit d. No profit or loss.

15. On December 20, 20x8, United Appeal, a voluntary health and welfare organization,
received a donation of computer equipment valued at P25,000 from a local computer
retailer. The equipment is expected to have a useful life of 3 years. The donor
placed no restrictions how long the computer equipment was to be used and United
has an accounting policy which does not imply a time restriction on gifts of long-
lived assets. On United’s statement of activities prepared for the year ended
December 31, 20x8, the donation of computer equipment should be reported:
a. As an increase in temporary restricted net assets
b. Only in the notes to the financial statements
c. As an increase in unrestricted net assets
d. As either an increase in temporary restricted net assets or as an increase
in unrestricted net assets.

16. Falcon Corporation sold equipment to its 80%-owned subsidiary, Rodent Corp., on
January 1, 20x4. Falcon sold the equipment for P110,000 when its book value was
P85,000 and it had a 5-year remaining useful life with no expected salvage value.
Separate balance sheets for Falcon and Rodent included the following equipment and
accumulated depreciation amounts on December 31, 20x4:
Falcon Rodent
Equipment . . . . . . . . . . . . . . . . . . P 750,000 P 300,000
Less: Accumulated depreciation . . . . . . ( 200,000) ( 50,000)
Equipment-net . . . . . . . . . . . . . . . . . P 550,000 P 250,000

Consolidated amounts for equipment and accumulated depreciation at December 31,


20x4 were respectively,
a. P1,025,000 and P245,000 c. P1,050,000 and P245,000
b. P1,025,000 and P250,000 d. P1,050,000 and P250,000

17.On March 1, 20x8, Evan and Helen decide to combine their business and form a
partnership. The balance sheets of Evan and Helen on March 1, 20x8 before
adjustments show the following:
Evan Helen
Cash P 9,000 P 3,750
Accounts receivable 18,500 13,500
Inventories 30,000 19,500
Furniture and fixtures (net) 30,000 9,000
Office equipment (net) 11,500 2,750
Prepaid expenses 6,375 3,000
P105,375 P 51,500
Accounts payable P 45,750 P 18,000
Evan, capital 59,625
Helen, capital ________ 33,500
P105,375 P 51,500

Page 5 of 28 0915-2303213  resacpareview@gmail.com


ADVANCED FINANCIAL ACCOUNTING & REPORTING
ReSA Batch 44 – October 2022 CPALE Batch
25 September 2022  03:00 PM to 06:00 PM AFAR Final Pre-Board Exam
They agreed to provide 3% for Exam
doubtful accounts of their accounts
receivables and found Helen’s furniture and fixtures to be under-depreciated
by P900.

If each partner’s share in equity is to be equal to the net assets invested, the
capital accounts of Evan and Helen would be:
a. P58,170 and P33,095, respectively c. P 59,070 and P32,195, respectively
b. P58,320 and P32,495, respectively d. P104,820 and P50,195, respectively

18. The Pinoy Company acquired a foreign subsidiary on August 15, 20x4. Goodwill
arising on the acquisition was Nt Dollar 175,000. Consolidated financial statements
are prepared at the year end of December 31, 20x4 requiring the translation of all
foreign operations' results into the presentation currency of peso.

The following rates of exchange have been identified:


Rate at August 15, 20x4 Nt Dollar 1.321: P1
Rate at December 31, 20x4 Nt Dollar 1.298: P1
Average rate for the year ended December 31, 20x4 Nt Dollar 1.302: P1
Average rate for the period from August 15 to December 31, 20x4
Nt Dollar 1.292: P1

According to PAS 21 (The effects of changes in foreign exchange rates), at what


amount should the goodwill be measured in the consolidated statement of financial
position?
a. P134,409 c. P134,823
b. P135,449 d. P312,449

19. The Pinay Company acquired The Kanchengjunga Company, a foreign subsidiary, on
September 10, 20x4. The fair value of the assets of Kanchengjunga was the same
as their carrying amount except for land where the fair value was Nt dollar 50,000
greater than carrying amount. This fair value adjustment has not been recognized
in the separate financial statements of Kanchengjunga. Consolidated financial
statements are prepared at the year end of December 31, 20x4 requiring the
translation of all foreign operations' results into the presentation currency of
peso. The following rates of exchange have been identified:

Rate at 10 September 20x4 Nt Dollar 1.62: P1


Rate at 31 December 20x4 Nt Dollar 1.56: P1
Average rate for the year ended December 31, 20x4 Nt Dollar 1.60: P1
Average rate for the period from 10 September to December 31, 20x4
Nt Dollar 1.58: P1

According to PAS 21 (The effects of changes in foreign exchange rates), what fair
value adjustment is required to the carrying amount of land in the consolidated
statement of financial position?
a. P30,864 c. P31,250
b. P32,051 d. P31,646

20. The Witley Company has the peso as its functional currency. On October 16, 20x4
Witley ordered some inventory from a foreign supplier and agreed a purchase price
of 160,000 yens. The inventory was received on November 15, 20x4.

On December 31, 20x4 the inventory remained on hand and the trade payable balance
for the inventory purchase remained outstanding. The supplier was paid on January
27, 20x5 and the inventory was sold on January 31, 20x5.

The following information about exchange rates is available:


October 16, 20x4 P1 = 2.60 yens
November 15, 20x4 P1 = 2.50 yens
December 31, 20x4 P1 = 2.40 yens
January 27, 20x5 P1= 2.25 yens

According to PAS 21 (The effect of changes in foreign exchange rates), at what


amount should the trade payable balance due to the supplier be presented in the
statement of financial position of Witley on December 31, 20x4?
a. P61,538 c. P66,667
b. P64,000 d. P71,111

Page 6 of 28 0915-2303213  resacpareview@gmail.com


ADVANCED FINANCIAL ACCOUNTING & REPORTING
ReSA Batch 44 – October 2022 CPALE Batch
25 September 2022  03:00 PM to 06:00 PM AFAR Final Pre-Board Exam
21. Comely Company manufactures three Exam
products, R, S, and T, in a joint process. For
every 10 kilos of raw material input, the output is 5 kilos of R, 3 kilos of S and
2 kilos of T. During August, 50,000 kilos of raw material costing P120,000 were
processed and completed, with joint conversion costs of P200,000. Conversion costs
shall be allocated to the production on the basis of market values.

To make the product salable, however, further processing which does not require
additional material was done at the following costs: R, P30,000; S, P20,000 and T,
P30,000. Unit selling prices are R, P10; S, P12; and T, P15. The unit cost of
Product R is:
a. P7.12 c. P10.00
b. P8.00 d. P25.32

22. Kuchen Manufacturing uses backflush costing to account for an electronic meter it
makes. During August 2014, the firm produced 16,000 meters of which it sold 15,800.
The standard cost for each meter is:
Direct material P 20
Conversion costs 44
Total P 64

Assume that the company had no inventory on August 1. The following event took
place in August:
1. Purchased P320,000 of direct materials.
2. Incurred P708,000 of conversion costs.
3. Applied P704,000 of conversion costs to Raw and In Process Inventory.
4. Finished 16,000 meters.
5. Sold 15,800 meters for P100 each.

Compute the Finished Goods, ending and the amount of Cost of Goods Sold after the
adjustment of over-under applied conversion cost:
Finished Goods, ending Cost of Goods Sold as adjusted
a. P -0- P 1,015,200
b. 12,800 1,011,200
c. -0- 1,024,000
d. 12,800 1,015,200

23. Hartwell Company distributes the service department overhead costs to producing
departments and the following information for the month of January is presented
as follows:
Maintenance Utilities
Overhead costs incurred P18,700 P 9,000
Services provided to:
Maintenance department - 10%
Utilities department 20% -
Producing department A 40% 30%
Producing department B 40% 60%

Hartwell Company distributes service department overhead costs based on the


reciprocal method, what would be the formula to determine the total maintenance
costs?
a. M = P18,700 + .10U c. M = P18,700 + .30U +.40A + .40B
b. M = P 9,000 + .20U d. M = P27,700 + .40A + .40B

24. Cobb Company’s current receivables from affiliated companies at December 31, 20x9
are: (1) a P75,000 cash advance to Hill Corporation (Cobb owns 30% of the voting
stock of Hill and accounts for the investment by the equity method), (2) a
receivable of P260,000 from Vick Corporation for administrative and selling
services (Vick is 100% owned by Cobb and included in Cobb’s consolidated financial
statements), and (3) a receivable of P200,000 from Ward Corporation for merchandise
sales on credit (Ward is 90%-owned unconsolidated subsidiary of Cobb accounted for
the equity method). In the current assets section of its December 31, 20x9
consolidated balance sheet, Cobb should report accounts receivable from investees
in the amount of:
a. P180,000 c. P275,000
b. P255,000 d. P535,000

Page 7 of 28 0915-2303213  resacpareview@gmail.com


ADVANCED FINANCIAL ACCOUNTING & REPORTING
ReSA Batch 44 – October 2022 CPALE Batch
25 September 2022  03:00 PM to 06:00 PM AFAR Final Pre-Board Exam
25. The following information is available Exam
for KC Company for the month of June:
Started this month………………………………………………………………… 80,000 units
Beginning, WIP (40% complete)…………………………………… 7,500 units
Normal spoilage (discrete)…………………………………………… 1,100 units
Abnormal Spoilage (discrete)……………………………………… 900 units
Ending WIP (70% complete) …………………………………………… 13,000 units
Transferred –out……………………………………………………………………… 72,500 units
Beginning Work-in-Process costs:
Materials…………………………………………………………………………… P 10,400
Conversion costs………………………………………………………… 13,800
Current Costs: (added)
Materials…………………………………………………………………………… P 120,000
Conversion costs………………………………………………………… 350,000

All materials are added at the start of the production. Compute the cost per
equivalent unit for materials:
FIFO Average FIFO Average
a. P1.50 P1.49 c. P1.49 P1.50
b. P1.50 P1.50 d. P1.49 P1.49

26. Manila Sales Company established a branch in Baguio City early last year to which
it shipped merchandise before the branch opening with a billing price of P300,000.
During the year, the home office billed the branch a total of P120,000 for additional
shipments of merchandise. Some defective merchandise was shipped back by the branch
and was given credit for P7,500 on the return. The branch also made purchases of
merchandise totaling P72,500 from outside suppliers. At the end of the year, a
physical count disclosed a branch ending inventory of P185,000 which included
P20,000 of merchandise acquired from outside suppliers. If merchandise shipments
from the home office were billed at 20% above cost, what was the total cost of
merchandise available for sale, net of returns, at the branch during the year?
a. P300,000 c. P412,500
b. P343,750 d. P416,250

Items 27 and 28 are based on the following information:


The following information is available for K Co. for June:
Started this month 80,000 Units
Beginning WIP
(40% complete) 7,500 Units
Normal spoilage (discrete) 1,100 Units
Abnormal spoilage 900 Units
Ending WIP
(70% complete) 13,000 Units
Transferred out 72,500 Units

Beginning Work in Process Costs:


Material P 10,400
Conversion 13,800
Current Costs:
Material P120,000
Conversion 350,000

All materials are added at the start of production and the inspection
point is at the end of the process.

27. What is the cost assigned to ending inventory using FIFO?


a. P75,920 c. P56,420
b. P58,994 d. P53,144

28. What is the cost assigned to normal spoilage and how is it classified using weighted
average?
a. P6,193 allocated between WIP and Transferred Out
b. P6,424 assigned to units WIP
c. P6,193 assigned to loss account
d. P6,424 assigned to units Transferred Out

Page 8 of 28 0915-2303213  resacpareview@gmail.com


ADVANCED FINANCIAL ACCOUNTING & REPORTING
ReSA Batch 44 – October 2022 CPALE Batch
25 September 2022  03:00 PM to 06:00 PM AFAR Final Pre-Board Exam
29. Parcon Corporation owns an 80% Exam
interest in Shelly Corporation acquired several
years ago. Shelly Corporation regularly sells merchandise to its parent at 125%
of Shelly’s cost. Gross profit data of Parcon and Shelly for the year 20x8 are as
follows:
Parcon Shelly
Sales P1,000,000 P 800,000
Cost of goods sold 800,000 640,000
Gross profit P 200,000 P 160,000

During 20x8, Parcon purchased inventory items from Shelly at a transfer price of
P400,000. Parcon’s December 31, 20x7 and 20x8 inventories included goods acquired
from Shelly of P100,000 and P125,000, respectively.

Consolidated cost of goods sold of Parcon Corporation and Subsidiary for 20x8 was:
a. P1,024,000 c. P1,052,800
b. P1,045,000 d. P1,056,000

30. Splat Company filed a voluntary bankruptcy petition, and the statement of affairs
reflected the following amounts:
Estimated
Assets Book Value Current Value
Assets pledged with fully secured creditors… P 900,000 P 1,110,000
Assets pledged partially secured creditors………… 540,000 360,000
Free assets…………………………………………………………………………………………… 1,260,00 960,000
Liabilities
Liabilities with priority……………………………………………………… 210,000
Fully secured creditors…………………………………………………………… 780,000
Partially secured creditors………………………………………………… 600,000
Unsecured creditors……………………………………………………………………… 1,620,000

Assume the assets are converted to cash at their estimated current values. What
amount of cash will be available to pay unsecured non-priority claims?
a. P720,000 c. P 960,000
b. P840,000 d. P1,080,000

31. Mt. Carmel Hospital, a not profit hospital affiliated with a religious group,
reported the following information for the year ended December 31, 20x8:

Gross patient service revenue at the hospital’s full


established rates……………………………………………………………………………………… P 980,000
Bad debts expenses………………………………………………………………………………………………… 10,000
Contractual adjustments with third-party payors…………………… 100,000
Allowance for discounts to hospital employees………………………… 15,000

On the hospital’s statement of operations for the year ended December 31, 20x8,
what amount should be reported as net patient service revenue?
a. P865,000 c. P855,000
b. P880,000 d. P955,000

32. Some units of output failed to pass final inspection at the end of the manufacturing
process. The production and inspection supervisors determined that the incremental
revenue from reworking the units exceeded the cost of rework. The rework of the
defective units was authorized, and the following costs were incurred in reworking
the units:

Materials requisitioned from stores:


Direct materials………………………………………………………………… P 5,000
Miscellaneous supplies………………………………………………… 300
Direct labor…………………………………………………………………………………… 14,000
The manufacturing overhead budget includes an allowance for rework. The
predetermined manufacturing overhead rate is 150% of direct labor cost. The
account(s) to be charged and the appropriate charges for the rework cost would be:
a. Work-in-process inventory control for P19,000.
b. Work-in-process inventory control for P5,000 and factory overhead control for
P35,300.
c. Factory overhead control for P19,300.
d. Factory overhead control for P40,300.

Page 9 of 28 0915-2303213  resacpareview@gmail.com


ADVANCED FINANCIAL ACCOUNTING & REPORTING
ReSA Batch 44 – October 2022 CPALE Batch
AFAR Final Pre-Board Exam
25 September 2022  03:00 PM to 06:00 PM

Exam
33. The Porthos Manufacturing Company has a cycle of 3 days, uses a raw and in process
(RIP) account, and charges all conversion costs to Costs of Good Sold. At the end
month, all inventories are counted, their conversion cost components are estimated
and inventory account balances are adjusted. Raw material cost is back flushed from
RIP to Finished Goods. The following information is for June:

Beginning balance of RIP account, including P2,000 of conversion cost… P 15,000


Beginning balance of finished goods account, including P3,000 of
conversion cost…………………………………………………………………………………………………………………………………… 23,000
Raw materials credit on credit…………………………………………………………………………………………………… 500,000
Ending RIP inventory per physical count, including P2,500 conversion
cost estimate………………………………………………………………………………………………………………………………………… 22,500
Ending finished good inventory per physical count, including P1,000
conversion cost estimate…………………………………………………………………………………………………………… 16,000

Compute the amount of materials to be backflushed from Finished Goods to Cost of


Goods Sold:
a. P499,500 c. P498,000
b. P493,000 d. P500,000

34. Aguilar Sweets Factory manufactures a coconut candy, Coco, which is sold for P5.00
a box. The manufacturing process also results in a by-product Soloc. Without further
processing, Soloc sells for P1.00 per pack, with further processing, it sells for
P3.00 per pack.

During the month of April, the total joint manufacturing costs up to the point of
separation consisted of the following charges to work-n-process:

Raw materials…………………………………………………………………………………………………………… P 225,000


Direct labor……………………………………………………………………………………………………………… 100,000
Factory overhead…………………………………………………………………………………………………… 45,000

During the month, the production for the two products was as follows: Coco. 591,000
boxes; Soloc, 45,000 packs.

The following additional costs are necessary for further processing to complete
Soloc, in order to obtain a selling price of P3.00 per pack, during the month of
April:

Raw materials………………………………………………………………………………………………………………… P 30,000


Direct labor…………………………………………………………………………………………………………………… 22,500
Factory overhead………………………………………………………………………………………………………… 7,500

Assuming that the by-product Soloc, is further processed and then transferred to
the stockroom at net realizable value with a corresponding reduction of Coco’s
manufacturing costs, the journal entry would be:
A. By-product inventory – Soloc………………………………………………………… 45,000
Work-in-Process – Coco………………………………………………………………… 45,000

B. By-product inventory – Soloc………………………………………………………… 135,000


Raw materials………………………………………………………………………………………… 30,000
Direct labor…………………………………………………………………………………………… 22,500
Factory overhead………………………………………………………………………………… 7,500
Work-in-Process – Coco………………………………………………………………… 75,000

C. Work-in-Process – Soloc……………………………………………………………………… 6,750


Work-in-Process – Coco………………………………………………………………… 6,750

D. Work-in-Process – Soloc……………………………………………………………………… 60,000


Raw materials………………………………………………………………………………………… 30,000
Direct labor…………………………………………………………………………………………… 22,500
Factory overhead………………………………………………………………………………… 7,500

35. Following are situations regarding business combinations:

I. On June 1, 20x8, Cline Company paid P800,000 cash for all of the issued and
outstanding common stock Renn Corp. The carrying values for Renn’s assets and
liabilities on June 1, 20x8 follow:

Page 10 of 28 0915-2303213  resacpareview@gmail.com


ADVANCED FINANCIAL ACCOUNTING & REPORTING
ReSA Batch 44 – October 2022 CPALE Batch
AFAR Final Pre-Board Exam
25 September 2022  03:00 PM to 06:00 PM

Cash……………………………………………………………………………………………………………………………………… P150,000 Exam


Accounts receivable……………………………………………………………………………………………… 180,000
Capitalized software costs…………………………………………………………………………… 320,000
Goodwill…………………………………………………………………………………………………………………………… 100,000
Liabilities…………………………………………………………………………………………………………………… (130,000)
Net assets……………………………………………………………………………………………………………………… P 620,000

On June 1, 20x8, Renn’s accounts receivable had a fair value of P140,000.


Additionally, Renn’s in-process and development costs was estimated to have a
fair value of P200,000. All other items were stated at their fair values. On
Cline’s June 1 balance sheet. How much is reported for goodwill?

a. P320,000 b. P120,000 c. P80,000 d. P20,000

II. Prior to being united in a business combination, Atkins, Inc. and Waterson
Corporation had the following stockholders’ equity figures:

Atkins Waterson
Common stock, P1 par value P180,000 P 45,000
Additional paid-in capital 90,000 20,000
Retained earnings 300,000 110,000

Atkins issues 51,000 new shares of its common stock valued at P3 per share for
all of the outstanding stock of Waterson. Assume that Atkins acquires Waterson.
Immediately afterward, what are consolidated Additional Paid-In Capital and
Retained Earnings, respectively?

a. P104,000 and P300,000 c. P192,000 and P300,000


b. P110,000 and P410,000 d. P212,000 and P410,000

a. I – b; II – a c. I – c; II - d
b. I – b; II – c d. None of the above

36. For Job Order No. 369, Escalera Company incurred the following costs for the
manufacture of 200 units of a novelty gadget:

Original cost accumulation:


Direct materials……………………………………………………………………………………………… P 13,200
Direct labor………………………………………………………………………………………………………… 16,000
Factory overhead (150% of direct labor)………………………………… 24,000
Total…………………………………………………………………………………………………………………………… P 53,200
Direct costs of ten reworked units:
Direct materials……………………………………………………………………………………………… P 2,000
Direct labor………………………………………………………………………………………………………… 3,200
Total…………………………………………………………………………………………………………………………… P 5,200

The rework cost was attributable to exacting specifications required by the job
and was charged to the specific order. The units cost of Job Order No. 369 is:
a. P266 c. P292
b. P280 d. P316

37. The following are information regarding a non-profit organization:

I. On December 30, 20x8, Leigh Museum, a not-for-profit organization received a


P7,000,000 donation of Day Company shares with donor-stipulated requirements as
follows:
• Shares valued at P5,000,000 are to be sold, with the proceeds used to erect
a public viewing building.
• Shares valued at P2,000,000 are to be retained (invested indefinitely), with
the dividends used to support current operations.

As a consequence of the receipt of the Day shares, how much should Leigh report
as temporarily restricted net assets on its 20x8 statement of financial position
(balance sheet)?
a. P 0 b. P2,000,000 c. P5,000,000 d. P7,000,000

Page 11 of 28 0915-2303213  resacpareview@gmail.com


ADVANCED FINANCIAL ACCOUNTING & REPORTING
ReSA Batch 44 – October 2022 CPALE Batch
25 September 2022  03:00 PM to 06:00 PM AFAR Final Pre-Board Exam
II. An organization of high school Exam
seniors performs a volunteer services for
patients at a nearby nursing home. The nursing home would not otherwise provide
these services, such as wheeling patients in the park and reading to them. At
the minimum wage rate, these services would amount to P21,320, but their actual
value is estimated to be P27,400. In the nursing home’s statement of revenues
and expenses, what amount should be reported in public support?
a. P 27,400 b. P 21,320 c. P 6,080 d. P 0

a. I – b; II – b c. I – c; II – d
b. I – d; II – d d. None of the above

Items 38 and 39 are based on the following information:


The income statement submitted by the Pampanga Branch to the Home Office for the month
of December 2020 is shown below. After effecting the necessary adjustments, the true
net income of the Branch was ascertained to be P156,000.
Sales ………………………………………………………………………………………………… P 600,000
Cost of sales:
Inventory, December 1…………………………………………………… P 80,000
Shipments from Home Office……………………………………… 350,000
Local purchases…………………………………………………………………… 30,000
Total available for sale…………………………………………… P460,000
Inventory, December 31………………………………………………… 100,000 360,000
Gross margin ……………………………………………………………………………… P240,000
Operating expenses ……………………………………………………………… 180,000
Net income P 60,000
The branch inventories were:
12/01/2020 12/31/2020
Merchandise from home office……………………………………… P 70,000 P 84,000
Local purchases………………………………………………………………………… 10,000 16,000
Total ………………………………………………………………………………………………… P 80,000 P100,000

38. The billing price based on cost imposed by the home office to the branch;
a. 140% c. 40%
b. 100% d. 29%

39. The balance of allowance for overvaluation of branch December 31, 2020 after
adjustment.
a. P10,000 c. P16,000
b. P24,000 d. None of the above

40. Pasig Garment Company operates a branch in Cabanatuan City. At the end of the year,
the Branch account in the books of the home office at Manila shows a balance of
P150,000. The following information are ascertained:

1. The home office has billed the branch the amount of P37,500 for the merchandise,
which was in transit on December 31.
2. A home office accounts receivable for P10,500 was collected by the branch.
Said collection was not reported to the home office by the branch.
3. Supplies of P4,500 was returned by the branch to the home office but the home
office has not yet reflected in its records the receipt of the supplies.
4. The branch made profit of P10,100 for the month of December but the home office
erroneously recorded it as P11,180.
5. The branch has not received the cash in the amount of P25,000 sent by home
office on December 31. This was charged to General Expense account.

All transactions are presumed to have been properly recorded.

What is the balance of the Home Office account on the books of the branch as of
December 31, before adjustments?
a. P121,920 c. P117,420
b. P123,000 d. P106,920

41. On May 1, RR Products Company ships five (5) of its appliances to SZ Company on
consignment. The cost of the appliances shipped is P155 per unit. The consignor
paid shipping costs totaling P50. Each unit is to be sold at P250 payable P50 in
the month of purchase and P10 per month thereafter. The consignee is entitled to
20% of all amounts collected on consignment sales.

Page 12 of 28 0915-2303213  resacpareview@gmail.com


ADVANCED FINANCIAL ACCOUNTING & REPORTING
ReSA Batch 44 – October 2022 CPALE Batch
25 September 2022  03:00 PM to 06:00 PM AFAR Final Pre-Board Exam
SZ Company was able to sell 3 Exam
appliances in May and 1 in June. Regular monthly
collections by the consignee, and appropriate cash remittances have been made to
the consignor at the end of each month. The profit on consignment is:
a. P294 c. P150
b. P160 d. P140

42. The Carly Company owns 75% of The Halley Company. The following figures are
from their separate financial statements:

Carly: Trade receivables P1,040,000, including P30,000 due from Halley.


Halley: Trade receivables P215,000, including P40,000 due from Carly.

According to PAS 27 Consolidated and separate financial statements, what figure


should appear for trade receivables in Carly's consolidated statement of financial
position?
a. P1,215,000 c. P1,255,000
b. P1,225,000 d. P1,185,000

43. The White Company acquired an 80% interest in The Pulley Company when Pulley's
equity comprised share capital of P100,000 and retained earnings of P500,000.
Pulley's current statement of financial position shows share capital of P100,000,
a revaluation reserve of P400,000 and retained earnings of P1,400,000.
Under PAS 27 Consolidated and separate financial statements, what figure in respect
of Pulley's retained earnings should be included in the consolidated statement of
financial position?
a. P 720,000 c. P1,040,000
b. P1,440,000 d. P1,520,000

44. The Snipes Company owns 65% of The Genie Company. On the last day of the accounting
period Genie sold to Snipes a non-current asset for P200,000. The asset originally
cost P500,000 and at the end of the reporting period, its carrying amount in Genie's
books was P160,000. The group's consolidated statement of financial position has
been drafted without any adjustments in relation to this non-current asset.

Under PAS27 Consolidated and separate financial statements, what adjustments should
be made to the consolidated statement of financial position figures for non-current
assets and retained earnings?
Non-current assets Retained earnings
a. Increase by P300,000 Increase by P195,000
b. Reduce by P40,000 Reduce by P26,000
c. Reduce by P40,000 Reduce by P40,000
d. Increase by P300,000 Increase by P300,000
45. Wynn, Inc. has a contract to construct a large hotel for P12,000,000. The contract
was signed on January 2, 20x4 and it was expected that the hotel would be complete
on December 31, 20x7. At the date the contract was signed, Wynn, Inc. anticipated
the costs of construction would total P11,000,000. At the end of 20x5 the total
cost estimate rose to P11,870,000 and at the end of 20x6 the total cost estimate
rose to P12,400,000. Due to certain circumstances, Wynn, Inc. believes there are
inherent hazards in the contract beyond the normal, recurring business risks. Wynn,
Inc. expects to recover all its costs under the contract. Under these conditions,
what amount of loss, if any, should Wynn, Inc. recognize in each of the following
years?
20x5 20x6 20x5 20x6
a. P870,000 P400,000 c. P870,000 P530,000
b. P -0- P400,000 d. P -0- P -0-
46. Wynn, Inc. has a contract to construct a large hotel for P12,000,000. The contract
was signed on January 2, 20x4 and it was expected that the hotel would be complete
on December 31, 20x7. At the date the contract was signed, Wynn, Inc. anticipated
the costs of construction would total P11,000,000. At the end of 20x5 the costs
incurred were P3,490,000 and its estimate of total contract costs rose to
P11,870,000. During 20x6, the company incurred costs of P4,020,000 and by the end
of 20x6 the total cost estimate rose to P12,400,000. Due to certain circumstances,
Wynn, Inc. believes there are inherent hazards in the contract beyond the normal,
recurring business risks. Wynn, Inc. expects to recover all its costs under the
contract. Under these conditions, what amount of revenue should Wynn, Inc. recognize
in each of the following years?

Page 13 of 28 0915-2303213  resacpareview@gmail.com


ADVANCED FINANCIAL ACCOUNTING & REPORTING
ReSA Batch 44 – October 2022 CPALE Batch
25 September 2022  03:00 PM to 06:00 PM AFAR Final Pre-Board Exam
20x5 20x6 20x5 20x6 Exam
a. P3,490,000 P4,020,000 c. P3,528,222 P3,890,323
b. P -0- P 400,000 d. P8,380,000 P4,890,000

47. An entity purchases plant from a foreign supplier for 3 million baht on January 31,
20x6, when the exchange rate was 2 baht = P1. At the entity’s year-end of March 31,
20x6, the amount has not been paid. The closing rate was 1.5 baht = P1. The entity’s
functional currency is the peso.
Which of the following statements is correct?

a. Cost of plant, P2 million, exchange loss P0.5 million, trade payable


P1.5 million.
b. Cost of plant P1.5 million, exchange loss P0.6 million, trade payable
P2 million.
c. Cost of plant P1.5 million, exchange loss P0.5 million, trade payable
P2 million.
d. Cost of plant P2 million, exchange loss P0.5 million, trade payable P2
million.

48. An entity acquired all the share capital of a foreign entity at a consideration
of 9 million baht on June 30, 20x9. The fair value of the net assets of the foreign
entity at that date was 6 million baht. The functional currency of the entity is
the peso. The financial year-end of the entity is December 31, 20x9. The exchange
rates at June 30, 20x9, and December 31, 20x9, were 1.5 baht = P1 and 2 baht = P1
respectively.

What figure for goodwill should be included in the financial statements for the
year ended December 31, 20x9?

a. P2 million c. P1.5 million


b. 3 million baht d. P3 million

49. An entity acquired 60% of the share capital of a foreign entity on June 30, 20x2.
The fair value of the net assets of the foreign entity at that date was 6 million
baht. This value was 1.2 million higher than the carrying amount of the net assets
of the foreign entity. The excess was due to the increase in value of non-depreciable
land. The functional currency of the entity is the peso. The financial year-end of
the entity is December 31, 20x2. The exchange rates at June 30, 20x2, and December
31, 20x2, were .5 baht = P1 and 2 baht = P1, respectively.

What figure for the fair value adjustments should be included in the group financial
statements for the year ended December 31, 20x2?

a. P600,000 c. P2 million
b. P800,000 d. P3 million

50. Property was purchased on December 31, 2019 for 20 million baht. The general price
index in the country was 60.1 on that date. On December 31, 2021, the general price
index had risen to 240.4. If the entity operates in a hyperinflationary economy,
what would be the carrying amount in the financial statements of the property after
restatement?

a. 20 million baht c. 80 million baht


b. 1,200.2 million baht d. 4.808 million baht

51. Agency 007 received a request for replenishment of petty cash fund for the
following expenses:

Office supplies P 500


Transportation fares 100
Repair of aircon 200
JRS mail 160

Page 14 of 28 0915-2303213  resacpareview@gmail.com


ADVANCED FINANCIAL ACCOUNTING & REPORTING
ReSA Batch 44 – October 2022 CPALE Batch
25 September 2022  03:00 PM to 06:00 PM AFAR Final Pre-Board Exam
The entry for this transaction would be: Exam
a. No entry
b. Memorandum entry to the RAOD-MOOE
c. Office supplies expense…………………………………………………… 500
Travelling expense………………………………………………………………… 100
Repairs and maintenance…………………………………………………… 200
Other maintenance and operating expenses………… 160
Cash – National Treasury, MDS…………………… 960
d. Office supplies expense……………………………………………………… 500
Travelling expense…………………………………………………………………… 100
Repairs and maintenance……………………………………………………… 200
Other maintenance and operating expenses………… 160
Petty Cash Fund………………………………………………… 960

52. Agency AAA has the following allotment for year 20x8:

Capital Outlay (CO) P 70,000,000


Maintenance and Other Operating Expenses (MOOE) 14,000,000
Personal Services (PS) 7,000,000
Financial Expenses (FE) ____140,000
P91,140,000

The entry to record the receipt of allotment from DBM would be:

a. No entry
b. Memorandum entry/Posting in Registry of Allotments and Obligations
c. National Clearing Account 91,140,000
Appropriations Allotted 91,140,000
d. Cash-Modified Disb. System 91,140,000
Subsidy from National Government 91,140,000
e. None of the above

53. On January 1, 2021 SME A and B each acquired 30 per cent of the ordinary shares
that carry voting rights at a general meeting of shareholders of entity Z for
P300,000. Entities A and B immediately agreed to share control over entity Z. For
the year ended December 31, 2021 entity Z recognized a profit of P400,000.

On January 2, 2021 entity Z also declared a dividend of P100,000 for the year
2020.

On December 30, 2021 entity Z declared and paid a dividend of P150,000 for the
year 2021. At December 31, 2021 the fair value of each venturers’ investment in
entity Z is P400,000. However, there is no published price quotation for entity
Z.

SME A and B must each recognize dividend income for the year 2021 amounted to:
Cost Model Fair Value Model Cost Model Fair value Model
a. P 45,000 P75,000 c. P 75,000 P75,000
b. P 75,000 P45,000 d. None

54. In activity-based costing, preliminary cost allocations assign costs to:


a. departments. c. products.
b. processes. d. activities.

55. Which of the following costing methods of valuation are acceptable in a job order
costing system?
Actual Standard Actual Predetermined
Material Material Labor Overhead
Cost Cost Cost Cost
a. yes yes no yes
b. yes no yes no
c. no yes yes yes
d. yes yes yes yes

Page 15 of 28 0915-2303213  resacpareview@gmail.com


ADVANCED FINANCIAL ACCOUNTING & REPORTING
ReSA Batch 44 – October 2022 CPALE Batch
AFAR Final Pre-Board Exam
25 September 2022  03:00 PM to 06:00 PM

Exam
56. Job order costing and process costing have which of the following characteristics?
Job Order Costing Process Costing
a. homogeneous products heterogeneous products
and large quantities and small quantities
b. homogeneous products heterogeneous products
and small quantities and large quantities
c. heterogeneous products homogeneous products
and large quantities and small quantities
d. heterogeneous products homogeneous products
and small quantities and large quantities

57. A hybrid costing system combines characteristics of


a. job order and standard costing systems.
b. job order and process costing systems.
c. process and standard costing systems.
d. job order and normal costing systems.

58. Averaging the total cost of completed beginning inventory and units started and
completed over all units transferred out is known as
a. strict FIFO.
b. modified FIFO.
c. weighted average costing.
d. normal costing.

59. The primary difference between the FIFO and weighted average methods of process
costing is
a. in the treatment of beginning Work in Process Inventory.
b. in the treatment of current period production costs.
c. in the treatment of spoiled units.
d. none of the above.

60. Which of the following has sales value?


By-products Waste
a. no no
b. yes no
c. yes yes
d. no yes

61. Company B acquired the net assets of Company S in exchange for cash. The acquisition
price exceeds the fair value of the net assets acquired. How should Company B
determine the amounts to be reported for the plant and equipment, and for long-
term debt of the acquired Company S?
Plant and Equipment Long-Term Debt
a. Fair value S’s carrying amount
b. Fair value Fair value
c. S’s carrying amount Fair value
d. S’s carrying amount S’s carrying amount

62. In 2025, Palex sold inventory costing P45,000 to its 100%-owned subsidiary, Salex,
for P70,000. By 12/31/25, Salex had resold all this inventory for P100,000. Which
of the following accounts would have to be eliminated in consolidation at 12/31/25?
Intercompany Sales Intercompany Cost of Sales
a. Yes Yes
b. No No
c. Yes No
d. No Yes

63. P Corp. owns 90% of the outstanding common stock of S Company. On December 31,
2024, S sold equipment to P for an amount greater than the equipment’s book value
but less than its original cost. The equipment should be reported on the December
31, 2024 consolidated balance sheet at
a. P’s original cost less 90% of S’s recorded gain.
b. P’s original cost less S’s recorded gain.
c. S’s original cost.
d. P’s original cost.

Page 16 of 28 0915-2303213  resacpareview@gmail.com


ADVANCED FINANCIAL ACCOUNTING & REPORTING
ReSA Batch 44 – October 2022 CPALE Batch
AFAR Final Pre-Board Exam
25 September 2022  03:00 PM to 06:00 PM

Exam
64. Philippine based Corporation X has a number of importing transactions with companies
based in UK. Importing activities result in payables. If the settlement currency
is the British Pound, which of the following will happen by changes in the direct
or indirect exchange rates?
Direct Exchange Rate Indirect Exchange Rate
Increases Decreases Increases Decreases
a. NA NA NA NA
b. Loss Gain Gain Loss
c. Loss Gain NA NA
d. Gain Loss Loss Gain
65. Hedging a forecasted transaction is a
a. Cash flow hedge. d. Undesignated hedge.
b. Fair value hedge. e. None of the above
c. Net investment hedge.
66. FX (foreign currency) forwards are valued using
a. The change in the forward rate.
b. The change in the spot rate.
c. The change in the forward rate or the spot rate, depending on whether the
hedge is a fair value hedge or cash flow hedge.
d. The change in the intrinsic value.
e. None of the above.
67. In accordance with PAS 21 (generally accepted accounting principles), which
translation combination is appropriate for a foreign operation whose functional
currency is the U.S. dollar?
Method Treatment of Translation adjustment
a. Temporal Other comprehensive income
b. Temporal Gain or loss in net income
c. Current rate Other comprehensive income
d. Current rate Gain or loss in net income
68. At what rates should the following balance sheet accounts in foreign statements be
translated (rather than remeasured) into pesos?
Accumulated
Depreciation—Equipment Equipment
a. Current Current
b. Current Average for year
c. Historical Current
d. Historical Historical

69. Which of the following accounts is not a monetary item?


a. Accounts Receivable d. Accrued liabilities
b. Inventory e. None of the above
c. Accounts payable
70. Which of the following statements is not correct?
a. Joint arrangements may be entered into to manage risks involved in a project.
b. Joint arrangements may be entered into to provide the parties with access to
new technology or new markets.
c. Joint arrangements require investors to have equal interests in the joint
arrangement.
d. The key feature of a joint arrangement is that the parties involved have
joint control over the decision making in relation to the joint arrangement.

End of Examination
Goodluck and GOD BLESS!!!
*When GOD measures a man, He puts the tape around the heart instead of the head.*
*Until you make peace with who you are, you’ll never be content with what you have.*
*Only passions, great passions, can elevate the soul to great things.*
*Most of the things worth doing in the world had been declared impossible before they were done.*
*The world belongs to the man who is wise enough to change his mind in the presence of facts.*
*There are only two things in the world to worry over; the things you can control,
and the things you can’t control. Fix the first forget the second.*
*No act of kindness, no matter how small is ever wasted. *
*One individual plus courage is a majority. *
*There is no great and no small to the Soul that makes it all:
And where it comes, all things are equal; And it comes everywhere.*
***Ask not for a larger garden, but for a finer seeds***
***Ask not for a lighter burden, but for a broader shoulder***
***There are divine things more beautiful than words can tell***
*The only thing that stands between a man and what he wants from life is often merely the will to try it and the faith to believe that it is possible*
*In every trial, there’s a treasure waiting to be unearthed*
*Never take direction from a crowd for your personal life. And never choose to quit just because somebody disagrees with you*
*Opportunities are usually disguised as hardwork, so most people don’t recognize them

Page 17 of 28 0915-2303213  resacpareview@gmail.com


ADVANCED FINANCIAL ACCOUNTING & REPORTING
ReSA Batch 44 – October 2022 CPALE Batch
25 September 2022  03:00 PM to 06:00 PM AFAR Final Pre-Board Exam
Exam
ANSWERS & SOLUTIONS/CLARIFICATIONS
1 C 26 D 51 B
2 A 27 B 52 B
3 C 28 D 53 C
4 A 29 B 54 D
5 C 30 D 55 D
6 B 31 A 56 D
7 B 32 D 57 B
8 C 33 C 58 B
9 A 34 B 59 A
10 D 35 B 60 B
11 A 36 D 61 B
12 A 37 C 62 A
13 C 38 A 63 B
14 A 39 B 64 B
15 C 40 C 65 A
16 A 41 D 66 A
17 C 42 D 67 C
18 C 43 A 68 A
19 B 44 B 69 B
20 C 45 B 70 C
21 A 46 A
22 D 47 C
23 A 48 C
24 C 49 A
25 A 50 C

1. (C)
Fair value hedge
10/02/2028: Original forward rate (180 days)……………………………………P .53
12/31/2028: Current (remaining) forward rate (90 days)……………… .58
Forex loss per unit.......…………………………………………………………………………………P .05
Multiplied by: Number of foreign currencies……………………………………………… 1,000,000
Foreign exchange loss due to hedged item/commitment……………………… P 50,000 (C)

The forward rate generally differs from the spot rate, but as one moves closer
to the expiration date (or settlement date), the difference between the spot
rate and the forward rate for the remaining period of the contract becomes
smaller and smaller so that at the expiration date, the forward rate will have
converged with the spot rate.

Protecting against an adverse change in the exchange rate between the order date
(commitment date) and the transaction date is hedging a firm foreign-currency-
denominated commitment.

Cash Flow Hedge - Not applicable [No entry yet on October 02, 2028 and December
31, 2028].

2. (A)
Fair value hedge – Income Statement
10/02/2028: Original forward rate (180 days……………………………………P .53
12/31/2028: Current (remaining) forward rate (90 days……… .58
Forex gain per unit.......…………………………………………………………………………………P .05
Multiplied by: Number of foreign currencies……………………………………………… 1,000,000
Foreign exchange gain due to forward contract – I/S……………… P 50,000 (A)

Cash flow hedge - Equity


10/02/2028: Original forward rate (180 days)..……………………………….P .53
12/31/2028: Current (remaining) forward rate (90 days)………… .58
Forex gain per unit.......………………………………………………………………………………………P .05
Multiplied by: Number of foreign currencies…………………………………………... 1,000,000
Foreign exchange gain due to forward contract – OCI (Equity)P 50,000 (A)

Page 18 of 28 0915-2303213  resacpareview@gmail.com


ADVANCED FINANCIAL ACCOUNTING & REPORTING
ReSA Batch 44 – October 2022 CPALE Batch
25 September 2022  03:00 PM to 06:00 PM AFAR Final Pre-Board Exam
The forward rate is the rate quoted for the exchange of two currenciesExam
at a
specified future date. It differs from the spot rate because of the difference in
interest rates in the international financial markets.

A premium exists on an foreign exchange forward when a party buys or sells forward
at more than the spot rate. A discount exists on an foreign exchange forward when
a party buys or sells forward at less than the spot rate.

For recording purposes, premiums or discounts have no bearing at all meaning there
is no need to set-up such account. However, for option contracts wherein the
writer assumes the responsibility of incurring a potential loss, the writer charges
a fee called a premium. Thus, the premium is the price paid to acquire the option.

The forward rate generally differs from the spot rate, but as one moves closer to
the expiration date (or settlement date), the difference between the spot rate
and the forward rate for the remaining period of the contract becomes smaller and
smaller so that at the expiration date, the forward rate will have converged with
the spot rate.

To determine if a gain or loss on forward contracts occurred during any two


dates, always view:
(1) the forward rate at the inception date as the buying rate (when buying
forward), or the selling rate (if selling forward), and
(2) all subsequent forward rates as the opposite rate. Because the forward rate
at inception is fixed, merely ask: “Did the opposite rate mover favorably or
unfavorably?” An increase in the selling rate is favorable, whereas an
increase in the buying rate is unfavorable.

It should be noted that on the settlement date, the spot rate will be used since
the spot rate on that date is simply the same with the forward rate also on the
same date. .

3. (C)
I. - a
The foreign currency is the functional currency, so a translation method or
closing rate method is appropriate. All assets are translated at the current
exchange rate of P430,000.
II – c
Because the peso is the functional currency, a remeasurement (temporal method)
is required. All receivables are remeasured at current rates. Assets carried at
historical cost, such as prepaid insurance and goodwill, are remeasured at
historical rates.
4. (A)
I. - a
A P10,000 bonus is paid to Costello (P100,000 is paid rather than the P90,000
capital balance). This bonus is deducted from the two remaining partners according
to their profit and loss ratio (2:3). A reduction of 60 percent (3/5) is assigned
to Burns or a decrease of P6,000 which drops that partner’s capital balance from
P30,000 to P24,000.
Or,
Amount paid…………………………………………………………………………………………………P100,000
Less: Book value of interest of Costello (50%)…… 90,000
Bonus to retiring partners…………………………………………………………P 10,000
Burns, capital: P30,000 - (P10,000 x 3/5)…………………P 24,000
I. - d
Craig receives an additional $10,000. Since Craig is assigned 20 percent of all
profits and losses, this allocation indicates total goodwill of P50,000.
20% of Goodwill = P10,000
.20 G = P10,000
G = P10,000/.20
G = P50,000
Montana is assigned 30% of all profits and losses and would, therefore, record
P15,000 of this goodwill, an entry that raises this partner's capital balance from
P130,000 to P145,000.
Or,
Page 19 of 28 0915-2303213  resacpareview@gmail.com
ADVANCED FINANCIAL ACCOUNTING & REPORTING
ReSA Batch 44 – October 2022 CPALE Batch
AFAR Final Pre-Board Exam
25 September 2022  03:00 PM to 06:00 PM

Amount paid…………………………………………………………………………………………………P 90,000 Exam


Less: Book value of interest of Craig (20%)…………… 80,000
Excess………………………………………………………………………………………………………………P 10,000
Divided by…………………………………………………………………………………………………… 20%
Goodwill – total implied………………………………………………P 50,000
Montana, capital: P130,000 + (P50,000 x 30%)…………P145,000
5. (C)
I. - b
Angela Woodrow Cassidy
Reported balances P19,000 P18,000 P(12,000)
Potential loss from
Cassidy deficit
(split 5/8:3/8) ( 7,500) (4,500) 12,000
Cash distributions P11,500 P13,500 P -0-

II. - a
Art Raymond Darby Total
Reported balances P18,000 P25,000 P26,000 P69,000
Possible loss (26,000) (19,500) (19,500) (65,000)
P(8,000) P 5,500 P 6,500 P 4,000
Possible insolvency (3:3) 8,000 ( 4,000) ( 4,000) -0-
Payment to Partners P 1,500 P 2,500 P 4,000
6. (B)
Free Assets:
Assets pledged to fully secured liabilities (P150,000 – P60,000)…P 90,000
Free Assets…………………………………………………………………………..……………………………………………………….. 80,000
Total Free Assets…………………………………………..…………………………………………………………………………………P170,000
Less: Unsecured liabilities with priority………………………………………………………….. 14,000
Net Free Assets………………………………………………………………………………………………………………..…………………P156,000
Divided by: Unsecured Liabilities without priority:
Partially secured liabilities (P120,000 – P104,000)P16,000
Add: Unsecured liabilities without priority……….……. 224,000 240,000
Expected Recovery % of Unsecured Liabilities: P156,000/P240,000…………………………65%

Estimated payment to Partially Secured Creditors: P104,000 + 65%(P16,000)=P114,400


7. (B)
8. (C)
I. - c
MV of By-product Zest…………………………………………………………………………………………P 5
Less: Selling and administrative expense……………………………………… 2
Operating profit………………………………………………………………………………… 1
Share in Joint Cost per unit………………………………………………………………………P 2
x: Units produced…………………………………………………………………………………………………… 1,000
Share in joint cost………………………………………………………………………………………………P 2,000

II. - d
Hyp. MV Jt. Costs
Pep: 5,000 x (P50-P10) = P200,000 x 50% = P100,000
Vim: 4,000 x (P40-P 5) = 140,000
P340,000 P170,000*

Joint Costs……………………………………………………………………………………………………………………P172,000
Less: Joint costs allocated to By-product…………………………………… 2,000
Joint costs to joint products……………………………………………………………………P170,000

Sales of Pep: (P50 x 5,000)………………………………………………………………P 250,000


Less: Cost of Sales:
Joint costs…………………………………………………………………………P100,000
Further processing cost………………………………………… 50,000 150,000
Gross profit……………………………………………………………………………… P 100,000
9. (A)
I. - c
Current Assets before eliminating entries…………………………………P 320,000
Less: Unrealized profit in ending inventory
Upstream sales: P60,000 x P48,000/P240,000………__ 12,000
Consolidated Current Assets………………………………………………………………………P 308,000
Page 20 of 28 0915-2303213  resacpareview@gmail.com
ADVANCED FINANCIAL ACCOUNTING & REPORTING
ReSA Batch 44 – October 2022 CPALE Batch
25 September 2022  03:00 PM to 06:00 PM AFAR Final Pre-Board Exam
Exam
The relationship between Clark and Dean does not give rise to any consolidation
entries, since there is no parent-subsidiary relationship (Clark only owns 15% of
Dean)
II. - a
In a consolidated balance sheet, reciprocal balances, such as receivables and
payables, between a parent and a consolidated subsidiary should be eliminated
in its entire amount regardless of the portion of the subsidiary’s stock held
by the parent. Thus, the entire P70,000 advance should be eliminated in the
preparation of the year-end consolidated balance sheet.

10. (D)
12/01/20x8: Original forward rate (2 months)………………………………….P 1.01
12/31/20x8: Current (remaining) forward rate (1month)……… . _.99
Forex loss per unit.......………………………………………………………………………………. P .02
Multiplied by: Number of foreign currencies…………………………………….. 500,000
Foreign exchange loss – equity/OCI…………………………………………………………..P 10,000 (D)

11. (A)
January 1, 20x9 beginning balance of foreign exchange loss – equity
(No. 10)…………………………………………………………………………………………………………………………P 10,000
Settlement date of forward contract:
12/31/20x8: Current (remaining) forward rate (1 month)P .99
1/31/20x9: Spot rate……………………………………………………………………….. .98
Forex loss per unit………………………………………………………………………………. P .01
Multiplied by: Number of foreign currencies…………….…… 500,000
Foreign exchange loss – equity/OCI……………………….. 5,000
January 31, 20x9 balance – equity/OCI-loss debit balance… P 15,000 (A)

12. (A)
Job 102:
Direct materials……………………………………………………………………………. P 12,000
Direct labor………………………………………………………………………………….. 2,000
Overhead:
Machine Setup: P20,000/200 = P100 x 2……………P 200
Inspection: P130,000/6,500 = P20 x 10…………………. 200
Material Moves: P80,000/8,000 = P10 x 10…………. 100
Engineering: P50,000/1,000 = P50 x 50…………………… 2,500 3,000
Production/Manufacturing Costs………………………………….. P 17,000
Divided by: Units completed………………………………….. 50
Cost per unit under ABC…………………………………………………… P 340

13. (C) - 2,000 x (P3.00 P1.00) = P4,000

14. (A)
Joint Operations
before net income or loss 15,000 25,000 ending inventory
10,000 net income

15. (C) – Gift of long-lived assets should be reported as unrestricted support if the
organization has an accounting policy which does not apply a time restriction on
such gifts.

16. (A)
Combined equipment amounts P1,050,000
Less: gain on sale 25,000
Consolidated equipment balance P1,025,000

Combined Accumulated Depreciation P 250,000


Less: Depreciation on gain 5,000
Consolidated Accumulated Depreciation P 245,000

Page 21 of 28 0915-2303213  resacpareview@gmail.com


ADVANCED FINANCIAL ACCOUNTING & REPORTING
ReSA Batch 44 – October 2022 CPALE Batch
25 September 2022  03:00 PM to 06:00 PM AFAR Final Pre-Board Exam
17. (C) Exam
Evan Helen
Unadjusted capital P59,625 P33,500
Add (deduct): adjustments
Doubtful accounts (3% of A/R) (555) (405)
Understatement of depreciation _______ (900)
Adjusted capital P59,070 P32,195

18. (C) – Nt Dollar 175,000 / Nt Dollar 1.298 = P134,823


Goodwill is translated at the closing (current rate).
19. (B) – Nt 50,000 x P1 / Nt 1.56 = P32,051
PAS 21 par. 47 requires fair value adjustments to the carrying amounts of assets
and liabilities arising on the acquisition of a foreign operation to be treated as
assets and liabilities of the foreign operation. Therefore, they are translated at
the closing rate of exchange.
20. (C) – 160,000 yens x P1 / 2.40 yens = P66,667
PAS 21 par. 23 (a) requires the foreign currency monetary items, such as trade
payables, of an entity to be retranslated at the closing rate at the end of a
reporting period.
21. (A)
Materials
Product Unit Produced Ratio Materials Costs
R 25,000* 5/10 P 60,000
S 15,000 3/10 36,000
T 10,000 2/10 24,000
50,000 P120,000
*50,000 x 5/10
Conversion Costs
Product Unit Produced Final Total Ult. FPC HyMV/NRV % Conv. Costs
SP MV
R 25,000* P10 P250,000 P30,000 P220,000 40% P88,000
S 15,000 12 180,000 20,000 160,000 40%
T 10,000 15 150,000 30,000 120,000 40%
50,000 P500,000

Total Cost
Product Materials FPC Conv. Costs Total Units Unit Cost
Costs Costs Prod.
R P 60,000 P30,000 P88,000 P178,000 25,000 P7.12
S 36,000 20,000
T 24,000 30,000
P120,000

22. (D)
Raw and In Process Finished Goods Cost of Goods Sold

320,000 320,000 320,000 1,011,200


704,000 4,000
Actual Conversion Cost
1,024,000 1,011,200 1,015,200
708,000 704,000
* 12,800
4,000 4,000

Applied Conversion Cost Unit Cost : P1,024,000 / 16,000 = P64


* P64 x (16,000 – 15,800) = P12,800
704,000 704,000

23. (A) – under the reciprocal method, simultaneous equations are developed to determine
the total costs of each service department, taking into account the interactive
effect of other service departments providing service to other departments. For
maintenance, the cost is P18,700 plus 10% of the utilities cost, 40% of this will
be applied to A and 40% to B. The 20% that has already been assigned to the
utilities department is reflected in the set of simultaneous equations.
Page 22 of 28 0915-2303213  resacpareview@gmail.com
ADVANCED FINANCIAL ACCOUNTING & REPORTING
ReSA Batch 44 – October 2022 CPALE Batch
25 September 2022  03:00 PM to 06:00 PM AFAR Final Pre-Board Exam
24. (C)
Exam
Consolidated financial statements should not include intercompany payables and
receivables. Therefore, the P260,000 receivable from Vick Corp. will be eliminated
during the consolidation procedures. The P75,000 advances to Hill is not a
subsidiary of Cobb. Ward Corp. is an unconsolidated subsidiary, and so the P200,000
receivable from Ward will also be included in Cobb’s balance sheet.
25. (A)
In process, beginning…7,500
Started in Process………80,000
87,500
FIFO Actual WD EP Average Actual WD EP
IP, beg., F and T 7,500 0 0 F and T 72,500 100% 72,500
Started, F and T 65,000 100% 65,000 IP, end 13,000 100% 13,000
IP, end 13,000 100% 13,000 NL 1,100 100% 1,100
NL 1,100 100% 1,100 AL 900 100% 900
AL 900 100% 900 87,500 87,500
87,500 80,000

FIFO: P120,000 / 80,000 = P1.50


AVERAGE : (P120,000 + P10,400) / 87,500 = P1.49

26. (D)
Merchandise:
From HO Available for Sale, at cost
(P300,000 +P120,000 – P7,500)/1.2………………………………………………P 343,750
Purchases from Outsiders, at cost…………………………………………………………… 72,500
Merchandise available for sale at cost (net of returns)… P 416,250
The requirement of the problem is vague, but since there is no other answer
available, cost of goods available for sale is assumed to be at cost.

27. (B)
Actual
IP, beginning 7,500
Started in Process 80,000
87,500
FIFO
Actual WD EUP- M WD EUP-CC
IP, beginning 7,500 0 0 60% 4,500
Started, Fin. and Transf 65,000 100% 65,000 100% 65,000
IP, ending 13,000 100% 13,000 70% 9,100
NL 1,100 100% 1,100 100% 1,100
AL ___900 100% ___900 100% ___900
87,500 80,000 80,600
Cost per EUP P120,000 P350,000
80,000 80,600
P1.50 P4.34
Cost of IP, ending:
CPD:………………………………………………………………………………………………………………………………………………………………P -0-
CTD: (Current)
Materials: 13,000 x P1.50……………………………………………………………………………………… 19,500
Conversion cost: 9,100 x P4.34………………………………………………………………………… 39,494
P 58,994
28. (D)
Actual
IP, beginning 7,500
Started in Process 80,000
87,500
FIFO
Actual WD EUP- M WD EUP-CC
Finished and Transferred 72,500 100% 72,500 100% 72,500
IP, ending 13,000 100% 13,000 70% 9,100
NL 1,100 100% 1,100 100% 1,100
AL ___900 100% ___900 100% ___900
87,500 87,500 83,600
Cost per EUP (P10,400 + P120,000) (P13,800+P350,000)
87,500 83,600
= P1.49 = P4.35
Page 23 of 28 0915-2303213  resacpareview@gmail.com
ADVANCED FINANCIAL ACCOUNTING & REPORTING
ReSA Batch 44 – October 2022 CPALE Batch
25 September 2022  03:00 PM to 06:00 PM AFAR Final Pre-Board Exam
Cost of NL Units: (1,100 x P0) + (1,100 x P1.49) + (1,100 x P4.35) Exam
= P6,424 assigned to units transferred out
Note: No NL lost units allocated to EI since it did not pass through the inspection
point (which happens at the end of the production)

29. (B)
Consolidated Cost of Sales:
Cost of Sales before consolidation:
Parcon……………………………………………………………………………………………………………………………………..P 800,000
Shelly………………………………………………………………………………………………………………………………………. 640,000
Combined Cost of Sales…………………………………………………………………………………………………..P1,440,000
Less: Intercompany Cost of Sales (or Purch) to be eliminated… 400,000
Eliminating entry for 100% RPBI of P** (EI of 20x7)……………. 20,000
Add: Eliminating entry for 100% UPEI of P*** (EI of 20x8)…………. 25,000
Consolidated Cost of Sales………………………………………………………………………………………..P 1,045,000

Further, the additional eliminating entries are as follows: (Cost Method)


**100% RPBI of P:
Retained Earnings – P, beginning (Cost Model)/
Investment in S Company(Equity Method)…………………………………16,000
Retained Earnings – S, beginning………………………………………… 4,000
Cost of Sales (Beginning Inventory in Income Statement)… 20,000

***100% UPEI of P:
Cost of Sales (Ending Inventory in Income Statement) 25,000
Inventory (Ending Inventory in Balance Sheet)…………………… 25,000

30. (D) – [(P1,110,000 – P780,000) + P960,000] – P210,000 = P1,080,000

31. (A)
Gross patient service revenue………………………………………………………………………..P 980,000
Less: Contractual adjustments………………………………………………………………………. 100,000
Allowance for discounts to hospital employee……………. 15,000
Net Patient Service Revenue…………………………………………………………………………..P 865,000

Bad debts expense is classified as expenses. (refer to AFAR-19 page 9)

32. (D)
Materials: P5,000 + P300……………………………………………………………………………………P 5,300
Direct labor……………………………………………………………………………………………………………….. 14,000
Applied factory overhead (150% x P14,000)…………………………………… 21,000
P 40,300
Since, the allowance for rework was included in the manufacturing
overhead budget, therefore, the rework cost should be charged to
factory overhead control.

33. (C)
Raw and In Process Finished Goods Cost of Goods Sold
13,000 20,000
500,000 493,000 493,000 498,000 498,000

20,000 15,000

34. (B)
Incidentally, the entry to record the transactions related to by-product would be
as follows:
Share in Joint Costs (corresponding reduction in Coco, main product as stated in
the problem)

Work in Process – Soloc (by-product)……………… 75,000


Work in Process – Coco (main product)… 75,000
MV of by-product (P3 x 45,000)… P 135,000
Less: FPC of by-product
(P30,000 + P22,500 + P7,500)… 60,000
Net Revenue – reduction in Coco’s costs… P 75,000

Page 24 of 28 0915-2303213  resacpareview@gmail.com


ADVANCED FINANCIAL ACCOUNTING & REPORTING
ReSA Batch 44 – October 2022 CPALE Batch
25 September 2022  03:00 PM to 06:00 PM AFAR Final Pre-Board Exam
Further Processing Costs: Exam
Work in Process – Soloc (by-product)……………………………… 60,000
Raw materials……………………………………………………………………… 30,000
Direct labor or Payroll…………………………………………… 22,500
Factory Overhead – applied…………………………………… 7,500

Transferred to Warehouse/Stockroom:
By-product Inventory – Soloc………………………………………………… 135,000
Work in Process – Soloc………………………………………… 135,000

Or, alternatively the following compound entry would be made:


By-product Inventory – Soloc……………………………………………………………………………… 135,000
Raw Materials…………………………………………………………………………………………………………… 30,000
Direct labor or Payroll………………………………………………………………………………… 22,500
Factory Overhead – applied………………………………………………………………………… 7,500
Work in Process – Coco…………………………………………………………………………………… 7,500

35. (B)
I. - b
Consideration transferred (fair value)……………………………………… P800,000
Less: Fair value of net identifiable assets acquired:
Cash……………………………………………………………………………………………………………… P150,000
Accounts receivable……………………………………………………………………… 140,000
Software…………………………………………………………………………………………………… 320,000
In-process R&D…………………………………………………………………………………… 200,000
Liabilities…………………………………………………………………………………………… (130,000) 680,000
Goodwill……………………………………………………………………………………………………………………… P120,000

The application of recognition measurements in business combination means that


an acquirer must, in recognizing separately the acquirer’s intangible assets,
recognize intangible assets that the acquiree has not recognized in its records,
such as in-process research and development that cannot be recognize under PAS
38 as internally generated assets. As noted in par 34, recognition by an acquirer
of an acquiree’s in-process research and development project only depends whether
the project meets the definition of an intangible asset. It can be seen that
intangible assets in a business combination will be able, and in fact are
required, to recognize intangible assets that are not separately recognizable
when acquired by other means. The costs on individual assets acquired are measured
but reference to the fair value of those assets.

Therefore, In-Process Research and Development is capitalized as an asset of the


combination and reported as intangible assets with indefinite lives subject to
impairment reviews.

II. - c
Atkins records new shares at fair value
Value of shares issued (51,000 × P3) P153,000
Par value of shares issued (51,000 × P1) 51,000
Additional paid-in capital (new shares) P102,000
Additional paid-in capital (existing shares) 90,000
Consolidated additional paid-in capital P192,000

At the date of acquisition, the parent makes no change to retained earnings.

36. (D)
Original costs charged to Work-in-Process P 53,200
Add: Rework Costs
Direct Materials P 2,000
Direct Labor 3,200
Applied Overhead (150% of P3,200) 4,800 10,000
Total Costs of Job No. 369 P 63,200
Divided by: Good Units _____200
P 316
37. (C)
I. - c
The P5,000,000 are considered temporary restricted since it has a purpose which
have not yet been fulfilled.

Page 25 of 28 0915-2303213  resacpareview@gmail.com


ADVANCED FINANCIAL ACCOUNTING & REPORTING
ReSA Batch 44 – October 2022 CPALE Batch
25 September 2022  03:00 PM to 06:00 PM AFAR Final Pre-Board Exam
The P2,000,000 principal which had Exam
to be retained (meaning to be held in
perpetuity – permanent) is classified as permanently restricted, while the
dividends is classified as temporary restricted because of purpose restriction,
but to no avail, amount is not given.

II. - d
These services do not meet the criteria for donated services that are recognized.
38.(A)
True Branch Net Income ………………………………………………………………… P156,000
Less: Branch Net Income as reported (by the Branch) ………………… 60,000
Overvaluation of Cost of goods sold ……………………………………………… P 96,000
Less: Cost of goods sold from home office at billed price:
Inventory, December 1 ……………………………………… P 70,000
Shipments from home office ………………………………. 350,000
Cost of goods from home office available for sale P420,000
Less: Inventory, December 31 ……………………………… 84,000 336,000
Cost of goods sold from home office, at cost ………………………………… P240,000

Billed Price: (P336,000/P240,000) ……………………………………… 140%

39.(B)
Allowance for overvaluation after adjustment:
P84,000 x 40/140 ………………………………………………………… P 24,000

40.(C)- use T-accounts for your comfortability,


Home Office Books Branch Books
(Branch Current- (Home Office Current
Dr. balance) – Cr. balance)
Unadjusted balance P150,000 P117,420
Add (deduct) adjustments:
In transit 37,500
HO A/R collected by br. 10,500
Supplies returned ( 4,500)
Error in recording Br. NI ( 1,080)
Cash sent to branch
to General Expense by HO 25,000 25,000
Adjusted balance P 179,920 P 179,920

41. (D)
Charges Related to
Total Consignment Inventory on
Charges Sales Consignment
(5) (4) (1)
Consignor’s charges:
Cost P 775 P 620 P 155
Freight 50 40 10
Consignee’s charges:
Commission 200 200 ____
Total P1,025 P 860 P165
Sales price (4 units x _ 1,000
P250/unit)
Profit on Consignment P 140

42. (D) - refer to PAS 27 par. 20-21.


Carly’s trade receivables……………………………………………………………….P1,040,000
Halley’s trade receivables…………………………………………………………… 215,000
Total………………………………………………………………………………………………………………… P1,255,000
Less: Intercompany receivable (due from Halley)… 30,000
Intercompany receivable (due from Carly)…… 40,000
Consolidated trade receivables………………………………………… P1,185,000
43. (A) – refer to PAS 27 par. 26.
This is the parent company's share of the post-acquisition retained earnings of
the subsidiary. This is determined by deducting (i) the parent company's share of
the retained earnings of the subsidiary at the date of acquisition from (ii) the
parent company's share of the retained earnings of the subsidiary at the end of
the current reporting period.
Page 26 of 28 0915-2303213  resacpareview@gmail.com
ADVANCED FINANCIAL ACCOUNTING & REPORTING
ReSA Batch 44 – October 2022 CPALE Batch
25 September 2022  03:00 PM to 06:00 PM AFAR Final Pre-Board Exam
Exam
Pulley’s retained earnings, date of acquisition………………… P 500,000
Less: Pulley’s retained earnings, end of the current 1,400,000
P 900,000
X: Controlling interest %...................... _ 80%
Pulley’s RE included in the consolidated SFP P 720,000
44. (B)
Upstream sales:
Selling price of non-current asset…………………………………………………….P 200,000
Less: Book/carrying value, date of sale……………………………………… 160,000
Gain on intercompany sale …………………………………………………………………… P 40,000
Incidentally, the eliminating entry (assuming books are already closed) would
be as follows:
Retained earnings – Parent (65% x P40,000)…………………… ………………26,000
Non-controlling interest (35% x P40,000)………………………………………… 14,000
Non-current asset………………………………………. 40,000

PAS 27 pars. 20-21 require the profit on intragroup assets to be eliminated in


full. Only the group share of the profits of the subsidiary are taken to group
retained earnings.
This is because the subsidiary sold the asset to the parent. This gain is not
realized from a group perspective per PAS 27 par. 21 and must be removed in full.
It is then allocated between the shareholders of the subsidiary, in the form of
retained earnings (group share of the gain) and the non-controlling interest.
45. (B) 20x5: P12,000,000 > P11,870,000, No loss;
20x6: P12,000,000 – P12,400,000 = P400,000 loss.

46. (A)- revenue recognized to the extent of costs incurred

47. (C)
Date of purchase: 3 million baht / 2 baht per peso…………………P 1.5 million
Balance sheet date: 3 million baht / 1.5 baht per peso……… 2.0 million
Exchange loss……………………………………………………………………………………………………………………P 0.5 million

48. (C)
Consideration Transferred…………………………………………………………………………………… 9.0 million
Less: Fair value of net assets acquired……………………………………………… 6.0 million
Goodwill………………………………………………………………………………………………………………………………… 3.0 million
Divided by: CURRENT RATE on the balance sheet for
purposes of translation on the date of
acquisition.……………………………………………………………………………………… 2.0 baht per peso
Goodwill in the Consolidated Balance Sheet (date of
Acquisition)…………………………………………………………………………………………………………P 1.5 million

Examinees may be misled that since the functional currency is peso, the temporal
method (applied only in case of subsequent to date of acquisition) should then be
applied wherein goodwill or any fair value adjustments is considered as a non-
monetary asset carried at historical cost be remeasured (or translated) using
historical rate (which in this problem is 1.5 baht = P1). But the problem do not
fall under this category – the temporal method, instead it is a classical example
of a goodwill and fair value adjustments arising from acquisition of subsidiaries.

Goodwill arising from the Acquisition of Subsidiaries (Date of Acquisition)


When a company acquires a controlling interest in another company, the excess of
the purchase price over the acquirer’s interest in the fair value of the
identifiable net assets of the acquired company is recognized as goodwill on
consolidation. In the context of a foreign company, the issue arises as to whether
goodwill is an asset of the acquired company or an asset in the acquirer’s books.
If it is an asset of the acquired subsidiary, the goodwill is a foreign asset
which should be translated in the same manner as any other asset of the acquired
subsidiary, which may give rise to a translation difference. However, if it is
treated as an asset in the acquirer’s books, there is no need for translation.

Page 27 of 28 0915-2303213  resacpareview@gmail.com


ADVANCED FINANCIAL ACCOUNTING & REPORTING
ReSA Batch 44 – October 2022 CPALE Batch
25 September 2022  03:00 PM to 06:00 PM AFAR Final Pre-Board Exam
Pas 21 par. 47 states that: Exam
“Any goodwill arising on the acquisition of a foreign operation and any
fair value adjustments to the carrying amount of assets and liabilities
arising on the acquisition of that foreign operation shall be treated as
assets and liabilities of the foreign operations. Thus they shall be
expressed in the functional currency of the foreign operation and shall
be translated at the closing rate…”
Subsequent to date of acquisition, accordingly goodwill has to be measured in the
functional currency of the foreign operation. If the functional currency of the
foreign operation is the local currency, the goodwill on acquisition is to be
translated at the closing rate. On the other hand, if the functional currency of
the foreign operation is the parent’s currency (or the presentation currency),
goodwill on acquisition is treated as a non-monetary asset and remeasured at the
exchange rate of the acquisition of the foreign operation.

49. (A)
Allocated Excess arising from consolidation…………………………… 1.2 million baht
Divided by: CURRENT RATE on the balance sheet for
purposes of translation the date of
acquisition….…………………………………………………………………………… 2.0 baht per peso
Allocated Excess (over/under valuation)………………………………………P600,000

Refer to No. 48 for further discussion of using closing/current rate on the


acquisition of a foreign operation resulting in fair value adjustments. Again,
the same with No. 48, the functional currency of peso is somewhat misleading; it
does not refer to the use of temporal method on the date of acquisition.

50. (C) – 20 million x 240.4/60.4 = 80 million


51. (B)
52. (B)
53. (C)
Dividends declared in 2021 (P100,000 + P150,000)…………………………………P 250,000
X: ownership percentage…………………………………………………………………………………………………… 30%
Dividend income…………………………………………………………………………………………………………………………P 75,000

Page 28 of 28 0915-2303213  resacpareview@gmail.com


ReSA - THE REVIEW SCHOOL OF ACCOUNTANCY
CPA Review Batch 44  Oct 2022 CPALE  24 September 2022  11:45 AM - 02:45 PM

AUDITING FINAL PRE-BOARD EXAMINATION

INSTRUCTIONS: Select the correct answer for each of the questions. Mark only one
answer for each item by shading the box corresponding to the letter of your choice on
the answer sheet provided. STRICTLY NO ERASURES ALLOWED. Use pencil no. 2 only.

1. A number of factors influence the sample size for a substantive test of details of
account balance. All other factors being equal, which of the following would lead
to a larger sample size?
a. Greater reliance on internal control.
b. Greater reliance on analytical procedures.
c. Smaller expected frequency of errors.
d. Smaller measure of tolerable misstatement

2. Eli CPA, has been asked to audit and report on the statement of financial position
of Jane Co., but not on the statement of comprehensive income, changes on equity,
or cash flows. This audit will not be performed in conjunction with audit of the
complete set of financial statements. Under these circumstances, Eli may:
a. Not accept the engagement because it would constitute a violation of
the profession’s ethical standards.
b. Not accept the engagement because it would be tantamount to rendering
a piecemeal opinion.
c. Accept the engagement because such engagements merely involve special
considerations in the application of PSA.
d. Accept the engagement but should disclaim an opinion because the
complete set of financial statements was not audited.

3. Which of the following circumstances most likely would cause an auditor to suspect
that there are material misstatements in an entity’s financial statements?
a. The entity’s management strictly enforces its integrity and ethical
value.
b. Monthly bank reconciliation ordinarily includes several outstanding
checks.
c. Management outsources the internal audit function to another CPA
firm.
d. The auditor identifies an inappropriate valuation method that is
widely applied by the entity.

4. Which of the following auditing procedures most likely would assist an auditor in
identifying related party transactions?
a. Inspecting correspondence with lawyers for evidence of unreported
contingent liabilities.
b. Vouching accounting records for recurring transactions recorded just
after the balance sheet date.
c. Reviewing confirmations of loans receivable and payable for
indications of guarantees.
d. Performing analytical procedures for indications of possible
financial difficulties.

5. In assessing control risk for purchases, an auditor vouches a sample of entries in


the voucher register to the supporting documents. Which assertion would this test
of controls most likely support?
a. Completeness
b. Occurrence
c. Allocation and valuation
d. Rights and obligations

6. An auditor wishes to test the completeness assertion for sales. Which of the
following audit tests would most likely accomplish this objective?
a. Select a sample of shipments occurring during the year and trace each
one to inclusion in the sales journal.
b. Compare accounts receivable turnover (net credit sales/average gross
receivables) in the current year to that achieved in the prior year.
c. Use common size analysis to compare recorded sales to sales recorded
by other companies in the same industry.
d. Select large individual sales recorded during the year and review
supporting documentation.

Page 1 of 21 0915-2303213  resacpareview@gmail.com


AUDITING
ReSA Batch 44 – October 2022 CPALE Batch
24 September 2022  11:45 AM to 02:45 PM AUD Final Pre-Board Exam
7. Which of the following is not true regarding fraud risk factors?
a. They include incentives/pressures, opportunity, and rationalization.
b. Lack of observation of the three fraud risk factors implies that
there is no fraud risk.
c. The existence of all three fraud risk factors is not an absolute
indication that fraud has occurred.
d. Fraud risk factors are often present in circumstances where fraud has
occurred.
8. Under which of the following circumstances would a disclaimer of opinion not be
appropriate?
a. The auditor is unable to determine the amounts associated with an
employee fraud scheme.
b. Management does not provide reasonable justification for a change in
accounting principle.
c. The client refuses to permit the auditor to confirm certain accounts
receivable or apply alternative procedures to verify their balances.
d. The chief executive officer is unwilling to sign the management
representation letter.

9. Which of the following is not an example of the application of professional


skepticism?
a. Design additional auditing procedures to obtain more reliable
evidence in support of a particular financial statement assertion.
b. Obtaining corroboration of management’s explanations through
consultation with a specialist.
c. Inquiring or prior year engagement personnel regarding their
assessment of management’s honesty and integrity.
d. Using third-party confirmations to provide support for management’s
representations.

10. An auditor suspects that certain client employees are ordering merchandise for
themselves over the internet without recording the purchase or receipt of the
merchandise. When vendors’ invoices arrive, one of the employees approves the
invoices for payment. After the invoices are paid, the employee destroys the
invoices and the related vouchers. In gathering evidence regarding the fraud, the
auditor most likely would select items for testing from the file of all:
a. Cash disbursements
b. Approved vouchers
c. Receiving reports
d. Vendors’ invoices

11. An auditor is confirming accounts receivable using positive confirmation. The


auditor decides to leave the accounts receivable amount blank rather than stating
the amount owed. The auditor should be aware that the blank form may be less
efficient because:
a. Subsequent cash receipts need to be verified.
b. Statistical sampling may not be used.
c. A higher assessed level of detection risk is required.
d. More nonresponses are likely to occur.

12. In statistical sampling methods used in substantive testing, an auditor most likely
would stratify a population into meaningful groups if:
a. Probability-proportional-to-size (PPS) sampling is used.
b. The population has highly variable recorded amounts.
c. The auditor’s estimated tolerable misstatement is extremely small.
d. The standard deviation of recorded amounts is relatively small.

13. In determining whether transactions have been recorded, the direction of the audit
testing should be from the:
a. General ledger balances.
b. Adjusted trial balances.
c. Original source documents.
d. General journal entries.

Page 2 of 21 0915-2303213  resacpareview@gmail.com


AUDITING
ReSA Batch 44 – October 2022 CPALE Batch
24 September 2022  11:45 AM to 02:45 PM AUD Final Pre-Board Exam
14. Before accepting an engagement to report on summary of financial statements, the
auditor shall perform the following, except:
a. Determine whether the applied criteria are acceptable.
b. Obtain the agreement from management that it acknowledges and
understand its responsibility
c. Evaluate whether the summary financial statements adequately disclose
their summarized nature and identify the audited financial
statements.
d. Agree with management the form of opinion to be expressed on the
summary financial statements.

15. IT specific controls include IT general controls and IT application controls. Which
of the following is an IT application controls?
a. Controls over interfaces, integrations, and e-commerce.
b. Controls over data center and network operations
c. System software acquisition, change and maintenance controls.
d. Access security controls
16. When financial statements contain a departure from PFRS because, due to unusual
circumstances, the statements would otherwise be misleading, the auditor should
express an opinion that is:
a. Unmodified.
b. Qualified.
c. Adverse.
d. Qualified or adverse, depending on pervasiveness.
17. Which of the following types of audit evidence is the most persuasive?
a. Prenumbered client purchase order forms.
b. Client work sheets supporting cost allocations.
c. Bank statements obtained from the client.
d. Client representation letter.
18. An auditor’s letter issued on significant deficiencies relating to internal control
observed during a financial statement audit should:
a. Include a brief description of the tests of controls performed in
searching for significant deficiencies and material weaknesses.
b. Indicate that the significant deficiencies should be disclosed in the
annual report to the entity’s shareholders.
c. Include a paragraph describing management’s assessment concerning the
effectiveness of internal control.
d. Indicate that the audit’s purpose was to report on the financial
statements and not to provide an opinion on internal control.
19. During the audit of a new client, the auditor determined that management had given
illegal bribes to City officials during the year under audit and for several prior
years. The auditor notified the client’s board of this act of noncompliance with
laws and regulations, but the board decided to take no action because the amounts
involved were immaterial to the financial statements. Under these circumstances,
the auditor should:
a. Add an explanatory paragraph emphasizing that certain matters, while
not affecting the unmodified opinion, require disclosure.
b. Report the illegal bribes to the City official at least one level
above those persons who received the bribes.
c. Consider withdrawing from the audit engagement and disassociating
from future relationships with the client.
d. Issue and “except for” qualified opinion or an adverse opinion with
a separate paragraph that explains the circumstances.
20. A CPA’s report on agreed-upon procedures relating to management’s assertion about
an entity’s compliance with specified requirements should contain:
a. A statement of limitations on the use of the report.
b. An opinion about whether management’s assertions is fairly stated.
c. Negative assurance that control risk has not been assessed.
d. An acknowledgement of responsibility for the sufficiency of the
procedures.

Page 3 of 21 0915-2303213  resacpareview@gmail.com


AUDITING
ReSA Batch 44 – October 2022 CPALE Batch
24 September 2022  11:45 AM to 02:45 PM AUD Final Pre-Board Exam
21. Which of the following statements is not correct about materiality?
a. The concept of materiality recognizes that some matters are important
for fair presentation of financial statements in conformity with
PFRS, while other mattes are not important.
b. An auditor considers materiality for the financial statements as a
whole in terms of the largest aggregate level of misstatements that
could be material to any one of the financial statements.
c. Materiality judgments are made in light of surrounding circumstances
and necessarily involve both quantitative and qualitative judgments.
d. An auditor’s consideration of materiality is influenced by the
auditor’s perception of the needs of a reasonable person who will
rely on the financial statements.
22. After performing risk assessment procedures, an auditor decided not to perform
tests of controls. The auditor most likely decided that:
a. The available evidence obtained through tests of controls would not
support an increased level of control risk.
b. A reduction in the assessed level of control risk is justified for
certain financial statement assertions.
c. It would be inefficient to perform tests of controls that would result
in a reduction in planned substantive tests.
d. The assessed level of inherent risk exceeded the assessed level of
control risk.
23. To gain assurance that all inventory items in a client’s inventory listing schedule
are valid, an auditor most likely would trace:
a. Inventory tags noted during the auditor’s observation to items listed
in the inventory-listing schedule.
b. Inventory tags noted during the auditor’s observation to items listed
in receiving report and vendors’ invoices.
c. Items listed in the inventory-listing schedule to inventory tags and
auditor’s recorded count sheets.
d. Items listed in receiving reports and vendor’s invoices to the
inventory-listing schedule.
24. Which of the following would an auditor least likely consider with respect to
fair values?
a. Segregation of duties between those committing the entity to certain
transactions and those responsible for undertaking the valuations
related to those transactions.
b. The effect on fair value measurement and disclosures of information
available subsequent to the audit.
c. The role of information technology in determining fair value
measurements and disclosures.
d. Whether the valuation methods used are appropriate in relation to the
industry in which the entity operates.
25. An auditor finds several errors in the financial statements that the client prefers
not to correct. The auditor determines that the errors are not material in the
aggregate. Which of the following actions by the auditor is most appropriate?
a. Document the errors in the summary of uncorrected errors and document
the conclusion that the errors do not cause the financial statements
to be misstated.
b. Document the conclusion that the errors do not cause the financial
statements to be misstated, but do not summarize uncorrected errors
in the working papers.
c. Summarize the uncorrected errors in the working papers, but do not
document whether the errors cause the financial statements to be
misstated.
d. Do not summarize the uncorrected errors in the working papers, and
do not document a conclusion about whether the uncorrected errors
cause the financial statements to be misstated.
26. Which of the following matters would an auditor most likely include in a management
representation letter?
a. Communications with those charged with governance concerning weakness
in internal control.
b. The reasonableness of significant assumptions used in making
accounting estimates.
c. Plans to acquire or merge with other entities in the subsequent year.
d. Management’s acknowledgement of its responsibilities for the
detection of employee fraud.
Page 4 of 21 0915-2303213  resacpareview@gmail.com
AUDITING
ReSA Batch 44 – October 2022 CPALE Batch
24 September 2022  11:45 AM to 02:45 PM AUD Final Pre-Board Exam
27. CJ, CPA, was engaged by a group of royalty recipients to apply agreed-upon
procedures to financial data supplied by Flong Co. regarding Flong’s written
assertion about its compliance with contractual requirements to pay royalties.
CJ’s report on these agreed-upon procedures should contain a(an):
a. Disclaimer of opinion about the fair presentation of Flong’s
financial statements.
b. List of the procedures performed (or reference thereto) and CJ’s
findings.
c. Opinion about the effectiveness of Flong’s internal control
activities concerning royalty payments.
d. Acknowledgement that the sufficiency of the procedures is solely CJ’s
responsibility.

28. Financial statements compiled by an accountant should be accompanied by a report


stating that:
a. The scope of the accountant’s procedures has not been restricted in
testing the financial information that is the representation of
management.
b. The accountant evaluated the appropriateness of the accounting
principle used and the reasonableness of significant accounting
estimates made by management.
c. The accountant has not audited or reviewed the financial statements.
d. A compilation primarily includes applying analytical procedures to
management’s financial data and making inquiries of company
management.

29. Which of the following actions by a CPA most likely violates the profession’s
ethical standards?
a. Using a records-retention agency to store confidential client
records.
b. Retaining client records after the client has demanded their return.
c. Arranging with a financial institution to collect notes issued by a
client in payment of fees dues.
d. Compiling the financial statements of a client that employed the
CPA’s spouse as a bookkeeper.

30. Which of the following auditor concerns most likely would be so serious that the
auditor would conclude that a financial statement audit cannot be performed.
a. The CPA lacks experience in the client’s operations and industry.
b. A portion of supporting evidence stored at an offsite storage facility
was destroyed by a hurricane.
c. Management has imposed a restriction that the auditor believes will
result in a qualified opinion.
d. There is a substantial risk of management intentionally manipulating
accounting records.

31. An auditor’s engagement letter most likely would include a statement regarding:
a. The advantages of statistical sampling.
b. The inherent limitations of an audit.
c. Billings to be paid in the form of stock of the entity.
d. The assessment of risk of material misstatement.

32. As of August 31, A CPA had obtained sufficient appropriate audit evidence with
respect to fieldwork on an engagement to audit financial statements for the year
ended June 30. On August 27, an event came to the CPA’s attention that should be
disclosed in the notes to the financial statements. The event was properly
disclosed by the entity, but the CPA decided not to dual date the auditor’s report
and dated the report August 27. Under these circumstances, the CPA was taking
responsibility for:
a. All subsequent events that occurred through August 27.
b. Only the specific subsequent event disclosed by the entity.
c. All subsequent events that occurred through August 13 and the specific
subsequent event disclosed by the entity.
d. Only the subsequent events that occurred through August 13.

Page 5 of 21 0915-2303213  resacpareview@gmail.com


AUDITING
ReSA Batch 44 – October 2022 CPALE Batch
24 September 2022  11:45 AM to 02:45 PM AUD Final Pre-Board Exam
33. In a probability-proportional-to-size sample with a sampling interval of
P1,000,000, an auditor discovered that a selected account receivable with a recorded
amount of P500,000 had an audited amount of P400,000. If this were the only
misstatement discovered by the auditor, the projected misstatement of this sample
would be:
a. P100,000
b. P200,000
c. P500,000
d. P1,000,000

34. Which of the following procedures would an accountant least likely perform during
an engagement to review the financial statements of a client?
a. Observing the safeguards over access to and use of assets and records.
b. Comparing the financial statements with anticipated results in
budgets and forecast.
c. Inquiring of management about actions taken at the board of directors’
meetings.
d. Studying the relationships of financial statement elements expected
to conform to predictable patterns.

35. Comparative financial statements include the prior year’s statements that were
audited by a predecessor auditor whose report is not presented. If the
predecessor’s report was unmodified, the successor should:
a. Add an emphasis-of-matter paragraph that expresses only limited
assurance concerning the fair presentation of the prior year’s
financial statements.
b. Express an opinion only on the current year’s financial statements
and make no reference to the prior year’s financial statements.
c. Indicate in an other-matter paragraph that the predecessor auditor
expressed an unmodified opinion on the prior year’s financial
statements.
d. Obtain a letter of representation from the predecessor auditor
concerning any matters that might affect the successor’s opinion.

36. Which of the following is generally appropriate in relation to the timing of the
substantive test procedures in auditing the cash account of a client?
a. Schedule the cash count in advance of the balance sheet date in order
to discover any kiting operations at year-end.
b. Correlate the count of cash with the cut-off of accounts payable.
c. Correlate the count of cash with the count of marketable securities
and other negotiable assets.
d. Schedule the cash count immediately upon the return of the
confirmation letters from the banks.

37. Which of the following cash transfers indicates kiting which results in an
overstatement of cash balance per books at December 31, 2021?

Bank Transfer Schedule


Disbursement Receipt
Recorded in Paid by Recorded in Received
books Bank A books by bank B
a. 1/2/22 1/2/22 12/31/21 12/31/21
b. 12/31/21 1/2/22 12/31/21 12/31/21
c. 12/31/21 1/5/22 1/3/22 1/4/22
d. 1/4/22 1/11/22 1/4/22 1/4/22

PROBLEM 1:
The following information were made available to you, in line with your audit of the
cash accounts of your client, Snow Corp. for the period ended December 31, 2021:

Below is the bank reconciliation statement for the month of November as furnished by
the client’s accountant:

Page 6 of 21 0915-2303213  resacpareview@gmail.com


AUDITING
ReSA Batch 44 – October 2022 CPALE Batch
24 September 2022  11:45 AM to 02:45 PM AUD Final Pre-Board Exam
Cash per bank statement, November 30 125,000
Add: Deposit-in-transit 61,200
Erroneous November bank charge 5,500
Unrecorded November bank service charge 1,500
Unrecorded November NSF check 54,000
Total P247,200
Less: Outstanding checks (95,600)
Unrecorded bank credit, Note collection (100,000)
Unrecorded bank credit, Note interest collection (10,000)
Cash per General Ledger P41,600

• All November reconciling items have cleared in December in both the bank records
and in the general ledger.
• The bank statement for December showed total debits amounting to P1,980,000 and
total credits amounting to P2,097,000.
• December books showed total debits at P2,120,000 and total credits at P2,005,000.
• Bank loan proceeds for the month of December amounting to P120,000 and December
bank service charge amounting to P2,100 did not appear in the client’s December
cash records yet.
• An NSF check amounting to P28,000 was returned by the bank to the client in
December. The check was redeposited also in December. The records revealed that
the client did not record both the receipt and redeposit of the NSF check.
• A P35,000 disbursement was recorded in the books at P53,000 in December.
Investigation revealed that it was detected and was corrected in January of the
following year.
• Another P25,000 disbursement was recorded in the books at P52,000, investigation
revealed that this error was detected and was already corrected by the end of
December

Required:
38. What is the correct deposit-in-transit as of December 31?
a. 127,700 c. 100,700
b. 72,700 d. 95,200

39. What is the correct outstanding check as of December 31?


a. 50,200 c. 86,200
b. 77,200 d. 44,700

40. What is the adjusted balance of cash as of December 31?


a. 287,000 c. 294,600
b. 292,500 d. 256,500

41. In line with your audit of a manufacturing client’s financial statements, you were
assigned to audit the trade receivables. In preparing your audit program, which of
the following control objectives would be the least concern?
a. Ensuring that all shipments made have been billed
b. No shipment has been billed more than once
c. Each shipment has been billed for the proper amount
d. All billings corresponds to actual shipments of goods

42. Which of the following alternative audit procedures is necessary in instances where
replies on positive confirmation requests are not received even after sending a
second set of confirmation requests?
a. Examining subsequent receipts of year-end accounts receivable.
b. Reviewing accounts receivable aging schedule prepared at the balance
sheet date and at a subsequent date.
c. Requesting that management increases the allowance for uncollectible
accounts by an amount equal to a certain percentage of the balances
in those accounts that cannot be confirmed
d. Performing an overall analytical review of accounts receivable and
sales on a year-to-year basis.

PROBLEM 2:
In line with your audit of Sand Corp.’s trade receivables for the period ended December
31, 2021, the client furnished you with the following SL and GL reconciliation:

Page 7 of 21 0915-2303213  resacpareview@gmail.com


AUDITING
ReSA Batch 44 – October 2022 CPALE Batch
24 September 2022  11:45 AM to 02:45 PM AUD Final Pre-Board Exam
Balance per Subsidiary Ledger P2,590,000
Sales invoice dated December 29 for goods delivered on
the same date but were still in transit as of December
31. Goods were shipped FOB Shipping point, freight (27,000)
prepaid. The amount included the freight cost of P2,000.
Sales invoice dated December 30 for goods delivered on (30,000)
January 2, 2022.
Sales invoice dated December 30 for goods delivered on
December 30 on sale with a repurchase agreement. The
right to resell rests with the customer and the customer 22,000
has a significant economic incentive to resell.
Sales invoice dated December 31 for goods delivered on
January 2 under a Bill and Hold agreement with the
customer, which was executed in December. 18,000
Credit balance in one of the customer accounts resulting
from overcollection (6,000)
Credit balance in one of the customer accounts resulting
from a collection of a previously written off account.
This amount was deducted from the “Current” accounts in 5,000
preparing the aging schedule (see information below)
Customer check dated January 2, 2022. The collection was
for a customer invoice dated October 20, 2021 20,000
Customer collection check dated December 1, 202, but was
returned by the bank with the December bank statement
because of insufficiency of fund. The check was for the
payment of a customer invoice dated August 5, 2021 41,000
Customer collection check dated December 30, 2021 in
payment of an October 25 invoice (28,000)
Subscriptions receivable on ordinary shares, due March 90,000
31, 2023
Balance per General Ledger P2,695,000

Additional information:
An aging schedule and the company’s policy of providing bad debt allowance is shown
below:
Age Amount % uncollectible
Current 1,040,000 -
1-60 days past due 1,200,000 5%
More than 60 days past due 350,000 20%

The company’s sales term is 5/30, n/60. It was determined that based on past experience,
40% of accounts which are considered current will probably pay within the discount
period and that from the accounts that is more than 60 days past due, P120,000 is
definitely uncollectible and therefore must be written off.

Required:
43. What is the correct carrying value of the accounts receivable, trade as of
December 31, 2021?
a. 2,351,740 c. 2,382,200
b. 2,389,540 d. 2,361,540

44. Assuming that the allowance for bad debts had a January 1, 2021 balance at
P127,000, what is the bad debt expense per audit for 2021?
a. 111,800 c. 106,800
b. 103,200 d. 101,800

PROBLEM 3:
Rivers Inc. extended a 4-year loan with a principal amount of P2.5M to Elaria Corp. on
January 1, 2021. The loan has a nominal interest of 12%, payable every December 31.
The company incurred direct origination costs which resulted to an effective interest
rate on the loan of 10%. Based on the company’s best estimate of 12-months expected
credit loss the present value of the credit loss at 10% was at P50,000 with the
probability of default estimated at 40%.

Interest was collected at the end of 2021. There was no change in the expected credit
loss.

Page 8 of 21 0915-2303213  resacpareview@gmail.com


AUDITING
ReSA Batch 44 – October 2022 CPALE Batch
24 September 2022  11:45 AM to 02:45 PM AUD Final Pre-Board Exam
While interest at the end of 2022 was collected, it was ascertained that there was a
significant increase in credit loss. The company decided to shift from a 12-month ECL
to a Life-time ECL. The present value of the life-time ECL was at P500,0000 with the
probability of default estimated at 50%.

No interest was collected at the end of 2023, it was ascertained based on the current
financial difficulties of the Elaria Corp. that certain concessions have to be agreed
upon if only to optimize recovery of the loan. The concessions included an extension
of the original maturity date to December 31, 2025 and a reduction in the principal
amount to P2.2M. Furthermore, the company agreed to forgive any current and future
interests on the loan.

Required:
45. What is the initial carrying value of the loans receivable on January 1, 2021?
a. 2,638,493 c. 2,689,539
b. 2,658,493 d. 2,624,343

46. What is the bad debt/credit loss/impairment loss on the loan to be recognized in
2022?
a. 250,000 c. 225,800
b. 230,000 d. 226,000

47. What is the bad debt/credit loss/impairment loss on the loan to be recognized in
2023?
a. 1,027,273 c. 777,273
b. 827,273 d. 752,273

48. The primary objective of a CPA’s observation of a client’s physical inventory


count is to:
a. Discover whether a client has counted a particular inventory item
or group of items
b. Provide an appraisal of the quality of the merchandise on hand on
the day of the physical count
c. Allow the auditor to supervise the conduct of the count in order to
obtain assurance that inventory quantities are reasonably accurate.
d. Ascertain that the physical count has been properly conducted in
accordance with the client’s internal control policies and
procedures and obtain direct knowledge that the inventory exists.

PROBLEM 4:
The following is a summary of your audit staff’s sales and purchases cut-off in line
with your firm’s audit of Flowers Corp. financial statements for the period ended
December 31, 2021. All sales were made at 30% gross profit based on sales. Commission
on consignment sales was agreed upon at 10%. Physical count of goods was conducted on
December 29. (All goods delivered on or before December 29 were excluded from the count
and all goods received on or before December 29 were include in the count)
SALES CUT-OFF
December Sales Journal Entries
SI Number Delivery Date Amount Remarks
1235 December 26 P60,000 FOB Destination
1236 December 27 89,000 FOB Shipping Point, in transit as of
12/31
1237 December 27 60,000 Shipped on consignment; 60% sold as of
12/31 according to consignee
1238 December 28 90,000 FOB Buyer
1239 December 29 50,000 FOB Buyer, in transit as of 12/31
1240 December 30 20,000 FOB Seller, in transit as of 12/31

January Sales Journal Entries


SI Number Delivery Date Amount Remarks
1241 December 31 P35,000 FOB Seller’s Warehouse, in transit as
of 12/31
1242 January 2 45,000 FOB Shipping Point
1243 January 3 25,000 Bill and Hold agreement executed in
Dec.
1244 January 3 90,000 FOB Seller

Page 9 of 21 0915-2303213  resacpareview@gmail.com


AUDITING
ReSA Batch 44 – October 2022 CPALE Batch
24 September 2022  11:45 AM to 02:45 PM AUD Final Pre-Board Exam
PURCHASE CUT-OFF
December Purchase Journal Entries
RR Number Receipt Date Amount Remarks
2309 December 28 P30,000 FOB Destination
2310 December 29 45,000 FOB Shipping point
2311 December 29 30,000 Received on consignment
2312 December 30 14,000 Purchased with resale agreement. Resale
price is higher than the purchase
price.
2313 December 30 21,000 FOB Destination

January Purchase Journal Entries


RR Number Receipt Date Amount Remarks
2314 December 31 P26,000 FOB Destination
2315 January 2 16,000 FOB Shipping point, In transit as of
12/31
2316 January 4 20,000 FOB Seller’s Warehouse

Required:
49. What is the net adjustment to accounts receivable?
a. 17,600 credit c. 32,400 debit
b. 14,000 credit d. 37,600 credit

50. What is the net adjustment to accounts payable?


a. 2,000 debit c. 28,000 debit
b. 12,000 credit d. 18,000 debit

51. What is the net adjustment to inventories?


a. 7,800 debit c. 12,000 debit
b. 28,800 debit d. 46,300 debit

PROBLEM 5:
Storm Corporation provided the following analysis of its Merchandise Inventory Item
AS234 in line with your audit of its financial statements for the period ended December
31, 2021:
Date # of Units Cost Sales Price
January 1, balance 20,000 P100
February Purchase 30,000 120
March Sale 25,000 P300
April Sale 10,000 290
May Purchase 40,000 125
June Purchase 20,000 140
July Sale 45,000 325
August Purchase 25,000 150
September Purchase 15,000 180
October Sale 30,000 300
November Sale 20,000 280
December Purchase 24,000 200
December Sale 15,000 260

Your review of subsequent events revealed that the sales price based on the latest
transaction was at P260. Cost to sell was estimated at P65. Assuming that the company
uses the allowance method to account for inventory write-down and that the allowance
at the beginning of the year was zero.

Required:
52. What is the correct carrying value of the inventories at year end assuming the
company uses periodic method of recording and FIFO cost formula?
a. 5,700,000 c. 5,800,000
b. 5,655,000 d. 5,556,000

53. What is the correct carrying value of the inventories at year end assuming the
company uses perpetual method of inventory recording and Average cost formula?
a. 5,655,000 c. 5,556,000
b. 5,292,225 d. 5,092,325

Page 10 of 21 0915-2303213  resacpareview@gmail.com


AUDITING
ReSA Batch 44 – October 2022 CPALE Batch
24 September 2022  11:45 AM to 02:45 PM AUD Final Pre-Board Exam
54. In your audit of a manufacturing client’s property, plant and equipment, you are
suspecting that there is a high likelihood of significant missing permanent assets
that are still recorded on the accounting records, an appropriate procedure is to
___________, this is necessary to support the financial statement assertion of
____________ over PPE.
a. Select sample from the asset master file and vouch to the documents
verifying the acquisition; existence
b. Select sample from the asset master file and vouch to the physical
assets; existence
c. Tracing the physical asset noted during the ocular inspection to the
asset master file; completeness
d. Tracing the acquisition documents to the asset master file;
completeness

PROBLEM 6:
You are auditing the property, plant and equipment accounts of your continuing client,
Pyke Inc. in line with your audit of its financial statements for the period ended
December 31, 2021. The following schedule was lifted from your prior-year working
papers:
Accumulated Depreciation Method;
December 31, 2020 Cost Depreciation Useful Lives
Land P2,000,000
150% Declining Balance,
Building 5,000,000 1,614,065 20 years
Office Equipment 3,000,000 1,963,636 SYD, 10 years
Factory Equipment 4,000,000 1,800,000 Straight-line, 10 years
All assets were acquired upon the company’s formation at the beginning of 2016. Assets
were estimated to have a salvage value of 10% of their original costs. In line with
your current year audit, the accountant furnished you the following schedules of
property additions and repairs and maintenance expense for the current period:
SCHEDULE OF PROPERTY ADDITIONS
Additions to Buildings:
Repainting costs incurred at the beginning of the year P125,000

Installation of elevator system at the beginning of the year 800,000


Additions to Office Equipment
Overhaul work necessary to bring a unit to its original working condition
incurred on March 31, 2021 90,000

Installment price of a replacement equipment acquired on June 30, 2021


to replace an old unit which had an original cost of P1.2M. The old unit
had to be retired due to an explosion totally damaging the unit on March
30, 2021. The installment price of the new equipment was payable in 3
annual payments starting June 30, 2022. This was supported by a non-
interest bearing note issued on June 30, 2021. Market rate for similar
debt security was at 6%. 1,500,000
Additions to Factory Equipment
Routinary repairs on factory equipment incurred at the end of the year 110,000

Cash paid on August 31, 2021 to acquire a new factory equipment


through a trade-in of an old unit with an original cost of P1.8M. The
trade-in allowance provided to the old unit was at P750,000. 1,200,000

SCHEDULE OF REPAIRS AND MAINTENANCE EXPENSE


Periodic annual repairs on building 150,236
Installation and commission costs incurred in
relation to the replacement office equipment 283,494
Installation of a smoke filtering device on the
building at the beginning of the year 540,000

Required:
55. What is the correct depreciation expense on the buildings for 2021, assuming that
any building improvements will have no estimated salvage value?
a. 333,945 c. 296,445
b. 346,445 d. 387,945

Page 11 of 21 0915-2303213  resacpareview@gmail.com


AUDITING
ReSA Batch 44 – October 2022 CPALE Batch
24 September 2022  11:45 AM to 02:45 PM AUD Final Pre-Board Exam
56. What is the correct depreciation expense on the office equipment for 2021?
a. 304,364 c. 321,818
b. 326,864 d. 307,091

57. What is the correct depreciation expense on the factory equipment for 2021?
a. 371,000 c. 342,000
b. 364,500 d. 376,500
58. The auditor’s audit program for substantive test procedures for auditing intangibles
include an examination of the schedule of current year charges to the research and
development expense account. Which of the following is correct?
a. The auditor shall test the propriety of the charges to the research
and development expense to support the existence assertion over
intangible assets.
b. The auditor shall trace the charges to the research and development
expense account to the supporting documents in support to the
valuation assertion over intangible assets.
c. Examining the entries to the research and development expense account
is necessary to ascertain whether no capitalizable costs to the
intangible asset account had been erroneously charged by the client
to the research and development expense account which is necessary
to support the completeness assertion over intangible assets.
d. Examining the entries to the research and development expense account
is necessary to evaluate whether these transactions have been
authorized to support the accuracy assertion of the recorded expense.
PROBLEM 7:
Hill Company reported patent at P1,760,000 as of December 31, 2021 before amortization.
Your audit investigation revealed the following:
• The patents were all acquired at the beginning of 2012 and are being amortized
over legal life.
• Legal fees in the successful defense of the patent amounting to P240,000 was
charged to the asset account on January 1, 2017 and was amortized over the
patents’ remaining life as of the said date.
• A patent with an original cost of P680,000 was ascertained to have a remaining
economic life of 4 years at the beginning of the current year.
• Another patent with an original cost of P720,000 has to be tested for possible
impairment at the end of the year. Estimates placed annual net cash flows from
the said patent at P52,079 over its remaining legal life. Market rate of interest
as at the end of the year was at 10%.

Required:
59. What is the amortization expense on patents in 2021?
a. 280,000 c. 203,500
b. 214,500 d. 204,955

60. What is the correct carrying value of patents as of December 31, 2021?
a. 1,340,500 c. 1,516,500
b. 1,380,500 d. 1,556,500

PROBLEM 8:
Waters Inc. reported the following portfolio of securities as of December 31, 2021, in
line with your audit of its financial statements:
FMV
Investment in equity securities: Cost (12/31/21)
Snow Corp. shares, 20,000 ordinary shares 1,200,000 1,400,000
Sand Inc. shares, 40,000 ordinary shares 2,500,000 2,800,000
Rivers Co. shares, 50,000 preference shares 6,100,000 6,400,000
Investment in debt securities
Pyke Corp. bonds, P2M face, 12%, due Dec. 11% 10%
31, 2024 effective rate effective rate
*all investments were acquired at the beginning of 2021
Additional information:
Outstanding Net Income Dividends
Shares for 2021 Declared in 2021
Snow Corp. 200,000 2,500,000 500,000
Sand Inc. 200,000 3,000,000 1,200,000
Rivers Co. 100,000 1,200,000 None
Pyke Corp. 250,000 4,340,000 2,000,000
Page 12 of 21 0915-2303213  resacpareview@gmail.com
AUDITING
ReSA Batch 44 – October 2022 CPALE Batch
24 September 2022  11:45 AM to 02:45 PM AUD Final Pre-Board Exam
Required:
61. Assuming that Waters Inc. has a business model which has an objective of collecting
contractual cash flows from its debt security investments, what is the correct
carrying value of all its investments as of December 31, 2021?
a. 12,648,874 c. 12,708,874
b. 12,759,474 d. 12,699,474

62. Assuming that Waters Inc. has a business model which has no objective of collecting
cash flows form its debt security investments and that, where applicable, gains or
losses on securities are reported in the profit or loss, what is the total amount
to be reported in the profit or loss for 2021 in relation to the investments?
a. 537,425 c. 1,414,251
b. 1,377,425 d. 1,427,425

63. You are preparing your audit program to audit a merchandising client’s trade payables
and other liabilities. Which of the following should not be ordinarily included in
your audit objective:
a. Detect accounts payable that are substantially past due
b. Verify that accounts payable were properly authorized
c. Ascertain the reasonableness of recorded liabilities
d. Determine that all existing liabilities at the balance sheet date
have been recorded.

64. Which of the following is the best audit procedure for determining existence of
unrecorded liabilities?
a. Examine confirmation requests returned by creditors whose accounts
appear on a subsidiary trial balance of accounts payable.
b. Examining entries before the balance sheet date in the purchases
journal or voucher register by tracing them to the supporting
documents ascertaining propriety of the recorded entries.
c. Sending blank confirmation letters to a sample of selected supplier
accounts with significant account balances as of the balance sheet
date.
d. Examine selected cash disbursements in the period subsequent to
year-end.

PROBLEM 9:
Stone Corp. reported the following liability balances as of December 31, 2021 in line
with your audit of its financial statements as of December 31, 2021:
Estimated liability for warranties 800,000
Salaries payable for compensated absences 720,000
Salaries payable for profit sharing bonus 167,442
Audit notes:
a. The estimated liability for warranties was the accrual made by the
company at the end of the year in relation to a 2-year “service-type”
warranty attached to each unit of the company’s product sold. Total
sales recorded for the year related to the said warranty was at P30M
which corresponds to 5,000 units sold. As a result of your
investigation, you ascertained that each 2-year warranty service would
have been sold separately at P400. The company estimates that only 40%
of the product sold will be returned for repairs in the first year and
another 60% will be returned for repairs the following year after sale.
Estimated cost to repair per unit was P250. Actual repair cost charged
to expense during the current year was at P450,000

b. The salaries for compensated absences was the accrual made at the end
of the previous year. The amount corresponds to 900 days of compensated
absences (600 earned in 2020, 300 earned in 2019).

Each employee is entitled 10 day-vacation leave and 10 day-sick leave for


every year of service. The company had 30 employees employed throughout
the current year. Unused leaves can be carried over 2 years thereafter
shall expire. During the current year, 550 days were exercised by the
employees (320 days earned in 2020). Salary rate increased by 10% during
the current year.
Page 13 of 21 0915-2303213  resacpareview@gmail.com
AUDITING
ReSA Batch 44 – October 2022 CPALE Batch
24 September 2022  11:45 AM to 02:45 PM AUD Final Pre-Board Exam
c. The salaries payable for profit sharing bonus was computed by the
company based on 10% of the unadjusted net income after bonus and after
25% income tax.

Required:
65. What is the correct liability for warranties as of December 31, 2021 assuming sales
were made evenly throughout the year?
a. 1,500,000 c. 1,125,000
b. 800,000 d. 937,500

66. What is the correct salaries payable for compensated absences as of December 31,
2021?
a. 836,000 c. 704,000
b. 774,400 d. 720,000

67. What is the correct salaries payable for profit-sharing bonus as of December 31,
2021?
a. 110,512 c. 119,721
b. 114,809 d. 118,605

PROBLEM 10:
On January, 2019 GOT Corp. grants each of its 100 employees in the sales department
share options. The share options will vest at the end of 2021, provided that the
employees remain in the entity’s employ and provided that the sales increase by at least
100% by 2021. Actual sales in 2018 (base year) is 2M units. If the sales volume increase
by an average of 100% to 120% by 2021, each employee will receive 200 options each. If
sales volume increase by 121%-150% by 2021, each employee will receive 300 options each.
If sales volume increase by more than 150% by 2021, each employee will receive 400
options each. Five options plus P120 shall entitle the holder to acquire one ordinary
shares (P100 par) at any time up to December 31, 2023.

On the grant date, the company estimates that the share options have a fair value of
P21 per option. There has been a 30% average increase in annual sales for the past three
years and that the company expects the same pattern during the vesting period.

The following information are deemed relevant for your analysis:

Actual employees Estimated additional Actual


Year leaving the employees who will Sales
company leave by the end 2021 (Units)
2019 4 5 2,500,000
2020 2 4 3,500,000
2021 9 - 5,100,000

Requirements:
68. What is the compensation expense in 2019?
a. 127,400 c. 191,100
b. 254,800 d. 134,400

69. What is the compensation expense in 2021?


a. 378,000 c. 363,000
b. 238,000 d. 336,000

70. Assuming that 40% of the options granted to employees were exercised, the entry to
record the exercise shall require a credit share premium at:
a. 320,000 c. 280,000
b. 340,000 d. 312,000

- END of EXAMINATION -

Page 14 of 21 0915-2303213  resacpareview@gmail.com


AUDITING
ReSA Batch 44 – October 2022 CPALE Batch
24 September 2022  11:45 AM to 02:45 PM AUD Final Pre-Board Exam

ANSWERS & SOLUTIONS/CLARIFICATIONS


1 D 26 B 51 B
2 C 27 B 52 B
3 D 28 C 53 D
4 C 29 B 54 B
5 B 30 D 55 D
6 A 31 B 56 A
7 B 32 A 57 B
8 B 33 B 58 C
9 C 34 A 59 C
10 A 35 C 60 A
11 D 36 C 61 C
12 B 37 A 62 D
13 C 38 C 63 A
14 C 39 A 64 D
15 A 40 B 65 A
16 A 41 A 66 B
17 C 42 A 67 B
18 D 43 D 68 A
19 C 44 D 69 D
20 A 45 A 70 B
21 B 46 C
22 C 47 D
23 C 48 D
24 B 49 A
25 A 50 A

Page 15 of 21 0915-2303213  resacpareview@gmail.com


AUDITING
ReSA Batch 44 – October 2022 CPALE Batch
24 September 2022  11:45 AM to 02:45 PM AUD Final Pre-Board Exam

Page 16 of 21 0915-2303213  resacpareview@gmail.com


AUDITING
ReSA Batch 44 – October 2022 CPALE Batch
24 September 2022  11:45 AM to 02:45 PM AUD Final Pre-Board Exam

Page 17 of 21 0915-2303213  resacpareview@gmail.com


AUDITING
ReSA Batch 44 – October 2022 CPALE Batch
24 September 2022  11:45 AM to 02:45 PM AUD Final Pre-Board Exam

Page 18 of 21 0915-2303213  resacpareview@gmail.com


AUDITING
ReSA Batch 44 – October 2022 CPALE Batch
24 September 2022  11:45 AM to 02:45 PM AUD Final Pre-Board Exam

Page 19 of 21 0915-2303213  resacpareview@gmail.com


AUDITING
ReSA Batch 44 – October 2022 CPALE Batch
24 September 2022  11:45 AM to 02:45 PM AUD Final Pre-Board Exam

Page 20 of 21 0915-2303213  resacpareview@gmail.com


AUDITING
ReSA Batch 44 – October 2022 CPALE Batch
24 September 2022  11:45 AM to 02:45 PM AUD Final Pre-Board Exam

Page 21 of 21 0915-2303213  resacpareview@gmail.com

You might also like